You are on page 1of 238

Calculus and Linear Algebra 18MAT11

CAMBRIDGE INSTITUTE OF TECHNOLOGY, BENGALURU-36

DEPARTMENT OF BASIC SCIENCES

COURSE: CALCULUS AND LINEAR ALGEBRA

COURSE CODE: 18MAT11

Prepared By
1. Dr. Shanthi S R
2. Dr. Suma S P
3. Mr. Ramesh T
4. Ms. Rekha J
5. Dr. Akkanagamma M
6. Mr. Aravind H R
7. Mr. Amaresha N
8. Ms. Mamatha V
9. Mr. Venugopal
10. Mr Kiran Gowda

Department of Mathematics Page 1


Calculus and Linear Algebra 18MAT11

CAMBRIDGE INSTITUTE OF TECHNOLOGY, BENGALURU-36


B.E.Syllabus for 2018-2022
CALCULUS AND LINEAR ALGEBRA
(Common to all branches)
[As per Choice Based Credit System (CBCS) scheme]
(Effective from the academic year 2018-19)
Course Code : 18MAT11 CIE Marks : 40
Contact Hours/Week : 05(3L+2T) SEE Marks: 60
Total Hours:50 (8L+2T per module) Exam Hours:03
Semester : I Credits: 04 (3:2:0)
Course Learning Objectives: This course Calculus and Linear Algebra (18MAT11) will
enable students:
• To familiarize the important tools of calculus and differential equations that is
essential in all branches of engineering.
• To develop the knowledge of matrices and linear algebra in a comprehensive manner.

MODULE-I
Differential Calculus-1: Review of elementary differential calculus, Polar curves - angle
between the radius vector and tangent, angle between two curves, pedal equation. Curvature
and radius of curvature- Cartesian and polar forms; Centre and circle of curvature (All
without proof-formulae only) –applications to evolutes and involutes. (RBT Levels: L1 &
L2)
MODULE-II
Differential Calculus-2: Taylor‟s and Maclaurin‟s series expansions for one variable
(statements only), indeterminate forms - L‟Hospital‟s rule. Partial differentiation; Total
derivatives-differentiation of composite functions. Maxima and minima for a function of
two variables; Method of Lagrange multipliers with one subsidiary condition. Applications
of maxima and minima with illustrative examples. Jacobians-simple problems. ( RBT
Levels: L1 & L2)
MODULE-III Integral Calculus: Review of
elementary integral calculus.Multiple integrals: Evaluation of double and triple integrals.
Evaluation of double integrals-change of order of integration and changing into polar co-
ordinates. Applications to find area volume and centre of gravity Beta and Gamma functions:
Definitions, Relation between beta and gamma functions and simple problems.( RBT Levels:
L1 & L2)
MODULE-IV
Ordinary differential equations(ODE‟s)of first order:Exact and reducible to exact differential
equations. Bernoulli‟s equation. Applications of ODE‟s-orthogonal trajectories, Newton‟s law
of cooling and L-R circuits. Nonlinear differential equations: Introduction to general and
singular solutions ; Solvable for p only; Clairaut‟s and reducible to Clairaut‟s equations
only.(RBT Levels: L1,L2 and L3)

Department of Mathematics Page 2


Calculus and Linear Algebra 18MAT11

MODULE-V
Linear Algebra: Rank of a matrix-echelon form. Solution of system of linear equations –
consistency. Gauss-elimination method, Gauss –Jordan method and Approximate solution by
Gauss-Seidel method. Eigen values and eigenvectors-Rayleigh‟s power method.
Diagonalization of a square matrix of order two. (RBT Levels: L1,L2 and L3)

Text Books:
1. B.S. Grewal: Higher Engineering Mathematics, Khanna Publishers, 43rd Ed., 2015.
2. E. Kreyszig: Advanced Engineering Mathematics, John Wiley & Sons, 10th Ed.(Reprint),
2016.
Reference books:
1. C.Ray Wylie, Louis C.Barrett : “Advanced Engineering Mathematics", 6th Edition,
2. McGraw-Hill Book Co., New York, 1995.
2. James Stewart : “Calculus –Early Transcendentals”, Cengage Learning India Private Ltd.,
2017.
3. B.V.Ramana: "Higher Engineering Mathematics" 11th Edition, Tata McGraw-Hill, 2010.
4. Srimanta Pal & Subobh C Bhunia: “Engineering Mathematics”, Oxford University
Press,3rd Reprint, 2016.
5. Gupta C.B., Singh S.R. and Mukesh Kumar: “Engineering Mathematics for Semester I &
II”, Mc-Graw Hill Education (India) Pvt.Ltd., 2015.
Course Outcomes: On completion of this course, students are able to:

CO1: Apply the knowledge of calculus to solve problems related to polar curves and its
applications in determining the bentness of a curve.

CO2: Learn the notion of partial differentiation to calculate rates of change of multivariate
functions and solve problems related to composite functions and Jacobians.

CO3: Apply the concept of change of order of integration and variables to evaluate
multiple integrals and their usage in computing the area and volumes.

CO4 : Solve first order linear/nonlinear differential equation analytically using


standard methods

CO5 : Make use of matrix theory for solving system of linear equations and compute
eigenvalues and eigenvectors required for matrix diagonalization process.

Department of Mathematics Page 3


Calculus and Linear Algebra 18MAT11

MODULE-1

Differential calculus-1

Index

I. Polar Curves-Angle Between Radius Vector And Tangent

II. Angle between two polar curves

III. Pedal Equation

IV. Curvature and Radius of curvature- Cartesian and polar forms.

V. Evolutes and Involutes

Department of Mathematics Page 4


Calculus and Linear Algebra 18MAT11

Introduction

In day to day life we are often interested in the extent to which a change in one quantity
affects a change in another related quantity. This is called a rate of change. For example, if you
own a motor car you might be interested in how much a change in the amount of fuel used
affects how far you have travelled. This rate of change is called fuel consumption. If your car
has high fuel consumption then a large change in the amount of fuel in your tank is
accompanied by a small change in the distance you have travelled. Sprinters are interested in
how a change in time is related to a change in their position. This rate of change is called
velocity. Other rates of change may not have special names like fuel consumption or velocity,
but are nonetheless important. For example, an agronomist might be interested in the extent to
which a change in the amount of fertilizer used on a particular crop affects the yield of the crop.
Economists want to know how a change in the price of a product affects the demand for that
product. Differential calculus is about describing in a precise fashion the ways in which related
quantities change.

Many of the practical situations that engineers have to analyze involve quantities that are
varying. Whether it is the temperature of a coolant, the voltage on a transmission line or torque
on turbine blade, the mathematical tools for performing such analyses are the same. One of the
most successful of these is calculus, which involves two fundamental operations:
differentiation and integration.

Differentiation is a mathematical technique for analyzing the way in which functions


change. In particular, it determines how rapidly a function is changing at any specific point. As
the function in question may represent the magnetic field of a motor, the voltage across a
capacitor, the temperature of a chemical mix, etc., it is often important to know how quickly
these quantities change. For example, if the voltage on an electrical supply network is falling
rapidly because of a short circuit, then it is necessary to detect this system should not take
action for normal voltage fluctuations and so it is important to distinguish different types of
rates of change. Another example would be detecting a sudden rise in the pressure of a
fermentation vessel and taking appropriate action to stabilize the pressure.

Department of Mathematics Page 5


Calculus and Linear Algebra 18MAT11

POLAR CURVE

We are conversant in representing the position of a point p  x, y  in the Cartesian form

The representation of any point P in the form  r,  is polar form.

Let  x, y  and  r,  respectively represent the Cartesian and polar co-ordinates of any point P
in the plane where the Origin „O‟ is taken as a pole

And the X axis is taken as initial line

From the figure we have OQ=x, PQ=y

Also from the right angled triangle OQP we have

OQ x
cos     x  r cos 
OP r

QP y
sin     y  r sin 
OP r

 y
 x 2  y 2  r 2 ;   tan 1  
x

I. Angle Between Radius Vector And Tangent


dr
The angle between radius vector and tangent is given by tan   r
d

Problems

1) Find the angle between the radius vector and the tangent for the curve r  a(1  Cos ) .

Given, r  a(1  Cos )

dr r
 asin . tan 
d  dr 
 
 d 


2sin2
a(1  cos ) 2
 tan  
asin  
2sin cos
2 2

Department of Mathematics Page 6


Calculus and Linear Algebra 18MAT11

 
 tan  tan 
2 2

2a
2) Find the angle between the radius vector and the tangent for the curve  1  cos .
r

2a
Given,  1  cos
r

2a dr dr r 2 sin r
 2   sin    tan 
r d d 2a  dr 
 
 d 

r 1  cos
tan  
 r Sin 
2
sin
 
 2a 

2cos 2
   
tan  2  cot  tan   
  2  2 2
2 sin cos
2 2

 
  .
2 2

3) Find the angle between the radius vector and the tangent for the
dr
curve mr m 1  a m (msinm  mcosm ),
d

Given, r m  am  cosm  sinm 

Department of Mathematics Page 7


Calculus and Linear Algebra 18MAT11

dr
mr m 1  a m  msinm  mcosm  ,
d
dr   sinm  cosm 
 am  ,
d  r m 1 
r r
 tan   m ,
 dr   a   sinm  cosm  
   
 d   r m 1 
rm
 ,
 sinm  cosm
a m  cosm  sinm 
 m ,
a   sinm  cosm 

Dividing both numerator and denominator by cosm , we get

1  tanm
tan  ,
1  tanm
 
tan  tan   m  ,
4 

  m .
4

4) Find the angle between the radius vector and the tangent for the curve r 2Cos 2  a s .

Given, r 2Cos 2  a 2

Differentiate w.r.to 

 dr 
r 2  2Sin 2   Cos 2  2r   0,
 d 

 by 2r
dr
rsin 2  cos 2 0
d

Department of Mathematics Page 8


Calculus and Linear Algebra 18MAT11

dr
cos 2  rsin 2

dr r
 rtan 2  tan 
d  dr 
 
 d 
r
 tan   cot 2
rtan 2
 
 tan  tan   2 
2 

  2
2


5) Find the angle between the radius vector and the tangent for the curve r  aSec 2 .
2


Given, r  asec 2 .
2

Differentiate w.r.to

dr   1
 2asec 2 t an
d 2 2 2

dr   1
 2asec 2 tan
d 2 2 2
dr  
 asec 2 tan
d 2 2

r
 tan 
 dr 
 
 d 


asec 2
tan  2
2 
asec tan
2 2

 tan  cot
2

Department of Mathematics Page 9


Calculus and Linear Algebra 18MAT11

  
 tan  tan   
 2 2
 
  .
2 2

6) Show that for the curve r  ac Cot , Where  is a constant, the radius vector is inclined at a

Constant angle to the tangent at every point.

Given, r  ae Cot 

Differentiate w.r. to  ,

dr
 ae cot  cot ,
d

r
 tan 
 dr 
 
 d 
ae Cot
 tan   cot
ae cot
 tan  tan

    , a constant

II. Angle between two polar curves

Consider two curves r  f   and r  g   . Let the tangents to the curves makes angles 1 and
 2 with the initial line respectively.

Therefore Angle between curves  1  2

Orthogonal curves


Two curves are said to be orthogonal if 12  or tan1  tan2  1.
2

Department of Mathematics Page 10


Calculus and Linear Algebra 18MAT11

tan1  tan2
Note: tan 1  2  .
1  tan1tan2

1) Find the angle of intersection between the curves r  a 1  cos  and r  2acos .

Given, r  a 1  cos  , r  2asin ,

dr dr
 asin ,  2asin ,
d d

r r
 tan   tan 
 dr   dr 
   
 d   d 

a 1  cos  2acos
 tan   tan 
asin 2asin


2sin2
 tan  2 ,  tan  cot ,
 
2sin cos
2 2

  
 tan  tan ,  tan  tan    
2 2 

 
 1  ,  2  
2 2

Angle between curves  1  2

      
Angle between curves                  1
2 2  2 2 2 2

At the point of intersection,

a (1  cos )  2acos ,
1  cos  2acos ,
3cos  1,
1 1
cos     cos 1  
3 3
1 
1
 Angle between curves   cos 1  
2 2
Department of Mathematics 3 Page 11
Calculus and Linear Algebra 18MAT11

2) Find the angle of intersection between the curves r  2sin and r  sin  cos .

Given, r  2sin r  sin  cos .

dr dr
 2Cos ,  cos  sin ,
d d

r r
 tan   tan 
 dr   dr 
   
 d   d 

2sin sin  cos


tan1  tan2    Nr. and Dr. by cosθ 
2cos cos  sin

tan  1  
 tan1  tan tan2   tan     ,
1  tan 4 


 1   ,  2  
4

Angle between curves  1  2 ,

 
Angle between curves        ,
4 

Angle between curves 
4

3) Find the angle of intersection between the curves r 2 Sin2  4 and r 2  16Sin2 .

Given, r 2 sin2  4 r 2  16sin2 .

dr dr
r 2 2cos 2  sin2  2r 0 2r  32cos 2
d d

dr rcos 2 dr 16cos 2
  rcot 2 
d sin2 d r

r r
 tan   tan 
 dr   dr 
   
 d   d 

Department of Mathematics Page 12


Calculus and Linear Algebra 18MAT11

r r r2
tan  tan   ,
 rcot 2   16cos 2  16cos 2
 
 r 

16sin2
tan  tan2 tan   tan2
16cos 2

 1  2 2  2

 Angle between curves  1  2 ,

Angle between curves  2  2  4 ,

Angle between curves  4       1

At the point of intersection,

4
 16 sin 2
sin 2
1
Sin 2 2 
4
1
Sin 2 
2
1
2  sin 1  
2
 
2   .
6 12

  
Angle between curves  4  
 12  3

4) Find the angle of intersection between the curves r n  an Sec  n    and


r n  bn Sec  n    .

Given, r n  an Sec  n    r n  bn Sec  n    .

dr dr
nr n1  a n Sec  n     Tan  n     n, nr n1  bn Sec  n     Tan  n     n,
d d

Department of Mathematics Page 13


Calculus and Linear Algebra 18MAT11

dr a Sec  n     Tan  n    dr b Sec  n     Tan  n   


n n

 ,  ,
d r n 1 d r n 1

r r
 tan   tan 
 dr   dr 
   
 d   d 

r r
tan  tan 
 a Sec  n     tan  n    
n
 a Sec  n     tan  n    
n

   
 r n 1   r n 1 

rn rn
 tan  ,  tan  ,
a n Sec  n     tan  n    a n Sec  n     tan  n   

a n Sec  n    a n Sec  n   
 tan  n ,  tan  n
a Sec  n     tan  n    a Sec  n     tan  n   

 tan  cot  n    ,  tan  cot  n   

   
 tan  tan    n      tan  tan    n    
2  2 

 
 1    n    .  2    n    .
2 2

 Angle between curves  1  2 ,

  
Angle between curves    n        n    ,
2 2 

Angle between curves     .

a
5) Find the angle of intersection between the curves r  a log  and r  .
log 

a
Given, r  a log  , r ,
log 

dr a dr a 1
 ,   ,
d  d  log   
2

Department of Mathematics Page 14


Calculus and Linear Algebra 18MAT11

r r
 tan   tan 
 dr   dr 
   
 d   d 

 tan1 
a log 
tan2 
 a / log  
a  a 
   2 
     log   
 

 tan1   log  ,  tan2   log 

tan1  tan2
 tan 1  2 
1  tan1tan2
 log     log  
tan 1  2 
1   log    log  

2 log 
tan 1  2        1
1   log  
2

At the point of intersection,

a
a log   ,
log 
  log    1,
 log   1,
  e.
2e
 1  tan   2  ,
1  e2
2e
 tan 1  2  ,
e 1
2

 2e 
 1  2  tan 1  2  .
 e 1 

a a
6) Find the angle of intersection between the curves r  and r  .
1 1 2

a a
Given, r  , r ,
1 1 2

Department of Mathematics Page 15


Calculus and Linear Algebra 18MAT11

dr 1    a  a dr a
 ,   2 ,
d 1    d 1   
2 2

dr a dr 2a
 ,  ,
d 1   2 d 1   2 
2

r r
 tan   tan 
 dr   dr 
   
 d   d 

 a   a 
   2 
 tan1  
1 
 tan2   1 
 a   
    2a 
 1   2   1   2 2 
   

1 2
 tan1   1     tan 2  
2

tan1  tan2
 tan 1  2  ,
1  tan1tan2

1 2
 1    
 tan 1  2  2       1
 1 2 
1   1     
 2 

At the point of intersection,

a a
 ,
1 1 2
   3  1 ,
  3  1,
  1
2 1
 1  tan 1  2  ,
1  2 1
 tan 1  2  3 ,
 1  2  tan 1 3 .

Department of Mathematics Page 16


Calculus and Linear Algebra 18MAT11

7) Show that the curves r n  a n Cosn and r n  bn Sinn are orthogonal.

Given, r n  a n Cosn , r n  bn Sinn

dr dr
nr n1  a n  nSinn  , nr n 1  a n  nCosn  ,
d d

dr a n Sinn dr b n Cosn
 ,  ,
d r n 1 d r n 1

r r
 tan   tan 
 dr   dr 
   
 d   d 

r r
 tan1   tan2 
 a Sinn 
n
 b cosn 
n

 n 1   n 1 
 r   r 

rn rn
 tan1    tan2  ,
a n Sinn b n cosn

a nCosn b n Sinn
 tan1    tan2 
a n Sinn b nCosn

 tan1  cotn  tan2  tann

 
 tan1  tan   n   2  n
2 


 1   n .
2

 1  2  .
2

Hence the given curves are orthogonal.

8) Show that the curves r  a 1  Cos  and r  b 1  Cos  are orthogonal.

Given, r  a 1  Cos  , r  b 1  Cos  ,

dr dr
 a   Sin  ,  bCos ,
d d

Department of Mathematics Page 17


Calculus and Linear Algebra 18MAT11

r r
 tan   tan 
 dr   dr 
   
 d   d 

r r
 tan  tan 
 a Sinn 
n
 b cosn 
n

 n 1   n 1 
 r   r 

rn rn
 tan   tan 
a n Sinn b nCosn

a nCosn b n Sinn
 tan   tan  ,
a n Sinn b nCosn

 tan  cotn , tan  tann ,

 
 tan  tan   n  2  n
2 

 
 1   n  1  2  .
2 2

Hence the given curves are orthogonal.

9) Show that the curves r  a 1  Sin  and r  b 1  Sin  are orthogonal.

Given, r  a 1  Sin  , r  b 1  Sin  ,

dr dr
 aCos ,  bCos ,
d d

r r
 tan   tan 
 dr   dr 
   
 d   d 

a 1  Sin  b 1  Sin 
 tan  ,  tan  ,
aCos bCos

Department of Mathematics Page 18


Calculus and Linear Algebra 18MAT11

 tan1 
1  Sin  ,
aCos
1  Sin  ,  Tan 1  Sin  1  Sin 
 tan2   Tan2   ,
Cos Cos Cos
1

 tan1  tan2 
1  Sin  
2

 tan1  tan2  1
Cos 2

Hence the given curves are orthogonal.

Exercise

Find the angle of intersection for the curves


1. r  a and r  2a cos ,
2. r n  a n sin n  cos n  r n  a n sin n
a b
3. r  r
1  cos 1  cos

Show that the following pair of curves interest orthogonally


4 . r  ae and re   b
5. r 2 sin 2  a 2 and r 2 cos 2  b 2 ,

III. Length of the perpendicular from the pole to the tangent

Consider a curve r  f   and a point „p‟ on it. Let  be the angle between the radius vector
and the tangent and  be the angle between the tangent and the initial line respectively. Draw
a perpendicular OQ from the pole on to the tangent at „p‟.

Department of Mathematics Page 19


Calculus and Linear Algebra 18MAT11

From OPQ,

P  rSin

 P 2  r 2 Sin 2 ,

1 1
 2
 2 Co sec2  ,
P r

 2 1  Cot 2  ,
1 1
 2
P r

1 1 1 
  2 1  ,
P 2
r  tan2 

d
But tan   r 
dr

1  
2
1 1  dr 
2
 2 
1 2  d  ,
P r  r 

2
1 1 1  dr 
  2  4         1
p 2
r r  d 

1
Put u 
r

du 1 dr
  2 ,
d r d

Squaring both sides,


2
 du  1  dr 
 d    d  ,
r4

2
1  du 
 1  2  u 2           2 
P  d 

IV. Pedal equation or p-r equation

An equation containing p and r only is called p-r equation.

Department of Mathematics Page 20


Calculus and Linear Algebra 18MAT11

1) Find the pedal equation of r n  a n Cosn

Given, r  a n Cosn .
dr
nr n1  a n  nSinn  ,
d

dr a n Sinn
 ,
d r n 1
r
 tan 
 dr 
 
 d 
r
 tan  n
 a Sinn 
 n 1 
 r 
rn
 tan  
a n Sinn

a nCosn
 tan  
a n Sinn
 
 tan  Cotn  tan   n 
2 

  n .
2
Clearly, p  rSin ,
 
 p  rSin   n  ,
2 
 p  rCosn ,
p
 Cosn  ,
r
But r  a Cosn
p
 r nan 
r
n 1
 r  an p

2) Find the pedal equation of r  a 1  Cos  ,

Given, r  a 1  Cos ,  ,

Department of Mathematics Page 21


Calculus and Linear Algebra 18MAT11

dr
 aSin ,
d
r
 tan 
 dr 
 
 d 
a 1  Cos 
 tan 
aSin

2Sin 2
 tan  2
 
2Sin Cos
2 2

 tan  tan
2

 .
2

3) Find the pedal equation of r m Cosm  a m .

Given, r mCosm  a m ,
Clearly, p  rSin .

 p  rSin
2
 p
 Sin 
2 r
But , r  a 1  Cos 

 r  a  2Sin 2
2
2
p
 r  2a
r2
 r 3  2ap 2
am
 rm  ,
Cosm
 r m  a m Secm ,

Differentiate w.r.t 
dr
mr m1  a m Secm  tanm  m
d
dr a m Secm  tanm

d r m 1
Department of Mathematics Page 22
Calculus and Linear Algebra 18MAT11

r r
 tan   m
 dr   a Secm  tanm 
   
 d   r m1 
rm
 tan  m
a Secm  tanm
a m Secm
 tan  m
a Secm  tanm
 tan  Cotm ,
 
 tan  tan   m 
2 

  m .
2
Clearly, p  rSin .
 
 p  rSin   m 
2 
 p  rCosm
p
 Cosm 
r

But r m Cosm  a m ,
p
 rm   am ,
r
 r m 1 p  a n


4) Find the pedal equation of r  a cosec 2 .
2


Given r  aCo sec2
2
Differentiate w.r.t. 
dr   1
 2aCo sec2  Cot 
d 2 2 2
dr  
 a  Co sec2  Cot
d 2 2
r
 tan 
 dr 
 
 d 

Department of Mathematics Page 23


Calculus and Linear Algebra 18MAT11


aCo sec2
tan  2
 
aCo sec 2
Cot
2 2


 tan  tan
2

 
 tan  tan   ,
2

  .
2
Clearly,  p  rSin ,
 
 p  rSin   ,
2

 p  rSin ,
2

 p  rSin , ( p is positive always)
2
 p
 Sin  ,
2 r

But r  aCo sec2 ,
2

 r  aCo sec2 ,
2
r
r a ,
p2
 p 2  ar

5) Find the pedal equation of r  ae Cot ,

Given, r  ae Cot 

Differentiate w.r.to  ,

dr
 ae Cot  Cot ,
d

Department of Mathematics Page 24


Calculus and Linear Algebra 18MAT11

r
 tan 
 dr 
 
 d 

 tan  tan

  

Clearly, p  rSin.

 p  rSin

6) Find the pedal equation of r 2  a 2 Sec2 .

Given, r 2  a 2 Sec2 .

Differentiate w.r.to  .

dr
2r  2a 2 Sec  tan2 ,
d

dr a 2 Sec 2  Tan 2
  ,
d r

r
 tan 
 dr 
 
 d 

r
 tan 
 a Sec 2  tan2 
2

 
 r 

r2
 tan  ,
a 2 Sec 2  tan2

a 2 Sec 2
 tan  2
a Sec  tan 2

7) Show that the pedal equation of the curve r n  a n Sinn  bnCosn is

p 2  a 2 n  b2 n   r 2 n  2 .

Given, r n  an Sinn  b Cosn

Department of Mathematics Page 25


Calculus and Linear Algebra 18MAT11

Differentiate w.r.to  .

dr
nr n 1  a Cosn  n  b Sinn  n.
d

dr aCosn  b n Sinn
  ,
d r 1

We have,

1 1 1  dr 
  4  ,
p r r  d 

2
1 1 1  a nCosn  b Sinn 
  2 4 
p r r  r n 1 

1 1 1  a Cosn  b Sinn 
   ,
p2 r 2 r 4 r 2 n2

 2 n 1 r 2 n  (a n Cosn  b n Sinn ) 2  ,
1 1
 2
p r


1
p
1
 2 n 1
r
 r    a Cosn  b Sinn  2 ,
n 2 n


1
p 2
1
 2n2
r
 a Sinn  b Cosn    a Cosn  b Sinn   ,
n 2 n n 2

 2 n  2 a 2 n Sin 2 nb  b 2 n Cos 2 n  2a n b n Sinn Cosn  a n Cos n  b 2 n Sin 2 n  2a n b n Sinn Cosn 


1 1
 2
p r

p
1
r
1

 2  2 n  2 a 2 n  Sin n  Cos 2 n   b 2 n  Sin 2 n  Cos 2 n  ,

 2n2  a 2n  b2n  ,
1 1
 2
p r

 p 2  a 2 n  b2 n   r 2 n  2 .

Department of Mathematics Page 26


Calculus and Linear Algebra 18MAT11

1 2 a
8) Obtain the pedal equation of   r  a 2  Cos 1   .
a r

1 2 a
Given,   r  a  Cos 1   .
a r

Differentiate w.r.to  ,

1 dr 1  a  dr
1  2r   2  ,
a2 r  a2 2 d  a   r  d
2

1  
r

r dr a dr
1  ,
a r  a d r r  a d
2 2 2

1 dr  r a 
1   ,
r 2  a 2 d  a r 

1 dr  r 2  a 
1  ,
r 2  a 2 d  ar 

r  a 2 dr
1 ,
ar d

dr ar
 ,
d r2  a

We have
2
1 1 1  dr 
    ,
p r r  d 

2
1 1 1  ar 
 2    
p r r  r  a2 

1 1 1  a2r 2 
 2  2 4 2 
p r r  r  a2 

1 1  a2 
  2 1  2
 r a 
2 2
p r

Department of Mathematics Page 27


Calculus and Linear Algebra 18MAT11

1 1  r 2  a  a2 
   
p2 r 2  r  a2 

1 1 r 
   
p r  r  a2 

1 1
  2
p 2
r  a2

 p2  r 2  a2

l 1  2l 
9) Show that the pedal equation of  1  e cos is 2   e 2   1 .
r l  r 

l
Given,  1  e cos
r

Differentiate w.r.to 

l dr
-  eSin ,
r 2 d

dr er 2 Sin
 ,
d l
2
 dr  e2 r 4
   Sin 2 ,
 d   l 2

2
 dr  e2 r 4
   Sin ,
 d  l2

2
 dr  e2 r 4
2 
   1  Cos 2  ,
 d  l

l
But  1  eCos ,
r

1 l 
 Cos    1 ,
er 

Department of Mathematics Page 28


Calculus and Linear Algebra 18MAT11

 dr 
2
e2 r 4  1 l  
2

   2 1  2   1  ,
 d  l  e  r  

We have
2
1 1 1  dr 
 2  4   ,
 d 
2
p r 4

1 1 1 e2 r 4  1 l  
2

 2  2  4. 2 1  2   1  ,
p r r l  e  r  

1 1 e2  1  l2 2l  
   2 1  2  2  1   ,
p 2
r 2
l  e r r  

1 1 e2 1  l 2 2l 
 2  2  2  2  2 1 ,
p r l l r r 

1 1  2l 
  2   1
p 2
l r 

Exercise

Find the pedal equations of the following curves

2a
1.  1  cos 
r .

2. r 1  cos   2a

3. r m  am  cos m  sin m 

Department of Mathematics Page 29


Calculus and Linear Algebra 18MAT11

V. Curvature and Radius of curvature- Cartesian and polar forms.

Consider a curve y =f(x). Let  and    be the angles made by the tangents drawn at „P‟
and „Q‟ respectively. The rate Consider a curve y = f(x). Let  and    be the angles
made by the tangents drawn at „P‟ and „Q‟ respectively. The rate of change of  with respect
to the arc length „S‟ is called the curvature of the curve.

 d
i.e, k = lim  .
 s 0 ds ds

Radius of curvature: The reciprocal of curvature is called the radius of curvature.

1 ds
 .
i.e, p = k d 

Radius of curvature in Cartesian form:

3/2
  dy  2 
 1    
 dx  

d 2x
dy 2

Radius of curvature in parametric form:

Consider a parametric curve x=x(t) and y =y(t).

The radius of curvature is given by,

p
1  y 
2

yn

Department of Mathematics Page 30


Calculus and Linear Algebra 18MAT11

OR

 x  y 2  dy dx d 2x d2y
32

Where y  & x  and x  


2
& y 
 . dt dt dt 2 dt 2
xy  yx

Radius of curvature in polar form:

r  r12 
3
2 2
dr d 2r
 2 2 where r1  & r2  2
2r1  r  rr2 dt dt

Radius of curvature in pedal form:

dr
pr
dp

2 2 2 1
1) Show that the radius of curvature of the curve x 3  y 3  a 3 is 3  axy  3 .

2 2 2

Given , x  y  a 3 3 3

Differentiate w.r.to.x.
1 1
2 3 2 3 '
x  y y  0,
3 3
1 1
 x 3  y 3 y '  0,

1 1
 y 3 y'   x 3 ,

1
3
x
 y'   1
,
3
y

1
y3
 y'   1
x3

1 1
 y'   y 3 x 3 ,

Department of Mathematics Page 31


Calculus and Linear Algebra 18MAT11

Differentiate w.r.to x

  1  
1 4 1 2
1
y"    y 3 .   x 3   x 3 . y 3 y '  ,
  3  3 

1   1 1  
1 4 1 2

 y ''    y 3 x 3  x 3 y 3   y 3 x 3   ,
3    

1   
1 4 2 1
 
 y"   y 3 x 3  x 3 y 3  ,
3  

1  3 3  3 
4 1 2 2
 y"  x y  y  x3 ,
3  

4 1 2
1  
 y  x 3 y 3a3
"

p
1  y '2 2

y"
3
  1 2 2

  y3  
1    1  
  x3  
  
 p 4 1 2
1 3 3 3
x y a
3
3

4 1 2  
2 2

 p  3x y a 3 3

3 1  3  ,
y
 2 
 3 
 x 
3

2  3 2
2 2

4 1
 p  3x 3 y 3 a 3   ,
 x y 3

2 
 x3 
 

Department of Mathematics Page 32


Calculus and Linear Algebra 18MAT11

2  3 2
2
4 1
 p  3x 3 y 3 a 3  2  ,
 a

 x3 
 

a
4 1 2

 p  3x 3 y 3 a 3
 x  ,

1 1 1
 p  3a 3 x 3 y 3 ,

1
 p  3  axy  3

2) Find the radius of curvature of the curve y  ax  bx  c at the point x 


2 1
2a
 a2 1  b 
y  ax 2  bx  c.
Given,
Differentiate w.r.to x.
y '  2ax  b,
y ''  2a,

y  
'
x
1
2a
a 1b
2

 2a
1
2a
 
a 2  1  b  b,

y  '
x
1
a 2 1 b 
 a2  1
2a

 y " x 1  a 2 1b   2a
2a

p
1  y  '2 2
,
y"
3

 2

2

1  a2  1 
 p  ,
2a
3

 p
1  a 2
 1 2
,
2a

Department of Mathematics Page 33


Calculus and Linear Algebra 18MAT11

 p
a  2 2
,
2a
a3
 p ,
2a
a2
 p
2

a2  a  x 
3) Find the radius of curvature of the curve y2  at  a, 0  .
x
a2  a  x 
Given, y  2
.
x
a3
 y2   a2 ,
x
Differentiate w.r.to x.

a3
2yy '   ,
x2
a3
y'   '
2 yx 2
 y 
'
a ,0 
 .

dx 2 yx 2
  3 ,
dy a
 dx 
    0.
 dy  a ,0

d2x 2 dx 
2
  3  y2 x  x2  ,
dy a  dy 
 d2x 
  3 0  a2  ,
2
 2 
 dy  a ,0 a

 d2x  2
  2 
 dy  a ,0 a

Department of Mathematics Page 34


Calculus and Linear Algebra 18MAT11

3
  dx 2  2
1    
  dy  
p 
d2x
2
dy
1
 p ,
 2
 
 a
a
 p
2

4) Show that the radius of curvature at an end point of the major axis of the ellipse
2 2
x y
2
 2  1, is equal to the semi latus rectum.
a b
x2 y2
Given,   1.
a 2 b2
Differentiate w.r.to x.
2 x 2 yy '
 2  0,
a2 b

x yy '
   0,
a 2 b2
b2 x
 y  2 '
'

a y
At an end point of the major axis
 x, y    a,0.
 y '  .
dx a2 y
 2 ,
dy b x
 dx 
   0
 dy  a ,0

d2x a 2  y dx 1 
    ,
dy 2 b2  x 2 dy x 
 d2x  a2  1
 2    2 0  ,
 dy  a ,0 b  a

Department of Mathematics Page 35


Calculus and Linear Algebra 18MAT11

 d2x  a
  2  2
 dy  a ,0 b
3
  dx 2  2
1    
  dy  
p 
d2x
dy 2
1
 p ,
 a 
 2 
 b 
b2
 p  semi latus rectum
a

5) Show that the radius of curvature of the curve x  aCos 3t , y  aSin3t is 3aSint Cost.

Given, x  aCos 3t , y  aSin3t.


dx
 3aCos 2 tS int,
dt

dy
 3aSin2 tCost ,
dt
dy
dy dt 3aSin 2 tCost
   ,
dx dx 3aCos 2 tS int
dt
dy
  tant
dx
d2 y dt
 2   Sec 2 t ,
dx dx
d2 y 1
   Sec 2 t '
dx 2
3a cos 2 tS int
d2y Sec 4 t
  .
dx 2 3aS int
3

p
1  y  '2 2

y ''

Department of Mathematics Page 36


Calculus and Linear Algebra 18MAT11

 p
1  tan t  2 2

 Sec 4t 
 
 3aS int 
3
 p
3aS int
Sec 4 t
 Sec t  ,
2 2

3aS int
 p 4
Sec3t ,
Sec t
 p  3aS int Cost

6) Find the radius of curvature of the curve

x  a Cost  tS int  , y  a  S int  tCost  .


Given, x  a Cost  tS int  , y  a  S int  tCost 
dx
 a   S int  tCost  S int   atCost ,
dt
 a  Cost   tS int  Cost    atS int,
dy
dt
dy
dy dt atS int
   ,
dx dx atCost
dt
dy
  tant
dx
d2y dt
 2
 Sec 2 t. ,
dx dx
d2 y 1
 2
 Sec 2 t. ,
dx atCost
d 2 y Sec 3 t
 2  ,
dx at
3

 p
1  y ' 2  2
y"
3

 p
1  tan2t  2 ,
 Sec3t 
 
 at 

Department of Mathematics Page 37


Calculus and Linear Algebra 18MAT11

at
 p 3
Sec3t ,
Sec t
 p  at

7) Find the radius of curvature of the curve

  t 
x  a  Cost  log tan    , y  aS int .
  2 
  t 
Given, x  a  Cost  log tan    , y  aS int
  2 
dy
 aCost ,
dt
 
dx  1  t  1
 a   S int  Sec 2   . 
dt  t 2 2
 tan   
 2 
 t 
 Cos  
dx 2 1 1
  a   S int  . 
 t 2
Cos 2   
dt t
 Sin
 2 2 
 
dx  1 
  a   S int  ,
dt  t  t 
 2Sin   Cos   
 2 2
dx  1 
  a   Si nt  ,
dt  S int 
dx  1  Sin2 t 
  a ,
dt  S int 
dx  Cos 2 t 
  a ,
dt  S int 
dy
dy dt aCost
   ,
dx dx  Cos 2 t 
a 
dt  S int 
dy
  tant ,
dx

Department of Mathematics Page 38


Calculus and Linear Algebra 18MAT11

d2 y dt
 2
 Sec 2 t ,
dx dx
d2 y S int S int Sec 4t
  Sec 2
t 
dx 2 aCos 2 t ' a
3

p
1  y' 2  2 ,
y"
3
a 1  tan2t  2
 p ,
S int Sec 4t
a
 p '
S int Sect
aCost
 p ,
S int
 p  a cot t

8) Show that the radius of curvature of the curve r n  a n Cosn varies inversely as r n1

Given, r n  a n Cosn .
dr
nr n1  a n  nSinn  ,
d
dr a n Sinn
 ,
d r n 1
r
 tan  ,
 dr 
 
 d 
r
 tan 
 a Sinn 
n

 
 r n 1 
rn
 tan   ,
a n Sinn
a nCosn
 tan   ,
a n Sinn
 
 tan  Cotn  tan   n  ,
2 

  n
2
Clearly, p  rSin ,

Department of Mathematics Page 39


Calculus and Linear Algebra 18MAT11

 
 p  rSin   n  ,
2 
 p  rCosn ,
p
 Cosn  ,
r
r n  a n Cosn
But p
 r n  an . ,
r
 r n1  a n p        1
Differentiate w.r.t p.
dr
 n  1 r n  a n ,
dp

dr an
  ,
dp  n  1 r n

dr
pr ,
dp
an
 pr ,
 n  1 r n

an 1
 p ,
 n  1 r n1
1
 p
r n1

9) Show for the curve r 1  Cos   2a, p2 Varies as r 3

Given, r 1  Cos   2a.


Differentiate w.r.to ,
dr
rSin  1  Cos   0,
d
dr rSin
 ,
d 1  Cos
r
 tan  ,
 dr 
 
 d 

Department of Mathematics Page 40


Calculus and Linear Algebra 18MAT11

r
tan  '
 rSin 
 
 1  Cos

1  Cos
 tan   '
Sin

2Sin2
 tan   2 ,
 
2Cos Sin
2 2

 tan  tan ,
2

  
2

Clearly, p  rSin .
 
 p  rSin    ,
 2
 p is always positive
 p
 Sin  ,
2 r
But r 1  Cos   2a,

 r 2Sin 2  2a ,
2
p2
r  a,
r2
 p 2  ar          1
Differentiate w.r.to p.
dr
2p  a ,
dp
dr 2 p
 ,
dp a

dr 2 2 r
  ar 
dp a a
dr
pr ,
dp

Department of Mathematics Page 41


Calculus and Linear Algebra 18MAT11

2 r
 pr ,
a
4r
 p2  r 2 ,
a
4
 p2  r3 ,
a
p2  r 3

r 2  a2 a
10) Find the radius of curvature of the curve    Cos 1  
a r

r 2  a2 a
Given,    Cos 1  
a r
Differentiate w.r.t  .
 
 
1 1 dr  1  a   dr
1 . .2r   .  2  ,
a 2 r a
2 2 d  a  r
2
  d
 1    
 r 
r dr a dr
1  ,
a r 2  a 2 d r r 2  a 2 d
1 dr  r a 
1    ,
r  a d  a r 
2 2

 r 2  a2
1  dr
1   ,
r 2  a 2  ar  d
r 2  a 2 dr
1
aar d

dr ar
 
d r 2  a2

2
1 1 1  dr 
We have 2  2  4  
p r r  d 
2
1 1 1 ar 
 2  2  4 
p r r  r 2  a2 

Department of Mathematics Page 42


Calculus and Linear Algebra 18MAT11

1 1  a 2 
  1  2 
p2 r 2 
 r 2  a 
1 1  r 2  a2  a2 

 2
 2  2 
p r  
 r2  a 
1 1
 2  2
p r  a2

 p  r 2  a2

Differentiate w.r.to r
dp 1
  2r 
dr 2 r  a 2
2

dp r

dr r 2  a2
dr
pr
dp

 r 2  a2 
 p  r   p  r 2  a2
 r 
 

Exercise
1. Show that for the catenary y = c cosh (x / c) the radius of curvature is equal to y 2 / c
which is Also equal to the length of the normal intercepted between the curve and
the x – axis

Find the Radius of curvature for the Following curves


  x 
2. y  a log sec  
  a 
 3a 3a 
3. x3  y 3  3axy at  , 
 2 2 

4. x  a log sec t  tan t  and y  a sec t

5. x  at 2 and y  2at

ρ
6 .Show that for the curve r  a e cot where a and  are constants  constant
r

Department of Mathematics Page 43


Calculus and Linear Algebra 18MAT11

2
7. Show that for the curve r  a 1  cos   cons tan t
r

8. Find R.O.C for the curve r  a sin n at the pole   0

VI. Evolutes and Involutes

The locus of the center of curvature for a curve is called the evolute of the curve itself is
called the involute of its evolute.

Equation of the circle of curvature

Let (X, Y) be the center of curvature and  the radius of curvature at any point ( x, y ) of the
curve. Then the equation of the circle of curvature is

( x  X )2  ( y  Y )2   2

1. Find the center of curvature of the curve y  x 3  6 x 2  3x  1 at (1, 1)

Solution: The given curve is y  x 3  6 x 2  3x  1

Therefore,

y1  3 x 2  12 x  3

and y1 (1, 1)  6

y2  6 x  12

y 2 (1, 1)  6

y1 (1  y12 )
X  x
y2

(6) 1  (6)2 


 1  36
6

Similarly

(1  y12 )
Y  y
y2

Department of Mathematics Page 44


Calculus and Linear Algebra 18MAT11

1  (6)2  43
 1 
6 6

 43 
Hence, the centre of curvature is (X, Y )   36,  
 6 

2) Find the center of curvature of the curve parabola y 2  4ax at (x,y).Allso find the equation
of the evolute of the given parabola.

Solution: The given curve is y 2  4ax

2a 2a a
Therefore y1   
y 4ax x
3
1 
And y2   ax 2
2

a  a
1  
x  x
Therefore, X  x  3
1 
 ax 2
2

 x  2(x  a)

 3x  2a (1)

 a
1  
Y  y 
x
3
1 
 ax 2
2

2(x  a)
2 a x 1

2
ax
xa
 2 a x (1  )
a
3
x2
 2 (2)
a

Hence, the center of curvature of the given parabola is

Department of Mathematics Page 45


Calculus and Linear Algebra 18MAT11

 3

(X, Y)   3x  2a,  
2
2 x
 
 a 
 

X  2a
From (1), we have x  . Putting this value in (2), we get
3
3
2(X  2a ) 2  X 2a 
3

Y  Or aY 2  4  
a  3 

Or 27aY 2  4( X  2 a)3 .

Therefore, the locus of the center of curvature  X , Y  is

27ay 2  4(x  2a)3 ,

which is the required evolute.

3) Show that the circle of curvature at the origin of the parabola


x2
y  mx  is x2  y 2  a 1  m2   y  mx 
a

x2
The equation of the given parabola is y  mx 
a

Therefore

2 x am  2 x
y1  m  
a a

So that y1 (0, 0)  m and

2 2
y2  so that y2 (0, 0) 
a a

Let  X , Y  be the center of curvature at the origin.

Then,

X  x

y1 1  y12   0  m 1  m   am 1  m 
2 2

y2 2 2
a

Department of Mathematics Page 46


Calculus and Linear Algebra 18MAT11

Y  y
1  y   0  1  m   a 1  m 
2
1
2 2

y2 2 2
a
3 3 3

 at  0, 0  
1  y  2 2
1

1  m  2 2


a 1  m2   2

y2 2 2
a

Therefore, the equation of the circle of the curvature is

( x  X )2  ( y  Y )2   2

       
2 2
 
3
am 1  m2 a 1  m2 a 2 1  m2
x   y  
 2   2  4


Or 4 x 2  4 y 2  a 1  m2 a 1  m2     am2 1  m2   a 1  m2   4  y  mx 
2

 


4a 1  m2   y  mx 
Which yields


x 2  y 2  a 1  m2   y  mx  .

Department of Mathematics Page 47


Calculus and Linear Algebra 18MAT11

MODULE-II

Differential Calculus-2

I. Taylor‟s and Maclaurin‟s series expansions for one variable

II. Indeterminate forms-L‟Hospitals Rule

III. Partial Differentiation

IV. Total Derivatives- Differentiation of composite functions

V. Maxima and Minima for a function of two variables

VI. Method of Lagrange‟s multipliers with one subsidiary condition

VII. Jacobians

Department of Mathematics Page 48


Calculus and Linear Algebra 18MAT11

Introduction

In many applications we use functions of several independent variables, for example, the
velocity of a fluid at a point depends on its space coordinates, and the temperature in heat
furnace depends upon its position and so on. The basic ideas of calculus apply to functions of
several variables as well as to function of one variable. Of course because more variable are
involved, the notation and technical detail are more complicated but the essential idea remains
the same. In the remainder of this chapter we will explore the extension of the process and
ideas of differentiation to functions of several independent variables. The rate of change of the
function with respect to its variables can be expressed in terms of rate of change of the function
with respect to each of the independent variables separately.

Often the value of a function and the values of its derivatives are known at a particular
point and form this information it is desired to obtain values of the function around that point.
The Taylor‟s polynomials and Taylor‟s series allow engineers to make such estimates. One of
the applications is to obtain linearized model is, it is much easier to analyze than the non-linear
systems. The great advantage of a linearized model is, it is much easier to analyze than the non-
linear one. It is possible to make use of principle of super position, it allows the effects of
multiple inputs to a system to be considered separately, and the resultant output to be obtained
by summing up the individual outputs.

I. Taylor’s and Maclaurin’s series expansions for one variable

Taylor’s Theorem

If the function can be expanded as an infinite series, then

Which is the Taylor‟s theorem in the interval [ a, a+h] with 0 < <1

(i) If the function can be expanded as an infinite series, then


+ ………….
is called Taylor series about the point Taking , we get Maclaurin‟s
Series.
(ii) If the function can be expanded as an infinite series, then
+ ………….
Is called Maclaurin‟s Series

(iii)If the function can be expanded as an infinite series, then

Department of Mathematics Page 49


Calculus and Linear Algebra 18MAT11

Is called Maclaurin‟s Theorem

1) Expand as Maclaurin‟s series up to fourth degree terms

Solution: Let =

( )

= y

= y

= y

= y

= y

Therefore Maclaurin‟s series of

+ +…………

=1+ + is the required series

2) Expand as Maclaurin‟s series up to fourth degree terms.

Solution: Let, =

= y (∵ )

= =0 ( )

= y ( )

Department of Mathematics Page 50


Calculus and Linear Algebra 18MAT11

= y

= y

Therefore Maclaurin‟s series of is

+ +…………

= is the required series

3) Expand as Maclaurin‟s series up to fourth degree terms

Solution: Let =

= y

= log a y

= y

= y

= y

Therefore Maclaurin‟s series of is

+ +…………

+…………

Department of Mathematics Page 51


Calculus and Linear Algebra 18MAT11

4) Expand log as Maclaurin‟s series and hence show


that

log = + + ……………….

Solution: Let =log

= log y

= y

= y

= y

= y

Therefore Maclaurin‟s series of is

+ +…………

.................................. ……(i)

Similarly by putting , we get

+................................. ……(ii)

Consider,

= .................................. hence the proof.

Department of Mathematics Page 52


Calculus and Linear Algebra 18MAT11

5) Expand as Maclaurin‟s series up to fourth degree terms

Solution: Let =

= y

= y

= =1+

= y

= y

Therefore Maclaurin‟s series of is

+ +…………

is the required series

6) Expand tan as Maclaurin‟s series upto third degree terms

Solution: Let, tan

tan

=1+

= +

Therefore Maclaurin‟s series of is

+ +…………

tan = 1+ +…………….. is required series

Department of Mathematics Page 53


Calculus and Linear Algebra 18MAT11

7) Expand = log as Maclaurin‟s series upto fourth degree terms.

Solution: Let = log

= log

+2y’’ ,

+3y’’’ ,

Therefore Maclaurin‟s series of y is

+ +…………

log + ………… is the required series.

8) Expand

Solution: Let,

Department of Mathematics Page 54


Calculus and Linear Algebra 18MAT11

+y’

+2y’’ ,

Therefore Maclaurin‟s series of y

+ ………

log ……… is the required series.

9) Expand as Maclaurin‟s series up to third degree terms

Solution: Let,

Therefore Maclaurin‟s series of is

+ +…………

……………… is the required series

Department of Mathematics Page 55


Calculus and Linear Algebra 18MAT11

10) Expand = as Maclaurin‟s series up to third degree term

Solution : Let, =

Therefore Maclaurin‟s series of is

+ +…………

+…………is the required series

11) Expand as Maclaurin‟s series up to fourth degree terms.

Solution: Let,

∵ )

Department of Mathematics Page 56


Calculus and Linear Algebra 18MAT11

Therefore Maclaurin‟s series of is

+ +…………

+…………is the required series

12) Expand x in powers of and hence evaluate correct to 4


decimal places.

Solution : The Taylor‟s series of is

……∞…………….(1)

Let,

Substitute these values in (1)

+ …………∞

…………

Now putting

Log 1.1=0.1-0.005+0.0003-0.00002=0.0953

Department of Mathematics Page 57


Calculus and Linear Algebra 18MAT11

13) Expand = in powers of up to third degree terms

Solution : The Taylor series of is

+ ……
….(1)

Let, =

=0

=0

Substitute these values in (1), we get

+ …………

= + +………………….is the required series

14) Expand = log in powers of up to fourth degree terms

Solution: The Taylor series of = is

…………………(1)

Let = log

= log

= =

=-

Department of Mathematics Page 58


Calculus and Linear Algebra 18MAT11

=- = -1 –

= -2

= -2 -2

Substitute these values in (1), we get

+…………………

15) Expand in ascending powers of h up to term containing


Solution: We have by the Taylor‟s theorem,

= + + + +

Taking a = ,y ⇒

= + + + + …………..(1)

Substitute these values in (1) we get

= is the required series

Department of Mathematics Page 59


Calculus and Linear Algebra 18MAT11

Exercises

1. Obtain the Maclaurin‟s expansion of sin 1 x up to the term containing x 5 .

2. Show that log tan   4



 x  2x 
4x3 4x5
3

3
 ....

3. Expand tan 1 x in ascending powers of x up to first three non-zero terms and hence

show that   4(1  1 / 3  1 / 5  ...)

sin 1 x 2 3 2.4 5
4. Show that  x x  x  ...
1 x 2 3 3.5

1
5. Obtain the Maclaurin‟s expansion of e tan x
up to the term containing x 5 .

II. Indeterminate forms

We have studied certain rules to evaluate the limits. But some limits cannot be
evaluated by using these rule. These limits are known as indeterminate forms. There are seven
types of indeterminate forms given as:

(i) (ii) (iii) 0 x ∞ (iv) ∞ - ∞

(v) (vi) (vii)


These limits can be evaluated using L Hospital‟s Rule

L Hospital’s Rule

Statement: If and are two functions of x which can be expanded by Taylor‟s series
in the neighborhood of x=a and

If, = = then

Proof: Let x=a+h

Department of Mathematics Page 60


Calculus and Linear Algebra 18MAT11

= = [ by Taylor‟s theorem]

= =

= = provided ≠0

Standard Limits

Following standard limits can be used to solve the problems

Evaluation of limits of the form

Problems under this type are solved by using L. „Hospital‟s rule considering the fact that

= if = 0 and

1) Evaluate

Let,

[ Applying L‟Hospital‟s Rule]

[ Applying L‟Hospital‟s Rule]

Department of Mathematics Page 61


Calculus and Linear Algebra 18MAT11

2) Evaluate

Solution: Let,

[ Applying L‟Hospital‟s Rule]

=1 [ ]

3) Evaluate

Solution: Let,

[ Applying L‟Hospital‟s Rule]

= =

4) Evaluate

Solution: Let,

[ Applying L‟Hospital‟s Rule]

5) Evaluate

Solution: Let,

Department of Mathematics Page 62


Calculus and Linear Algebra 18MAT11

6) Evaluate

Solution: Let, = . =

7) Evaluate

Solution: Let,

Now, and

8) Evaluate

Solution:

9) Find a,b,c if =2

Solution: 2 = =

= [as ]

But limit is finite, therefore, numerator must be zero

…………………..(1)

Department of Mathematics Page 63


Calculus and Linear Algebra 18MAT11

Thus, 2 =

But limit is finite, therefore, numerator must be zero

………………(2)

2=

………………..(3)

From equations (1) and (2), we have

Solving equations (3) and (4)

10) Evaluate

Solution: As , G.E , as =e

G.E

Apply L‟Hospital‟s Rule

G.E = …(1)

Let,

Diff. w.r.t x, + log

Department of Mathematics Page 64


Calculus and Linear Algebra 18MAT11

i.e. =

Substituting in (1), we get

G.E =

= e. (Applying L.H Rule)

= e.

= e. (again , Apply L.H Rule)

Thus G.E = -e . =-e. =

11) Find „a‟ and „b‟ such that =

Solution: When given expression Apply L.H Rule

G.E =

Here, the denominator becomes 0 as while the numerator becomes

Therefore the numerator is not zero also, the limit becomes .


We must choose …………….(i)

With this choice, we again get form and can apply L. H Rule.

G.E =

Department of Mathematics Page 65


Calculus and Linear Algebra 18MAT11

= = by data

= or =2 ……………..(ii)

Adding (i) & (ii) ,

Thus ,b

Evaluation of limits of the form ,

The limits which are of the form may be evaluated by reducing them to the form
or . For this purpose, we first take the logarithms of the function whose limit is
required and use the result.

If L = , then log L =

Evaluate the following limits

12)

Solution (12) Let, L = =

Then, log L = =

= , by L Hospital‟s Rule = =0

Therefore the required limits is L = =1

Department of Mathematics Page 66


Calculus and Linear Algebra 18MAT11

Then, log L = tan x log = =

= = =

= . = 1x0 =0

Therefore the required limit is L = =1

(14) Let, L= =

Then, log L = = = =

= =

Therefore the required limit is L = =1

(15) Let, L = =

Setting

Then, log L = = =

= = = -0 =0

Therefore, the required limit is L = =1

Evaluate the following limits:

Department of Mathematics Page 67


Calculus and Linear Algebra 18MAT11

Solution : (16) Let, L= =

Then, log L = . = = -1

Therefore the required limit is L = =

(17) Let, L= =

Then, log L = =0

Therefore the required limit is L = =1

(18) Let, L= =

Then, log L = =

Therefore the required limit is L =

(19) Let, =

Then, log L = =

= =

Therefore the required limit is L =

(20) Let, =

Then, log L = = = -1

Therefore the required limit is L = =

(21) Let, =

Then, log L = log = =

Department of Mathematics Page 68


Calculus and Linear Algebra 18MAT11

= =1

Therefore the required limit is L = =

Evaluate the following limits:

Solution: (22) Let, =

Then, log =

Therefore the required limit is L = =

= =

Therefore the required limit is L =

Then, log L log

Therefore the required limit is L =

Department of Mathematics Page 69


Calculus and Linear Algebra 18MAT11

25) Evaluate

Solution: Let, L= =

Taking log on both the sides, we get

= = =

Apply LHR , = = =

Apply LHR , = = =

=L=

26) Evaluate

Solution: Let, L = =

log L = =

Apply LHR, log L = =

log L = =0 ⇒ L = =1

27) Evaluate

Solution :

Let, l=

log l = =

= == =

Department of Mathematics Page 70


Calculus and Linear Algebra 18MAT11

log l = ⇒ 1 =

Hence, log l = = e-

28) Evaluate

Solution: ( )

Let, =

log = ( )

= ( )

= = =0 log

Hence, =1

29) Evaluate

Solution: ( )

Let,

( )

( )

( )

Department of Mathematics Page 71


Calculus and Linear Algebra 18MAT11

log l = l= or =

Exercises

Evaluate the following limits:


1)

sin x sin 1 x
2) lim x0
x2
3) lim x0 log sin x sin 2 x
1 
4) lim x0   cot x 
x 
1
 ax  bx  cx  d x x
5) limx0  

 4 
6) limx0 tan x log x
7) lim
x
π tan x cos x
2

a cosh x  b cos x
8) Find the value of the constants a and b such that lim may be equal to
x 0 x2
unity.

III. Partial Differentiation


1) If z = -3 Show that +

Solution: Let, z= -3 ………..(1)

Differentiating (1) partially w.r.t

=3 -3 ………………..(2)

Once again differentiating (2) partially w.r.t. x

= 6x-0+0+ ……………………….(3).

Then differentiating (1) partially w.r.t. y

Department of Mathematics Page 72


Calculus and Linear Algebra 18MAT11

=0-6xy+0- ………………………. (4).

Once again differentiating (4) partially w.r.t. y

= -6x- ………………………. (5).

Adding (3) & (5), we get

+ = =0

2) Verify that = for the function z =

Solution: Let, z= ……………………….(1)

Differentiating (1) partially w.r.t. x

= ……………………….(2)

Differentiating (2) partially w.r.t. y

…(3)

Differentiating (1) partially w.r.t. y

= ……………..(4)

Differentiating (4) partially w.r.t. x

= …………… (5)

From (3) and (5)

Department of Mathematics Page 73


Calculus and Linear Algebra 18MAT11

3) Verify that =

Solution: Let, …………………….(1)

Differentiating (1) partially w.r.t. „x‟

……………(2)

Differentiating (2) partially w.r.t. „y‟

= ………….(3)

Differentiating (1) partially w.r.t. „y‟

……….(4)

Differentiating (4) partially w.r.t „x‟

= = = ………(5)

From (3) and (5) we get

4) Verify that

Solution: Let, ……………. (1)

Differentiating (1) partially w.r.t „x‟

= sin y + y cos x ………………. (2)

Differentiating (2) partially w.r.t. „y‟

……………….. (3)

Department of Mathematics Page 74


Calculus and Linear Algebra 18MAT11

Differentiating (1) partially w.r.t. „y‟

………………….. (4)

Differentiating (4) partially w.r.t „x‟

……………………. (5)

From (3) and (5) we get,

5) If u =

Solution: Let, u= ……………. (1)

Differentiating (1) partially w.r.t. „y‟

………(2)

Differentiating (2) partially w.r.t. x,

6) If u= ϕ =

Solution: Let, u= ϕ ……………(1)

Department of Mathematics Page 75


Calculus and Linear Algebra 18MAT11

Differentiating (1) partially w.r.t y

Once again differentiating (1) partially w.r.t y

ϕ ………(2)

Differentiating (1) partially w.r.t x

Once again differentiating (1) partially w.r.t x

(from (2))

i.e. =

7) If V +

Solution: V=

. ……………..(1)

Again differentiating w.r.t x

= ……………. (2)

……………… (3)

Consider (using (1), (2) and (3) we get)

Department of Mathematics Page 76


Calculus and Linear Algebra 18MAT11

LHS =

=0

8) If find the value of n which will make

Solution: Let, ……………(1)

Diff. w.r. t. t

………………….. (2)

Differentiating (1) partially w.r.t r

Multiplying above equation by

Now, = =

Given condition from (2) and (3)

Department of Mathematics Page 77


Calculus and Linear Algebra 18MAT11

⟹ = ⟹ =

9) If u =

Solution: Consider, u ………(1)

Differentiating (1) partially w.r.t z

………..(2)

Differentiating (2) partially w.r.t y

………………..(3)

Diff. (3) partially w.r.t x

= (1+3 )

10) If u

Solution: u ………………….. (1)

……………. (2)

Multiply both sides of (2) by b, we get

…..….. (3)

…..….. (4)

Department of Mathematics Page 78


Calculus and Linear Algebra 18MAT11

Multiply both sides of (4) by a, we get

…..….. (5)

Adding (3) and (5), we get

(Using (1))

Partial derivative of symmetric functions

Symmetric functions

A function is said to be symmetric if and is said to be symmetric if

1) If

Solution: …………………(1)

Differentiating (1) partially w.r.t. x

Then x + +z = +

x + +z =3u

2) If =2

Solution: Consider, ……………… (1)

Department of Mathematics Page 79


Calculus and Linear Algebra 18MAT11

Differentiating partially w.r.t x

LHS = (∵ sin 2A = 2sin A cosA)

=2

3) If , show that =1

Solution: Let, = ……….(1)

= = …………… (2)

= ……….. (3)

= ……….. (4)

Adding (2), (3) and (4) we get

Department of Mathematics Page 80


Calculus and Linear Algebra 18MAT11

=1

4) If show that =-u

Solution: Let, u= ……………..(1)

Similarly, ……..(2)

……..(3)

Multiplying (1), (2) and (3) by x,y,z respectively and adding we get

+z =

+z =

5) If then prove that = and hence show

that

Solution: Let, …………….. (1)

Similarly,
……………..……….. (2)

…………………….. (3)

Adding (1), (2) and (3) we get,

Department of Mathematics Page 81


Calculus and Linear Algebra 18MAT11

…………………..(4)

Consider =

= …………using (4)

= + +

6) If = , show that =4

Solution: Let,

= = ………………. (1)

Similarly, = ……………….(2)

Consider, = =

= =

…………………………………(3)

Again consider,

4 =4

Department of Mathematics Page 82


Calculus and Linear Algebra 18MAT11

4 =4

=4 ==4 =4 =4

From (3) and (4)

4 hence verified

Indirect Partial Derivative

1) If = and hence deduce that =


when

Solution: Let,

treat z as a function of x and y in order to find required partial derivatives taking log on both
sides, we get

Differentiating partially w.r.t y we get

0+ + =0

Similarly, =

Differentiating partially w.r.t x, we get

= - , (sub )

=- =

When

= = (∵ log e=1)

Department of Mathematics Page 83


Calculus and Linear Algebra 18MAT11

2) If u show that + +

Solution: Consider, . ……………….. (1)

Differentiating partially w.r.t „x‟ and „y‟

= 2x ⇒ = ………………….. (2)

= 2y ⇒ = …………………. (3)

Let, u ⇒ = = (using (2))

= = (using uvw rule)

= + (using (2))

= …………..(4)

Similarly, = …………..(5)

Adding (4) and (5), we get

+ =

+ =

+ =

3) If x = r cos and y = r sin =

Solution: We have, x = r cos y = r sin = =

Department of Mathematics Page 84


Calculus and Linear Algebra 18MAT11

Differentiating partially w.r.t respectively

………………….(1)

………………….(2)

Differentiating (1) partially w.r.t x

Similarly, (3)

LHS = = = =

RHS = = = ==

Thus LHS = RHS

IV. Total Derivatives-Differentiation of composite functions

Homogeneous function

A function is said to be homogeneous of degree n in variables x and y if it can be


expressed in the form or

An alternative test for a function to be homogeneous of degree n is that

Total Differentiationals

Let , then the total differential (or the exact differential) dz is defined as

Note :

(i) If and where then the total derivative of z is

Department of Mathematics Page 85


Calculus and Linear Algebra 18MAT11

(ii) If and then the total derivative of u is

(iii)If then the total derivative of du is

Composite function

A function of a function is called composite function

i) If and , then z is called a composite function of

= +

= +
(i) If and , then u is called a
composite function of
= + +

= + +

= + +

1) Find when z = and verify by direct substitution

Solution: = + …….(1)

= = =

= using all this in (1), we get

Department of Mathematics Page 86


Calculus and Linear Algebra 18MAT11

= +

= (4 )2 + (2a )2 (using )

By direct substitution

=10 =2 . Hence the verification.

du 3
2) If u=sin 1  x  y  , x  3t , y  4t 3 show that  and verify the result by
dt 1 t2

direct differentiation.

u  sin1  x  y  ; x  3t, y  4t 3

u   x, y   t  u  t 
du
dt
is the total derivative.

du u dx u dy
 
dt x dt y dt

=
1
3
1
12t  2

1  x  y 1  x  y 
2 2

3 1  4t 2 
=
1   3t  4t 3 
2

3 1  4t 2  3
= 
1  9t  24t  16t
2 4 6
1  9t  24t 4  16t 6
2

1  4t  2 2

Dividing 1  9t 2  24t 4 16t 6  by 1  8t 2  16t 4  we get 1  t 2  .

Department of Mathematics Page 87


Calculus and Linear Algebra 18MAT11

du 3
Thus 
dt 1 t2

Now consider u=sin 1  x  y   sin 1  3t  4t 3 

In order to differentiate u w.r.t „t‟ directly it is convenient by using the substitution t=sin  so
that we have,

u=sin 13sin   4sin    sin 1 sin3   3

d
 3  sin 1 t  
du d 3
Thus u = 3  and hence 3
dt dt dt 1 t2

du 3

Thus dt 1 t2 …..(2)

From (1) and (2) the result is verified.

x y z du du du
3) If u=f  , ,  , prove that x y z  0.
 y z x dx dy dz

Here we need to convert the given function u into a composite function.

x y z
Let u= f  p, q, r  where p= , q  , r 
y z x

Ie., u   p, q, r    x, y, z   u  x, y, z
u u p u q u r
   
x p x q x r x

u u 1 u u  z 
Ie.,     0    2 
x p y q r  x 

u x u z u
 x   ……(1)
x y p x r

Similarly by symmetry we can write,

u y u x u
y   ……(2)
y z q y p

Department of Mathematics Page 88


Calculus and Linear Algebra 18MAT11

u z u y u
z   ……(3)
z x r z q

u u u
Adding (1), (2) and (3) we get x y z 0
x y z

4) If z  f  x, y  and x  u  v, y  uv show that

z z z  z z z 
(i) u  v  u v (ii)  u  v   
x u v  y u v 

z   x, y   u, v   z  u, v
z z x z y z z x z y
   and  
u x u y u v x v y v

z z z z z
ie.,  1   v v …..(1)
u x y x y

z z z z z
   1   u    u …..(2)
z x y x y

z z
(i) Consider R.H.S = u v
u v
 z z   z z 
=u   v   v  u 
 x y   x y 
z
=  u  v   L.H.S  R.H.S=L.H.S
x
z z
(ii) Consider R.H.S= 
u v

Adding (1) and (2) we have,

z z z
   u  v   L.H .S  R.H.S = L.H.S
u v y

z z
5) If z=x 2  y 2 where x=e u cos v find and as a composite function and verify the
u v
results by direct substitution.

z   x, y   u, v   z  u, v 

Department of Mathematics Page 89


Calculus and Linear Algebra 18MAT11

z z x z y z z x z y
  ;  
u x u y u v x u y u

z
   2 x   eu sin v    2 y   eu cos v 
u

 2eu eu sin v  sin v   eu cos v  cos v 

 2eu  eu  sin 2 v  cos2 v   2e2u

z
  2 x   eu cos v    2 y   eu sin v 
u

 2eu eu sin v  cos v   eu cos v  sin v   0

Thus as a composite function we have obtained

z z
 2e2u ;  0 . . . (1)
u v

Now consider z  x 2  y 2 by direct substitution.

z   eu sin v    eu cos v   e2u  sin 2 v  cos2 v   e2u


2 2

. .t u and also wr
Differentiating partially wr . .t v we get

z z
 2e2u and 0 . . . (2)
u v

From (1) and (2) we conclude that the result is verified.

6) Find
du
dx
using the concept of composite functions given that u=tan 1 x
y 
where x 2  y 2  a 2

Here u   x, y  and y  x  u  x
du u u dy
 1  ….(1)
dx x y dx

Consider u=tan 1  x y 

Department of Mathematics Page 90


Calculus and Linear Algebra 18MAT11

u 1 1 y2 1 y
 .  .  2
x 1   x y  y x  y y x  y 2
2 2 2

u 1 x y2 x x
 .   . 2   2
y 1   x y  2
y 2
x y
2 2
y x  y2

Also consider x 2  y 2  a 2 and differentiating w.r.t x, we get

dy dy x
2x  2 y  0 or 
dx dx y

Using these results in (1) we have,

du y x  x y 2  x2 1
 2  2 
.    
dx x  y 2
x  y  y  x  y  y y
2 2 2

du 1 1
Thus  
dx y a2  x2

5. If z is a function of x,y and u,v are connected by the relations u  3x  4 y,

v  3 y  4 x show that

2 z 2 z  2 z 2 z 
  25  2 2
x 2 y 2  u v 

Exercises

1. Verify that for the function

z z
2. If z  e axby f ax  by  prove that b a  2abz
x y
3. If + sin =2

4. If u = , show that + =0
5. 4. If
6. If
 x2  y2  u u 1
7. If u  tan 1   , then show that x y  sin 2u
 x y  x y 2

Department of Mathematics Page 91


Calculus and Linear Algebra 18MAT11

x2 y2 u u
8.If sin u  show that x y  3 tan u
x y x y
9. If z = +

10. If z =

z  f  x, y 
11. If where x  eu cos v, y  eu sin v prove
2 z 2 z 2u   z 2 z 
2
that   e  2  2 
x 2 y 2  u v 
V. Maxima and Minima for function of two variables

Working procedure for finding extreme values of f  x, y 

 We have to first find the stationary points (x,y) such that f x  0 and f y  0
 We then find the second order partial derivatives A  f xx , B  f xy , C  f yy .

We evaluate these at all the Stationary points and also compute the corresponding Value of
AC  B2

(a) A stationary point  x0 , y0  is a maximum point if AC  B2 > 0 & A < 0. f  x0 , y0  is a


maximum value

(b) A stationary point  x1 , y1  is a minimum point if AC  B2 > 0 & A > 0. f  x1 , y1  is a


minimum value.

Note: We can 0verlook the cases of AC  B2 < 0, AC  B 2  0, A  0

1) Find the extreme values of the function f  x, y   x3  3xy 2  3x2  3 y 2  4

f x  3x2  3 y2  6x; f y  6xy  6 y

We shall find points (x,y) such that f x  0 and fy  0

3x 2  3 y 2  6 x  0 or x2  y 2  2x  0 . . .(1)

6 xy  6 y  0 or y  x 1  0 . . . (2)

 y  0, x  1 from (2).From (1) if y  0 then x  0, 2 and if x  1, y  1

 the stationary points are  0,0 ,  2,0 , 1,1 , 1, 1 Let A  f xx , B  f xy , C  f yy

Department of Mathematics Page 92


Calculus and Linear Algebra 18MAT11

(0, 0)  2,0 1,1 (1, 1)

A  6x  6 -6 < 0 6>0 0 0

B  6y 0 0 6 -6

c  6x  6 -6 6 0 0

AC  B2 36 > 0 36 > 0 -36 < 0 -36 < 0

Conclusion Max.pt. Min.pt. Saddle pt. Saddle pt.

Maximum value of f  x, y  is f  0,0  4 and minimum value of f  x, y  is f  2,0  0

2) Find the maximum and minimum values of the function

x3  3xy 2  15 x 2  15 y 2  72 x

Let f  x, y   x3  3xy 2 15x2 15 y 2  72x

f x  3x2  3 y 2  30x  72. f y  6xy  30 y

We shall find points  x, y  such that f x  0 and f y  0

Ie., 3x 2  3 y 2  30 x  72  0 or x 2  y 2  10 x  24  0 . . . (1)

6 xy  30 y  0 or y  x  5  0 . . . .(2)

(2) gives us y  0 and x 5

Putting y  0 in (1) we get x 2  10 x  24  0

Or  x  4 x  6  0 ie., x  4, 6

 4,0 ,  6,0 are stationary points.

Putting x  5 in (1) we get y 2  1  0 or y  1

5,1 , 5, 1 are also stationary points.

Let us examine these points for maxima and minima

Department of Mathematics Page 93


Calculus and Linear Algebra 18MAT11

Let A  f xx , B  f xy , C  f yy

 4,0  6,0 5,1 5, 1


A  6x  30 6 < 0 6>0 0 0

B  6y 0 0 6 -6

C  6x  6 6 6 0 0

AC  B2 36 > 0 36 > 0 36 < 0 36 < 0

Conclusion Max.pt. Min.pt. Saddle pt. Saddle pt.

Maximum value of f  x, y  is f  0,0  4 and minimum value of f  x, y  is f  2,0  0

3) Find the maximum and minimum values of the function

x3  3xy 2  15 x 2  15 y 2  72 x

Let f  x, y   x3  3xy 2 15x2 15 y 2  72x

f x  3x2  3 y 2  30x  72, f y  6xy  30 y

We shall find points  x, y  such that f x  0 and f y  0

Ie., 3x 2  3 y 2  30 x  72  0 or x 2  y 2  10 x  24  0 . . .(1)

6 xy  30 y  0 or y  x  5  0 . . .(2)

(2) gives us y  0 and x  5

Putting y  0 in (1) we get x 2  10 x  24  0

0r  x  4 x  6  0 ie., x  4, 6

  4,0 ,  6,0 are stationary points.


Putting x  5 in (1) we get y 2  1  0 or y  1

 5,1 , 5, 1 are also stationary points.

Let us examine these points for maxima and minima Let A  f xx , B  f xy , C  f yy

Department of Mathematics Page 94


Calculus and Linear Algebra 18MAT11

(4, 0)  6,0 5,1 5, 1


A  6x  30 6  0 6>0 0 0

B  6y 0 0 6 -6

C  6x  30 6 6 0 0

AC  B2 36  0 36  0  36  0  36  0

Conclusion Max.pt. Min.pt. Saddle pt. Saddle pt.

Maximum value of f  x, y  is f  4,0  64  240  288  112

Minimum value of f  x, y  is f  6,0  216  540  432  108

4) Find the extreme values of f  x, y   x3 y 2 1  x  y 

f  x, y   x3 y 2  x4 y 2  x3 y3

We shall find points  x, y  such that f x  0 and f y  0

f x  3x2 y 2  4x3 y 2  3x2 y3 , f y  2x3 y  2x4 y  3x3 y 2

Consider f x  0 f y  0 and f y  0

Ie., x2 y 2  3  4x  3 y   0 and x3 y  2  2x  3 y   0

 x = 0, y = 0, 4x + 3y = 3 and x = 0, y = 0, 2x + 3y = 2

Let us form the pair of equations

X=0 x=0 y=0 4x +3y=3 4x +3y=3 4x +3y=3

Y=0 2x+3y 2x+3y=2 x=0 y =0 2x +3y =2

The stationary points are (0, 0 ) (0, 2/3), (1, 0 ) (0, 1) (3/4, 0) and (1/2, 1/3)

Also A= f xx  6 xy 2  12 x 2 y 2  6 xy 3  6 xy 2 (1  2 x  y )

B= f xy  6x2 y  8x3 y  9x2 y 2  x2 y  6  8x  9 y 

C= f yy  2x3  2x4  6x3 y = 2 x3 1  x  3 y 

Department of Mathematics Page 95


Calculus and Linear Algebra 18MAT11

It is evident that either A =0 or C =0 or both A and C are zero in respect of all the stationary
points except (1/2, 1/3)

When A = 0 or C = 0, AC-B 2 < 0 and we shall examine the nature of the point (1/2, 1/3). At
this point we get

A = -1/9, B = -1/12 C = -1/8

 Ac-B 2 =1/72 -1/144= 1/144 > 0. But A = -1/9< 0

Hence (1/2, 1/3) is a maximum point.

Thus maximum value of f  x, y   f (1/ 2,1/ 3)  1/ 432

5) Show that z  x, y   x3  y3  3xy  1 is minimum at (1, 1)

z  x, y   x3  y3  3xy  1

z x  3 x 2  3 y; z y  3 y2  3x

Let A = z xx , B = Z xy , C = Z yy  A = 6x, B= -3, C = 6y

Now, at ( 1,1) Z x  0 and ZY  0

Also A = 6, B = -3, C = 6  AC -B 2  27 > 0

Now at (1, 1), Z x  0 and Z y  0. AC-B 2  0, A = 6 > 0

 Z(x, y) at (1,1) satisfy the necessary and sufficient conditions for minima. Thus z(x,y) is
minimum at ( 1,1)

6) Examine the function f  x, y   1  sin  x2  y 2  for extremum.

f  x, y   1  sin  x 2  y 2 

f x  2 x cos  x 2  y 2  , f y  2 y cos  x2  y 2 

We shall find points such that f x  0, f y  0

2xcos  x2  y 2   0 and 2 y cos  x2  y 2   0

 x = 0, y =0 and (0, 0) is the stationary point.

Department of Mathematics Page 96


Calculus and Linear Algebra 18MAT11

A = f xx  4 x2 sin  x2  y 2   2cos  x2  y 2 

B = f xy  4 xy sin  x 2  y 2 

C = f yy  4 y 2 sin  x2  y 2   2cos  x 2  y 2 

At (0,0) : A = 2, B = 0 , C = 2  AC - B 2  4  0

Since AC - B 2  0, A = 2 > 0, (0, 0) is a minimum point and the minimum value of


f  x, y   f  0,0  1

VI. Method Of Lagrange’s Multipliers With One Subsidiary Condition

Let u  f  x, y, z  . . . .(1)

Be a function of three variables x,y,z which are connected by the relation.

  x, y, z   0 . . . .(2)

For u to have stationary values, it is necessary that

u x  0, u y  0, u z  0

u u u
 dx  dy  dz  du  0 . . . . . (3)
x y z

  
Also, differentiating (2), we get dx  dy  z  d  0 . . . . . (4)
x dy z

Multiply (4) by a parameter  and add to (3) . Then

 u    u    u  
    dx      dy      dz  0
 x x   y y   z z 

u  u  u 
This equation will be satisfied if   0,    0,   0
x x y y z z

These three equations together with (2) will determine the values of x,y,z and  for which u is
static .

Department of Mathematics Page 97


Calculus and Linear Algebra 18MAT11

Working rule: 1. Write F= f  x, y, z     x, y, z 

F F F
2. Obtain the equations  0,  0,  0.
x y z

3. Solve the above equations together with   x, y, z   0

The values of x,y,z so obtained will give the stationary value of f  x, y, z  .

1) A rectangular box open at the top is to have volume of

32cubic ft. Find the dimension of the box requiring least material for its construction.

Solution. Let x,y and z ft be the edges of the box and S be its surface.

Then S=xy+2yz+2zx and xyz= 32

Eliminating z from (i) with the help of (ii) , we get

32 1 1
S  xy  2  y  x   xy  64   
xy x y

 S x  y  64 x 2  0 and S y  x  64 y 2  0

Solving these, we get x=y=4

Now r   2 S x 2  128 x3 , s   2 S xy  1, t   2 S y 2  128 y 3 .

At x=y=4, rt  S 2  2  2  1  ve and r is also ve.

Hence S is minimum for x=y=4. Then from (ii) , z=2.

Otherwise (by Lagrange‟s method):

Write F  xy  2 yz  2zx    xyz  32

F
Then  y  2 z   yz  0
x

F
 x  2 z   zx  0
y

F
 2 y  2 x   xy  0
z

Department of Mathematics Page 98


Calculus and Linear Algebra 18MAT11

Multiplying (iii) by x and (iv) by y and subtracting, we get 2zx-2zy=0

Or x=y. [The value z=0 is neglected, as it will not satisfy (ii)]

Again multiplying (iv) by y and (v) by z and subtracting, we get y=2z.

Hence the dimensions of the box are x=y=2z=4

Now let us see what happens as z increases from a small value to a large one.

When z is small, the base flat with a large base showing that S is large. As z increases , the base
of the box decreases rapidly and S descreases. After certain stage, S again starts increasing as
z increases. Thus S must be a minimum at Intermediate stage which is given by (vi). Hence S
is minimum when x=y=4ft and z=2 ft.

2) Show that the rectangular solid of maximum volume that can be inscribed in a sphere is a
cube.

Solution: Let 2x, 2y, 2z be the length, breadth and height of the rectangular solid so that its
volume

V=8xyz . . .(i)

Let R be the radius of the sphere so that x 2  y 2  z 2  R 2 . . (ii)

Then F  x, y, z   8xyz    x 2  y 2  z 2  R 2 

And F x  0, F y  0 and F Z  0 give

8 yz  2 x  0,8 zx  2 y  0,8 xy  2 z  0

Or 2 x 2   8 xyz  2 y 2   2 z 2 

Thus for a maximum volume x=y=z.

i.e., the rectangular solid is a cube.

3) A tent on a square base of side x, has its sides vertical of height y and the top is a regular
pyramid of height h. Find x and y in terms of h, if the canvas required for its construction is to
be minimum for the tent to have a given capacity.

Solution: Let V be the volume enclosed by the tent and S be its surface area .

Then V=cuboid(ABCD, A‟B‟C‟D‟)+ pyramid (K,A‟B‟C‟D‟)

Department of Mathematics Page 99


Calculus and Linear Algebra 18MAT11

1
= x 2 y  x 2 h  x 2  y  h 3
3

1
S  4  ABGF   4 ∆KGH= 4 xy  4  x.KM 
2

 4 xy  x x 2
 4h 2  [ KM   KL  LM  
2 2
[h 2   x 2 

x2
 V  2 x  y  h 3  x  x 2  y   h  0
3

For minimum S, we have

x  4h 2   x. 
1 2
x  4h 2  .2 x] x
1 2
 S  [4 y  2


1 2
x  4h 2  .8h h  0
1 2
4 x y  x.
2

By Lagrange‟s method,

x  4h 2   x 2  x 2  4h 2 
1 2
[4 y  2
]  .2 x  y  h 3  0 . . .(i)

4 x  .x 2  0 . . . .(ii)

4hx  x 2  4h2 
1 2
 . x 2 3  0 . . . .. (iii)

(ii)gives    4 x. Then (iii)becomes

4hx  x 2  4h 2 
1 2
 4 x 3  0 or x  5h

Now putting x  5h,    4 x in (i), we get

5 4 14 8h
4 y  3h  h  .2 x  y  h 3  0 or 4 y  h  8 y   0 , i.e., y  h 2.
3 x 3 3

Department of Mathematics Page 100


Calculus and Linear Algebra 18MAT11

4) Find the volume of the greatest rectangular parallelepiped that can be inscribed in the
ellipsoid

x2 y 2 z 2
  1
a 2 b2 c 2

Solution: Let the edges of the parallelepiped be 2x, 2y and 2z which are parallel to the axes.
Then its volume V  8xyz.

Now we have to find the maximum value of V subject to the condition that

x2 y 2 z 2
  1  0 ... . ..(i)
a 2 b2 c 2

 x2 y 2 z 2 
Write F  8 xyz    2  2  2  1
a b c 

F  2x 
Then  8 yz    2   0 .. .. .(ii)
x a 

F  2y 
 8 zx    2   0
y b 

F  2z 
 8 xy    2   0 …. (iv)
z c 

Equating the values of  from (ii) and (iii) , we get x 2 a 2  y 2 / b 2

Similarly from (iii) and (iv) , wse obtain

y 2 b2  z 2 / c2  x2 a 2  y 2 b2  z 2 / c2

1 x2 y 2 z 2 1
Substituting these in (i) , we get x 2 a 2  i.e., 2  2  2 
3 a b c 3

These give x  a 3, y  b 3,z  c 3 . . . .(v)

When x=0, the parallelepiped is just a rectangular sheet and as such its volume

Department of Mathematics Page 101


Calculus and Linear Algebra 18MAT11

V=0. As x increases, V also increases continuously. Thus V must be greatest at the stage given
by (v).

8abc
Hence the greatest volume = .
3 3

VII. Jacobians

This topic is basically involved with the evaluation of determinants of second order and third
order whose elements are represented by first order partial derivatives of two or three given
functions/transformations.

The application of Jacobians is significant in the evaluation of double integrals of the form
 f  x, y dxdy and triple integrals of the form  f  x, y, z dx dy dz by

Transformation from one system (set) of coordinates (variables) to the other. The principle of
evaluation is analogous with the evaluation of  f  x dx by taking a suitable substitution.
Definition

Let u and v be functions of two independent variables x and y. The JACOBIAN (J) of u and v
w.r.t x and y is symbolically represented and defined as follows.

u u
 u, v    u, v  x y
J  or 
 x, y    x , y  v v
x y

Similarly if u,v,w are functions of three independent variales x,y,z then

u u u
x y z
 u , v, w    u , v, w  v v v
J  or 
 x, y , z    x, y , z  x y z
w w w
x y z

1) Find the Jacobians of u,v, w w.r.t x,y,z given that u=x+y+z, v=y+z, w=z

Department of Mathematics Page 102


Calculus and Linear Algebra 18MAT11

u u u
x y z
  u , v, w  v v v
We have to find J= 
  x, y , z  x y z
w w w
x y z

But u=x+y+z, v=y+z, w=z

1 1 1
 J= 0 1 1 On expanding by the last row we get J= 1(1-0)=1 Thus J=1
0 0 1

  u , v, w 
2) Find where u  x 2  y 2  z 2 , v  xy  yzzx, w  x  y  z
  x, y.z 

  u , v, w 
The definition of J= is same as in the previous problem.
  x, y , z 

But u  x 2  y 2  z 2 , v  xy  yz  zx, w  x  y  z

Substituting for the partial derivatives we get

2x 2y 2z
J y z x z yz
1 1 1

Expanding by the first row,

J  2x  x  z    y  x   2 y  y  z    y  x   2z  y  z    x  z 

 2 x  z  y   2 y  z  x   2z  y  x 

 2  xz  xy  yz  xy  yz  xz   0 Thus J=0

 u , v, w 
3) If u  x1 x2 , v  x2 x3 , w  x3 x1 find J 
 x1 , x2 , x3 

Department of Mathematics Page 103


Calculus and Linear Algebra 18MAT11

u u u
x1 x2 x3
 u , v, w  v v v
J 
 x1 , x2 , x3  x1 x2 x3
w w w
x1 x2 x3

Where u  x1 x2 v  x2 x3 , w  x3 x1

x x1
2 x1 x2 2 x1 x2 0
x3 x2
 0 2 x2 x3 2 x2 x3
x3 x1
2 x3 x1 0 2 x3 x1

x2  x1 x3  x1  x2 x3 
   0   0  
2 x1 x2  4 x3 x1 x2  2 x1 x2  4 x3 x1 x2 

x1 x2 x3 xx x 1 1 1 1
  1 2 3    Thus J=
8 x1 x2 x3 8 x1 x2 x3 8 8 4 4

4) If x  p cos  , y  p sin  , z  z evaluate the Jacobean of x,y,z w.r.t

p,  , z x  ρ cos  , y  ρ sin  , z  z

x x x
p  z cos   p sin  0
  x, y , z  y y y 
J  sin  p cos  0
  p,  , z  p  z
0 0 1
z z z
p  z

Department of Mathematics Page 104


Calculus and Linear Algebra 18MAT11

On expanding by the las row, J=1  p cos2   p sin 2    p

5. If x  r sin  cos  , y  r sin  sin  , z  r cos  show that


  x, y, z 
 r 2 sin 
  r , ,  
x x x
r  
  x, y , z  y y y

  r , ,   r  
z z z
r  
x  r sin  cos  , y  r sin  sin  z  r cos 

sin  cos r cos sin r sin sin


J= sin  sin r cos sin r sin cos
cos r sin 0

On expanding by the last row we get,

cos r 2 sin  cos  cos2   r 2 sin  cos  sin 2 

r sin  r sin 2  cos2   r sin 2  sin 2 

 r 2 sin  cos2   cos2   sin 2    r 2 sin3   cos2   sin 2  

 r 2 sin  cos2  .1  r 2 sin3  .1  r 2 sin   cos2   sin 2  

  x, y, z 
Thus  r 2 sin 
  r , ,  

Department of Mathematics Page 105


Calculus and Linear Algebra 18MAT11

yz zx xy   u , v, w 
6) If u  , v  , w  , show that 4
x y z   x, y , z 
yz zx xy
By data u  ,v  , w  ,
x y z

u u u  yz z y
x y z x2 x x
  u , v, w  v v v z  zx x
 
  x, y , z  x y z y y2 y
w w w y x  xy
x y z z z z2
 yz   zx    xy   x   x  
 2  2   2       
x  y   z   z   y  
z  z   xy  y x  y  z x y   zx  
   2  .   .   
x y  z  z y x y z z  y 2 
 yz  x 2 x 2  z   x x  y  x x
         
x 2  yz yz  x  z z  x  y y
 0 1111  4
  u , v, w 
Thus 4
  x, y , z 

Exercises

 u , v 
1. u  e x cos y v  e x sin y find
  x, y 
  x, y , z 
2. x yz u yz v z  uvw find
 u , v, w

  x, y , z 
3. x  r sin  cos y  r sin  sin  z  r cos find
 r ,  ,  

Department of Mathematics Page 106


Calculus and Linear Algebra 18MAT11

MODULE-III

INTEGRAL CALCULUS

I. Introduction

II. Evaluation of Double Integrals

III. Evaluation of Triple Integrals

IV. Evaluation of Double Integrals by Changing the Order of Integrations

V. Change of variables in Double and Triple Integrals

VI. Application to area and volume

VII. Beta And Gamma Functions

VIII. Properties of Beta Functions

IX. Relation between Beta and Gamma Functions

Department of Mathematics Page 107


Calculus and Linear Algebra 18MAT11

I. Introduction

Integral calculus is an important part of calculus, as important as differential calculus. In differential


calculus we study the relationship between two quantities, let's say between distance and time. For this
relationship we usually use the rate of change between two variables.

In integral calculus, however, we take the inverse process of the relationship between two quantities.
This is known as integration, anti-differentiation or anti-derivative. The most important application of
integral calculus is to compute the area or volume of a shape. In ancient times, the informal concepts
were developed by the Greek mathematicians Archimedes (287 BC – 212 BC) and Eudoxus (410 BC –
347 BC). They developed the approximate area of different geometric shapes, and these basic methods
were also developed by Chinese mathematician Liu Hui around the 3rd century to find the area of a
circle. In the 17th Century John Kepler further developed some important concepts regarding
astronomical investigations to find the area of a sector and the area of an ellipse. The concept of integral
calculus was formally developed further by Isaac Newton and Gottfried Leibniz; they developed basic
concepts to find area and volume

We are familiar with the integration in case of function of single variable, i.e., in case of
function of single variable, integration is taken only once.

Suppose if there are two or three independent variables, then we have to take integration two times or
three times. Such integrals are called respectively double and triple integrals.

In this chapter, we will learn about the multiple integrals which are the natural extension of definite
integrals to the integrals of function of two variables (double integrals) and Integrals of function of three
variables (triple integral). Here, we will also see how double and triple integrals are useful in evaluating
area of a curved surface, volume of solids, mass, center of gravity, center of pressure, moment of inertia
and product of inertia.

x2 ys
In general double integrals are represented as   f  x, y dxdy
x1 y1

x2 y2 z2
Triple integrals as    f  x, y, z dxdydz
x1 y1 z1

Department of Mathematics Page 108


Calculus and Linear Algebra 18MAT11

II. Evaluation of Double Integrals


x2 y
Consider the double integral in the form   f  x, y dxdy
x  x1 y  y

Case I: When y1 , y2 are functions of x and x1 , x2 are constants, then first integrate

f  x, y  wr
. .ty assuming x as constant between the limits y1, y2 and then integrate the resulting expression
w.r.t x between the limits x1 , x2. .

x2 y2

i.e., I1    f  x, y  dy dx
x1 y1

Where the integration is carried out from the inner to the outer rectangle.

Case II: When x1 , x2 are functions of y and y1 , y2 are constants, then integrate f  x, y  w.r.t y
between the limits y1 ' and ' y2 .

y2 x2

i.e., I 2    f  x, y  dx dy
y1 x1

Where integration is carried out from the inner rectangle to the outer rectangle.

Case III: Suppose all the four limits x1 , x2 , y1 , y2 are constants. i.e., if x1  a, x2  b, y1  c, y2

b d d b

  f  x, y dxdy  a c f  x, y  dy dy   f  x, y  dx dy
x2 y2
Then 
x1 y1 c a

Thus for constant limits, the order of integration is immaterial.

III. Evaluation of Triple Integrals


x2 y2 z2
Consider the triple integration of the form    f  x, y, z dxdy
x1 y1 z1

Here the method is to evaluate is similar to that of double integrals.

If x1 , x2 are constants y1 , y2 are either constants or functions of x and z1 , z2 are either constants or
functions of x and y then the integral is evaluated as follows:

Department of Mathematics Page 109


Calculus and Linear Algebra 18MAT11

First integrate f  x, y, z  w.r.tz between the limits z1 and z2 assuming x and y as constants.Then

integrate the resulting expression w.r.t y between the limits y1 and y2 , assuming x as constant. Then

fin ally integrate the obtained result w.r.t.x between the limits x1 and x2 .

x2 y2  x  z2  x , y 

i.e., I     f  x, y, z  dz dy dx
x1 y1  x  z1  x , y 

The integration is carried out from the innermost rectangle to the outermost rectangle. The order of
integration may be different form different types of limits.
1 6
1. Evaluate   xydxdy
0 0
Solution:

6
1 6
 y2  1
 62 1

Let I    xydxdy   x   dx   x   0 dx


x 0 
Y 0 Y 0
2 0 x 0 
2 
1
 x2  1
= 18     18     9
 2 0 2
3 2
2. Evaluate   xy 1  x  y dxdy
0 1
Solution:
3 2 3 2
Let I    xy 1  x  y dxdy     xy  x
2
y  xy 2 dxdy
0 1 x  0 y 1

2
3
 xy 2 x 2 y 2 xy 3 
=    
3 1
dx
x 0 
2 z

3
   22  2  22   23     12  2  12   13   
=    x    x    x      x    x    x    dx
x 0   2   2   3    2   2   3   

3
3
3 3 2 7   3 x 2 3 x3 7 x 2 
=   x  x  x dx =  .  .  . 
x 0 
2 2 3   2 2 2 3 3 2 0

Department of Mathematics Page 110


Calculus and Linear Algebra 18MAT11

3 32 3 33 7 32 315
= .  .  . 
2 2 2 3 3 2 12
1 x
3. Evaluate   xydydx
0 x
x
1 X 1 x
 y2  1
Solution: Let I  
0 X
xy dy dx    xy dy dx   x   dx
x 0 y  x x 0 
2  yx

 
1 1
 x 2  dx    x 2  x3 dx
x 2 1
= x0 2  x
 2 x 0

1
1  x 3 x 4  1  1 1   1
=         0  
2  3 4  0 2  3 4   24

1 x

  x  y 2 dx dy
2
4. Evaluate
0 x
Solution:

1 X
Let I    x  y 2 dydx
2

0 Y X

x
 2 1
y3 
= x y   dx
0 x
3

 5 x 2 4 3
3
1 2 2 1 3
= x 2
 x  dx =   =
0 3 3  7 15 3 35
 

5. Evaluate   xy
A
dx dy where A is the domain bounded by x-axis, ordinate

x  2a and the curve - x  4ay.


2

Solution:

The point A  2a, a  is the point of intersection of the line x  2a with the parabola x 2  4ay.

x2
x2 4a
2a 4a
y 
2a 2
   xy dx dy    xy dx dy   x   dx
x 0 
A x 0 y 0
2 0

Department of Mathematics Page 111


Calculus and Linear Algebra 18MAT11

2a
3
2a 3  x6  a4
= 5 
 x dx = 32a 2  6  0 3
32a 2 x 0

6. Evaluate  xy dx dy over the region in the positive quadrant for which x  y  1.


R
Solution:

Required region lines in the first quadrant for which x  y  1, which is shown in the figure

1
 y2   1 x 
1 1 2
1 1 x

 xy dx dy    xy dx dy =  x   dx   x   dx
x 0 
R x 0 y 0 x 0 
2  y 0 2 

1
=
1
2 x0
 x  2x2  x3 dx
1
1  x 2 2 x3 x 4  1  1 2 1  1
       
2  2
=
3 4  0 2  2 3 4  24

7. Evaluate  xy  x  y  dx dy taken over the region enclosed by the curve yx and y  x 2 .
Solution:

Solving the equations y  x ' and ' y  x 2 . we get x2  x  x  x 1  0  x  0 or 1 which gives

y  0, y  1. The points intersection are  0,0 and 1,1 .


1 x
  xy  x  y     x y  xy 2  dx
2
dy
x 0 y  x2

x
 2 y 2 xy 3 
1

=  x 
x 0 
2 3  y  x2

1
 x4 x4 x6 x7 
=   2  3  2  3 dx
x 0 

1
 x 5 x 5 x 7 x8  3
=     
 2 3.5 2.7 3.8  0 56

Department of Mathematics Page 112


Calculus and Linear Algebra 18MAT11

IV. Evaluation of Double Integrals by Changing the Order of Integrations


b f2  x 

The integral   f  x, y dx dy


a f1  x 
is the integral over the region bounded by the curves

y  f1  x  ' and ' y  f2  x  for the values of x between a and b. For changing its order one should
sketch the region of integration. From the sketch, the new limits x and y should be determined as usual.

In a double integral with the variable limits, the change of order of integration changes the limits of
integration. While doing so sometimes it is required to split up the region of integration and the given
integral is expressed as the sum of a number of double integrals with the changed limits. Note that the
change of order of integration does not in any way affect the value of the area. Sometime it is possible
to evaluate the integrals very easily by changing the order of integration. This method is explained in
the following example very clearly.

x
a a

  x  y 2  dy dx
2
1. Change the order of integration in the integral
x 0 x
a
and hence evaluate it.

x
a a
Solution: Let I    x  y 2 dy dx
2

x 0 y  x
a

x x
The point of intersection of y  with y 
a a

i..e, with the parabola y 2 


x
is  a,1
a

2
For the change of order of integration, y varies from 0 to 1 and x varies from ay to ay.
1 ay ay
 x3 
   
1
I x  y dy dx =    y 2 x 
2 2
dy
y 0   x ay 2
X  0 y  ay 2 3

1
  a3 y 3 3 a y
3 6
4 
 a3 a
=    ay     ay  dy  
y 0  3   3   28 20

4 a 2 ax
2. Change the order of integration and hence evaluate   xydy dx
0 x2
4a

Department of Mathematics Page 113


Calculus and Linear Algebra 18MAT11

4 A 2 aax
Solution: Let I  
X 0 y  x2
xydy dx
4a

x2
Given limits are x=0 and y  to 2 ax or x  0' to ' x  4a and x  4ay to y  4ax. Now we
2 2

4a
can draw the required region.

The two parabolas y 2  4ax and x 2  4ay intersect at  0,0 and  4a,4a  .

y2
For the change of order of integration the new limits are y  0 to 4a and x  to x  2 ay
4a
2 ay
4 aa 2 ay
 x2  4a
I   
y 0
xy dx dy   y  
y 0  2  y 2
dy
x y
2
x
4a 4a

1
4a
 y4  1 
4a
y3 
2 y0  2 y0 
=   2

16a 2  16a 2 
y 4 ay dy 4 ay dy

1   4a   4a  
4a 6
1  y3 y6 
3

=  4a    4a    
2 3 6 16a 2  0 2   3  96a 2 

1 x
3. Evaluate  
x 0 y  x
xy dy dx

For the required region x varies from 0 to 1

y  x to y  x i.e., y  x is a parabola.
2
and y varies from

The point of intersection of y=x with the parabola y2  x is 1,1 .

For the change of order of integration y varies from y=0 to 1

and x varies from x  y2 to x  y.


y
 x2 
1 y 1
 I    xy dy dx   y   dy
y 0 
y 0 x  y 2
2  y2

1 1
y  y 2  y 4  dy    y 3  y 5 dy
1 1
=
20 20

Department of Mathematics Page 114


Calculus and Linear Algebra 18MAT11

1
1  y4 y6  1  1 1  1
=       
2  4 6  0 2  4 6  24

1 z x z
4. Evaluate     x  y  z dz dy dx
1 0 x  z

Solution:
1 z x z
Let I      x  y  z  dz
Z 1 x  0 y  x  z
dy dx

Integrate first w.r.t y keeping x and z constants

x z
1
 y2 
z
 I     xy   zy  dz dx
z 1 x  0   x x z
2

 
1 z
1
=    x  z  2z  2 4xy  dz
z 1 x  0
dx

Now integrate w.r.t x keeping z as constant

z
 x2 z 2 x2 z 
1 1
 z3 3 z3  1
I  2  z x  dx  2    z  dz  2  z 3dz  0
1 
2 2 0 1 
2 2 1

1 1 x 2 1 x 2  y 2

5. Evaluate  
0 0

0
xyz dx dy dz

Solution:

1 1 x2 1 x2  y 2

Let I  
x 0 y 0

z 0
xyz dx dy dz

1 1 x 2
 z2 
1 x 2  y 2 1 1 x 2  1  x 2  y 2  
=    xy       xy    dx dy
x 0 y 0  2  z 0 0 0  2 

Department of Mathematics Page 115


Calculus and Linear Algebra 18MAT11

1 1 x 2
1
=    xy  x3 y  xy 3  dx dy
20 0

1 x 2
1  xy 2 x3 y 2 xy 4 
1
=   
4  0
dx
20 2 2

1
1  x2 2 x4 x6 
1
1 1
=   x  2 x  x       
3 5

80 8 2 4 6  0 48

a x x y

  e
x y z
6. Evaluate dz dy dx
0 0 0

Solution:

a x x y
Let I    e x  y  z dz dy dx
0 0 0

a x x y a x x y
=   
x 0 y 0 z 0
e x  y  z dz dy dx =   
x 0 y 0 z 0
e x . e y . e z . dz dy dx

a x a x
x y
  e . e . e z  . dx dy   e .e . e  e0  dx dy
x y
 x y
 x y
0
x 0 y 0 x 0 y 0

x
a x
 2 x e2 y x y  a
   e .e  e .e  dx dy   e .
2x 2y x y
 e .e  dx
x 0   y 0
x 0 y 0
2

a
 e 4 x 2 x   e 2 x x  
    e     e  dx
x  0    2 
2

a
 e4 x e2 x e2 x x  e4 a e2 a e2 a a   1 1 1 
   e      e       1
 4 2 2 2 2 0  8 2 4  8 2 4 

e4 a 3e2 a a 3
  e 
8 2 8

Department of Mathematics Page 116


Calculus and Linear Algebra 18MAT11

c b a

   x  y 2  z 2  dz dy dx
2
7. Evaluate
 c b  a
Solution:
c b a
I    x  y 2  z 2  dz dy dx
2
Let
 c b  a

a
c b
 z3 
    x 2 z  y 2 z   dy dx
x  c y  b 
3 a

c b
 2  a3 a3  
    x  a  a   y 2
 a  a       dy dx
x  c y  b   3 3 
b

c
y 3 2a 3 
   2ax 2 y  2a  y  dx
c   b
3 3

 2
c
 b 3 b 3  2a 3 
   2ax  b  b   2a      b  b dx
c  3 3 3 
c
 x3 4ab3 4a 3b 
  4ab   .x 
3   c
.x
 3 3

 c3 c3  4ab3 4a3b
  4ab      c  c  c  c 
 3 3 3 3

8abc3 8ab3c 8a 3bc 8abc 2 2 2



3

3

3

3
a  b  c 

V. Change of variables in Double and Triple Integrals

Sometimes the evaluations of a given double integral become more convenient by changing the
variables. By changing the variables, a given integral can be transformed into a simpler form involving
new variable.

Let the variables x,y be changed to new variables u,v by using the transformations

x  f u, v  , y  y u, v  .

Department of Mathematics Page 117


Calculus and Linear Algebra 18MAT11

Then,   f  x, y  dx
R xy
dy    f  u, v  u, v   |J| du
Rxy
dv

  x, y 
Where J =  0 is the Jacobian of x and y w.r.t u and v.
  u, v 

In case if mit is required to change Cartesian to polar co-ordinates by putting


x  r cos  , y  r sin  , then

x x
  x, y  r  cos  r sin 
J    r
  r ,  y y r sin  r cos 
r 

   f  x, y  dx
Rxy
dy    f  r ,  |J| drd    f  r ,  r
Rro Rr 0
dr d

Similarly for triple integrals change Cartesian to spherical co-ordinates by putting

x  r sin  cos 
y  r sin  sin 
z  r cos 

Then  f  x, y, z  dx
v
dy dz   f  r , ,   |J| dr
v
d d

  f  r , ,  r 2 sin  drd d


v

VI. Application to area and volume

Area, volume and volume of revolution given by the formulae are:

1.  dx dy 
R
Area of the region R in the Cartesian form.

2.  rdrd  Area of the region R in the polar form.


R

3.  dx dy dz  Volume of a solid.


v

4. V   r sin  dr d d is spherical form


v

Department of Mathematics Page 118


Calculus and Linear Algebra 18MAT11

5. Volume of a solid (in polars) obtained by the revolution of a curve enclosing an area A

about the initial line is given by

V   2 2 sin  dr d
A

x2 y2
1. Find the area of the ellipse 2  2  1 by double integration
a b
Area A   dxdy
A

b 2 2
  a x
a a
dydx  4   y 0 a 
a a2  x2

 
b
A  4 A1  dx
x 0 y 0 x 0

x 2 a2 x
 a  x dx 
2 2
a  x  sin 1  
2

2 2 a
a
4b  x a 2  x 2 a 2 1 x 
a
a 2 2
4 a  x dx    sin 
x 0
b a  2 2 a 
0

4b  a2  4b a 2 

a 
0   sin 1
 sin 1
0    a . 2 . 2   absq.units.
2 

Note: The area of the circle x2  y 2  a2 by double integration is  a 2 , this is a particular case of the
example when b  a.

2. Find by double integration the area enclosed by the curve r  a 1  cos  between   0 and
  .
Solution:

Area A =  r dr d
write r varies from 0 to a 1  cos   and  from 0 to  .

 a 1 cos 
i.e., A
 0
  r 0
r dr d

 a 1 cos  
 r2  1
=    d   a 2 1  cos   d
2

 0  2  0 2  0

Department of Mathematics Page 119


Calculus and Linear Algebra 18MAT11

 2 
a2      
=  
2  0 
2 cos 2    d  2a 2  cos 4   d
 2  0 2

 
Put   and  varies from 0 to
2 2
 
2 2
 A  2a  cos   4a  cos .d
2 4 2 4

0 0

(using reduction formula )

3 1  3 a 2
 4a 2 . . .  sq.units
4 2 2 4

Show that the area between the parabolas y  4ax and x  4ay is  a3 .
2 4
3.
3
Solution:

Solving the equations y 2  4ax and x 2  4ay, it is seen that the parabolas intersect at

O(0,0) and A(4a,4a). As such for the shaded area between these parabolas x varies from 0 to 4a and y
x2
varies from P to Q i.e y  to y2  ax . Hence the
4

4a 2  ax  4a
 x2 
Required Area =   dydx    2  ax    dx
0 
0 x2
4a 
4a

4a
2 3 1 x3 32 2 16 2 16 2
= 2 a. x 2  .  a  a  a
3 4a 3 0
3 3 3

4. Calculate by double integration, the volume generated by the revolution of the


Cardioid r  a 1  cos  about its axis.
Solution:

Since the cardioid is symmetrical about the x-axis, volumes generated by the upper and the lower halves
overlap. As such we consider the revolution of the upper half only for which r varies from 0 to a(1-
cos  ) and  varies from 0 to  .

 a 1 cos 
Required volume of revolution  2  
0 0
r 2 sin  dr d

Department of Mathematics Page 120


Calculus and Linear Algebra 18MAT11

 a 1 cos  
 r3  2
 2  sin    d   sin  .a 3 1  cos   d
3

o  3 0 3 o

2 a3
 1  cos  sin  d
3
=
3 0


2 a3  1  cos  4   a3   a3 4 8 3
        .2   a
4
 1 cos
3  4  0 6  6 3

5. Find by triple integration, the volume of the sphere x 2  y 2  z 2  a 2


Solution:

Changing to polar spherical coordinates by putting

x  r sin  cos  , y  r sin  sin  , z  r cos 

We have dx dy dz  r 2 sin  dr d d

Also the volume of the sphere is 8 times the volume of its portion in the positive octant for which r
 
varies from 0 to a,  varies from 0 to and  varies from 0 to . z
2 2
 
a 2 2
 Volume of the sphere  8  r
2
sin  dr d d
0 0 0

 
a 2 2
  r dr.  sin  d .  d
2

0 0 0

a
r3  /2  a3 4
 8.  cos  0
.  4 . .  0  1   a 3
3 0
2 3 3

6. Evaluate   x  y  z dx dy dz where R is the region (tetrahedron)


R
bonded by the

planes x  0, y  0, z  0 and x  y  z  1.
Solution:

Let, x  y  z  1 z  1  x  y

 For the given region z varies from z  0 to 1  x  y 

At z  0, x  y  z  1  x  y  1  y  1 x

Department of Mathematics Page 121


Calculus and Linear Algebra 18MAT11

 y var ies from y  0 to1  x

For y  0, z  0, x  y  z  1  x  1

 x varies from x  0 to 1

1 1 x 1 x  y

  x  y  z  dx dy dz =     x  y  z  dz dy dx
R x 0 y 0 z 0

1 1 x 1 x  y
 z2 
    x  y  z   dx dy
x 0 y 0  2  z 0

1 x
1 1 x
1   x  y 
1 3
1
   1   x  y  dx dy    y 
 
2
 dx
2 x 0 y 0   2 x 0  3 
y 0

1
1 3x 2 x 4 
1
1 1
 x0 6 
       
3
2 3 x x  dx 
6
2 x
2 
4 0 8

Exercise

a x x y

 e
x y  z
1. Evaluate dzdydx
0 0 0
a b c

x
2
2. Evaluate y 2 z 2 dxdydz
0 0 0
1 Z xZ
3. Evaluate    ( x  y  z)dydxdz
1 0 X  Z
a b c

   (x  y 2  z 2 )dydxdz
2
4. Evaluate
 a b  c
a 2 2
b y
b b
5. Change the order of integration and evaluate  
0 0
xy dxdy

1 x
6. Evaluate   xy dxdy
0 x
by changing the order of integration
a
1 x 2
1 b
7. Change the order of integration and evaluate  
0 0
y 2 dxdy

Department of Mathematics Page 122


Calculus and Linear Algebra 18MAT11

x
a a

  (x  y 2 )dxdy
2
8. Change the order of integration and evaluate
0 x
a
1 2 x

9.
Evaluate 
0 x2
xy dxdy by changing the order of integration



e
 ( x2  y 2 )
10. Change into polar coordinates and evaluate dxdy
0 0

16 2
11. Show that the area between the parabolas y 2  4ax and x 2  4ay is a
3

VII. BETA AND GAMMA FUNCTIONS

The common method for determining the value of n! is naturally recursive, found by
multiplying 1 ∗ 2 ∗ 3 ∗ ... ∗ (n − 2) ∗ (n − 1) ∗ n, though this is terribly inefficient for large n. So, in the
early 18th century, the question was posed: As the definition for the nth triangle number can be
explicitly found, is there an explicit way to determine the value of n! Which uses elementary algebraic
operations? In 1729, Euler proved no such way exists, though he posited an integral formula for n!
Later, Legendre would change the notation of Euler‟s original formula into that of the gamma function
that we use today. While the gamma function‟s original intent was to model and interpolate the factorial
function, mathematicians and geometers have discovered and developed many other interesting
applications. In this paper, I plan to examine two of those applications. The first involves a formula for
the n-dimensional ball with radius r. A consequence of this formula is that it drastically simplifies the
discussion of which fits better: the n-ball in the n-cube or the n-cube in the n-ball. The second
application is creating the psi and poly functions, which will be described in more depth later, and allow
for an alternate method of computing infinite sums of rational functions.

Here in this topic, we study two special functions namely Beta function and Gamma function

which help us to evaluate certain definite integral which are either difficult or impossible to evaluate by
various known methods.

Department of Mathematics Page 123


Calculus and Linear Algebra 18MAT11

Definitions and Notation


1
(i) The integral x m1 1  x 
n 1

0
dx for m >0, n>0 is known as Beta function and isdenoted by

1
  m, n  ie.,   m, n    x m1 1  x 
n 1
dx.
0

e
x
(ii) The integral x n 1dx for n >0 is known as Gamma function and is denoted by
0


n. i.e., n  e x x n 1dx 0

 1  1.
1. Prove that
Solution:


Let   n    e x x n 1dx
0


 1   e x x0 dx   e0  0  1  1

2. Prove that   n  1  nn, n  0 .

Solution:


Let   n    e x x x 1dx
0


  n  1   e x x n dx (Integrating by parts)
0


 e x     e x n  x n1  dx
x n 

0
0

As the first term vanishes on the R.H.S for the limits 0 to , we get,

Department of Mathematics Page 124


Calculus and Linear Algebra 18MAT11


  n  1  n  e x x x1dx  n  n  if n  0
0

Corollary:   n  1  n!, n is a positive integer

By the recurrence formula, we have,

  n  1  n  n   n  n 1   n 1

 n  n 1 n  2 .......... 1  n  n 1 ..........1  n!

 1  1

VIII. Properties of Beta Functions

3. Prove that   m, n     n, m
1
Solution: Let   m, n   x m1 1  x 
n 1
 0
x

 a a

 
 f  x  dx   f  a  x  dx 
 0 0 
1
n 1
  1  x  1  1  x  
m 1
dx
0

1
n 1
  1  x  1  1  x  
m 1
dx
0

1
  1  x  x n 1dx    n, m 
m 1


y m1
4. Prove that   m, n    1  y  m n
dy =
0

1
Solution: Let   m, n   x m 1 1  x 
n 1
 0
dx

Department of Mathematics Page 125


Calculus and Linear Algebra 18MAT11

y dy
Putting x  when x  0, y  0; x  1, y   and dx 
1 y 1  y 
2


y m1 1 dy
   m, n    . .
1  y  1  y  1  y 
m 1 n 1 2
0


y m1

1  y 
m n
0 dy


Prove that   n   2 et t 2 n 1dt 
2
5.
0

Solution: We have   n   e x x n 1dx 
0

Put x  t 2  dx  2tdt

  n   2 et t 2 n2tdt
2

IX. Relation between Beta and Gamma Functions

  mn 
To prove that   m, n  
  m  n


We have   n   et t 2 n1.dt
2


  m   2 e x x 2 m1dx
2
…….(1)
0


Similarly   n  = 2 e y y 2 m1dy

2
……(2)
0

 
   m    n   4 e  x dx   e y y 2 n1dy
2 m1 2

0 0

Department of Mathematics Page 126


Calculus and Linear Algebra 18MAT11



 x2  y 2  x 2 m1 y 2 n1dx dy
=4 e
0 0

In the above integral x and y vary from 0 to  , x, y may be taken as the co-ordinates of any point in the
first quadrant. By putting x=r cos  , y  r sin  we are transforming Cartesian into polar and in that
case, dxdy  r dr d and the first quadrant can be covered by taking limits from 0 to  for r and 0 to

for  .
2

2
  m   n  4   e  r  r cos    r sin  
2 m 1 2 n 1
r dr d
2

0 0


2
2 m  n  1
 4   e r r cos 2 m 1  sin 2 n 1  dr d
2

0 0


   r 2 2 m n 1   2 
  2 e r dr   2  cos 2 m1  sin 2 n 1  d 
 0   0 

We get,   m   n     m  n    m, n 

  m   n
  m, n  
  m  n

1. Show that  1  2  
Solution:

  m   n
We have ,    m, n 
  m  n


2
  m   n
2   sin    cos  
2 m 1 2 n 1

0
 m  n

Put m  n  1
2

Department of Mathematics Page 127


Calculus and Linear Algebra 18MAT11


2
2   sin  
1
2 1
d 
 2    12 
 1
 1  1 
2

0
2 2

  2 
2 2
2 
0
d   1

  2 
2
 1 

 2 
 1 

2. Prove that

 i    m,
1
2
2 m 1
  m, m  ,
 2


 ii    m    m  1 2   2 1
  2m  (Duplication formula)
2

Solution:


2
We have   m, n   2   sin    cos  
2 m 1 2 n 1
(i) d . . . .(1)
0

 1
2
1
Put n     m,   2   sin  
2 m 1
d ... (2)
2  2 0

2
Put n  m    m, m   2   sin    cos  
2 m 1 2 m 1
d
0

2
22 m 1
  m, m   2  2 m 1 
sin  cos  
2 m 1
d
0
2

2
2
  sin 2 
2 n 1
 2 m 1
d
2 0


Let 2   and limits   0    0 and   
2
 d
  d 
2 2

Department of Mathematics Page 128


Calculus and Linear Algebra 18MAT11


2 d
  sin  
2 m 1
 m 1
2 0
2


2
1
.2   sin  
2 m 1
 2 m 1
.d (Using(2))
2 0

1  1
  m, m   2 m 1
.  m, 
2  2

 ii 
  m   m 1   m  2
 2 m1 .
1  
  m  m 2  m 1
2  
 1
  m  m  1  2 m1    2m 
2 2 

2

 sin
p
3. Evaluation of x cos9 x dx
0
Solution: We know that the beta function can be expressed as

2
  m, n   2  sin 2 m 1 cos 2 n 1  d
0

Using this relation, we can write (by putting 2m  1  p and 2m  1  q) and


2
1  p 1 q 1 
 sin x cos q xdx  
p
, 
0
2  2 2 
 p 1  q 1
  
=
 2   2 
 pq2
2  
 2 
(Using relation between beta and gamma functions)


2
4. Evaluate 
0
cot  .d in terms of Gamma functions.

 p 1   q 1 
  
2
1 1  2   2 
1

 cot  .d   sin  cos  .d 


1
2 2
Solution:
2  pq2
0 0  
 2 

Department of Mathematics Page 129


Calculus and Linear Algebra 18MAT11

1 1
Put p   and q
2 2
 1 1   1 1
 2  2 
1  2   2  1  1   3 
    
2 1  1  2 4 4
 2 2  1
 2 
 

1 1  1
     1  
2 4  4

1   
  n   1  n  
 4 
= =
2 sin 1 sin  / 4  2

1 dx
5. Express 
0
1  x4
in terms of gamma functions.

Solution: Putting x2  sin   x  sin1 2 

1 1
dx  sin 2  cos  d
2

When x  0,  0, x  1 
2
1 1
1

2 sin 2  cos  .d
dx
   2
0 1  x4 0 1  sin 2 


2
1 1
=
2  sin
0
 cos 0  d
2
(m= 1
2,
n  0)

  1 1  0 1
 2  
1 1 1
      
1 1  22   2   4
 .  
1 2 2

2 2   1 0  4 3 4 3
   
 2  1  4 4
 2 
 
 
2 2
1
6. Show that  0
sin  d  
0 sin 
d  

Department of Mathematics Page 130


Calculus and Linear Algebra 18MAT11

 
2 2
1
 sin d   sin
1
Solution: Let LHS  2 2
d
0 0


1  2 12  2 
1
 2  sin  cos  d   2  sin 2 cos 0  d 
0

4 0  0 
  

1
Put p  , q  0 and using formulae
2

  3   1    1   1 
   
1   4   4     4   2  
 
4 5 

3 
  4 4 
  n  1  n  n  ,  1  1  1  1
4
   
  4     4  

  1   1   1 
  
1   2   4   2  
   
4 1 1 
 
 4 4 

Department of Mathematics Page 131


Calculus and Linear Algebra 18MAT11

Exercise

 /2
1. Express  0
cot d in terms of Beta function and hence evaluate


dx
2. Express  1 x
0
4
in terms of Beta function and hence evaluate

3. Show that
 

 xe dx   x e dx 
x 3
2 x 4

0 0 16 2

 /2  /2
d
4. Using Beta and Gamma functions evaluate 
0
sin  d  
0 sin 

1
x2
1
1 

0 1  x4
dx  
0 1  x4
dx 
4 2
5. Show that

1 1
dx dx
6. Express the integral  in terms of gamma function. Hence evaluate 
0 1  xn 0 1  x2 3
1
dx
7. Evaluate using gamma and beta functions  0 1  x4

Department of Mathematics Page 132


Calculus and Linear Algebra 18MAT11

MODULE –IV

Ordinary Differential Equations of First Order

I. Introduction

II. Exact and Reducible To Exact Differential Equations

III. Bernoulli‟s Equation

IV. Orthogonal Trajectories

V. Newton‟s Law of Cooling and LR- Circuits

VI. Nonlinear Differential Equations

VII. Clairaut‟s Equation

Department of Mathematics Page 133


Calculus and Linear Algebra 18MAT11

I. Introduction

A differential equation is a mathematical equation for an unknown function of one or


several variables that relates the values of the function itself and of its derivatives of various
orders. Differential equations play a prominent role in engineering, physics, economics and
other disciplines.

A simplified real world example of a differential equation is modeling the acceleration of a ball
falling through the air (considering only gravity and air resistance). The ball's acceleration
towards the ground is the acceleration due to gravity minus the acceleration due to air
resistance. Gravity is a constant but air resistance is proportional to the ball's velocity. This
means the ball's acceleration is dependent on its velocity. Because acceleration is the derivative
of velocity, solving this problem requires a differential equation.
Differential equations arise in many areas of science and technology; whenever
a deterministic relationship involving some continuously changing quantities (modeled by
functions) and their rates of change (expressed as derivatives) is known or postulated. This is
well illustrated by classical mechanics, where the motion of a body is described by its position
and velocity as the time varies. Newton's Laws allow one to relate the position, velocity,
acceleration and various forces acting on the body and state this relation as a differential
equation for the unknown position of the body as a function of time. In many cases, this
differential equation may be solved explicitly, yielding the law of motion.

Differential equations are mathematically studied from several different perspectives, mostly
concerned with their solutions, functions that make the equation hold true. Only the simplest
differential equations admit solutions given by explicit formulas. Many properties of solutions
of a given differential equation may be determined without finding their exact form. If a self-
contained formula for the solution is not available, the solution may be numerically
approximated using computers. The theory of dynamical systems puts emphasis on qualitative
analysis of systems described by differential equations, while many numerical methods have
been developed to determine solutions with a given degree of accuracy.

Separation of variables is a technique commonly used to solve first order ordinary differential
equations. It is so-called because we rearrange the equation to be solved such that all terms
involving the dependent variable appear on one side of the equation, and all terms involving the
independent variable appear on the other. Integration completes the solution. Not all first order
equations can be rearranged in this way so this technique is not always appropriate. Further, it
is not always possible to perform the integration even if the variables are separable. In this
Section you will learn how to decide whether the method is appropriate, and how to apply it in
such cases. An exact first order differential equation is one which can be solved by simply
integrating both sides. Only very few first order differential equations are exact. You will learn
how to recognize these and solve them. Some others may be converted simply to exact

Department of Mathematics Page 134


Calculus and Linear Algebra 18MAT11

equations and that is also considered Whilst exact differential equations are few and far
between an important class of differential equations can be converted into exact equations by
multiplying through by a function known as the integrating factor for the equation. In the last
part of this Section you will learn how to decide whether an equation is capable of being
transformed into an exact equation, how to determine the integrating factor, and how to obtain
the solution of the original equation.

II. Exact Differential Equations

Consider a function f  x, y   x2  xy  y 2  x  y  c where c is an arbitrary constant.

Let us take the differential of this function.

df  2 x dx  x dy  y dx  2 y dy  dx  dy  0

i.e.,  2x  y  1 dx   x  2 y  1 dy  0

Obviously we can say that the solution of this differential equation is

f  x, y   c Where f  x, y  is the function we started with

M ( x, y )  2 x  y  1 and N  x, y   x  2 y  1 then
In other words, if
d  f  x, y    M  x, y  dx  N  x, y  dy

 The solution of the equation M  x, y  dx  N  x, y  dy  0 is equivalent to the solution of the


equation

d  f  x, y   0 Which is f  x, y   c, on integration.

In other words, if we are able to identify that in a given differential equation:

M  x, y  dx  N  x, y  dy  0

M  x, y  dx  N  x, y  dy  d  f  x, y 

Then we can simply conclude that the solution of the equation is f  x, y   c. Thus we say that
M  x, y  dx  N  x, y  dy  0 is an exact differential equation if there exists a function f  x, y 
such that df  M  x, y  dx  N  x, y  dy.

Department of Mathematics Page 135


Calculus and Linear Algebra 18MAT11

How to identify that the given equation is exact? If so what is its solution? These two
questions are answered in the following theorem.

Statement: The necessary and the sufficient condition for the differential equation
M N
M  x, y  dx  N  x, y  dy  0 to be an exact equation is 
y x

Further the solution of the exact equation is given by

 Mdx   N  y dy  c
Where, in the first term we integrate M  x,y  w.r.t 'x'keeping y fixed and N  y  indicate the
terms in N without x (not containing x).

Worked Examples

1. Solve:  2x  y 1 dx   x  2 y 1 dy  0


Let M  2 x  y  1' and ' N  x  2 y  1

M N
  1' and ' 1
y x

M N
Since  , the given equation is exact.
y x

The solution is  M dx   N  y  dy  c where N(y) denotes terms in N not containing x.

   2 x  y  1dx    2 y  1dy  c

 x 2  xy  x  y 2  y  c

Thus x 2  xy  y 2  x  y  c, is the required solution.

2. Solve:
y 3
 3x2 y  dx   x3  3xy 2  dy  0

Let M  y 3  3x 2 y ' and ' N   x 3  3xy 2

M N
  3 y 2  3 x 2 ' and '  3 x 2  3 y 2
y x

Department of Mathematics Page 136


Calculus and Linear Algebra 18MAT11

M N
Since  , the given equation is exact.
y x

The solution is  M dx   N  y  dy  c
I.e. f  y3  3x 2 y  dx   0 dy  c

(Observe that there is no term in N which do not contain x)

 y 3 .x  x 3 y  c

Thus xy 3  x 3 y  c, is the required solution.

Remark: This is also a homogeneous form of DE.

3.Solve :  5x4  3x2 y 2  2 xy3  dx   2 x3 y  3x 2 y 2  5 y 4  dy  0

Let ' M  5 x 4  3x 2 y 2  2 xy 3 and N  2 x3 y  3 x 2 y 2  5 y 4

M N
 6 x 2 y  6 xy 2 and  6 x 2 y  6 xy 2
y x

M N
Since  , the given equation is exact.
y x

thesolution is  M dx   N  y  dy  c    5x4  3x2 y 2  2 xy3 dx   5 y 4 dy  c

Thus x5  x3 y 2  x 2 y 3  y 5  c, is the required solution.

Remark: This is also a homogeneous form of DE.

dy x  3 y  4
4. Solve:  0
dx 3x  9 y  2

The given equation is equivalent to the form

 x  3y  4 dx  3x  9 y  2 dy  0
Let M  x  3 y  4 and N  3x  9 y  2

M N
  3 and 3
y x

Department of Mathematics Page 137


Calculus and Linear Algebra 18MAT11

M N
Since  , the given equation is exact.
y x

The solution is  Mdx   N  y dy  c

I.e.   x  3 y  4dx    9 y  2 dy  c

x2 9 y2
I.e.  3xy  4 x   2y  c
2 2

Thus x 2  6 xy  8 x  9 y 2  4 y  2c, is the required solution.

Remark: The problem can also be solved by putting t  x  3 y

5.Solve :  y 1  1 x   cos y  dx   x  log x  x sin y  dy  0

Let M  y 1  1 x   cos y and N  x  log x  x sin y

M N
  1  1 x  sin y And  1  1 x  sin y
y x

M N
Since  , the given equation is exact.
y x

The solution is
 Mdx   N  y dy  c
ie.,   y 1  1 x   cos y dx   0 dy  c

Thus y  x  log x   x cos y  c, is the required solution.

6.Solve : cos x tan y  cos  x  y   dx  sin x sec 2 y  cos  x  y   dy  0


letM  cos x tan y  cos  x  y  ; N  sin x sec 2 y  cos  x  y 
M N
  ,
y x

The given equation is exact.

The solution is
 M dx   N  y  dy  c
ie.,   y cos x  sin y  y dx   0dy  c

Department of Mathematics Page 138


Calculus and Linear Algebra 18MAT11

Thus sin x tan y  sin  x  y   c is the required solution.

dy y cos x  sin y  y
7. Solve:  0
dx sin x  x cos y  x

The given equation is put in the form,

 y cos x  sin y  y  dx  sin x  x cos y  x  dy  0.


Let M  y cos x  sin y  y and N  sin x  x cos y  x

M N
  cos x  cos y  1 and  cos x  cos y  1
Y x

M N
Since  , the given equation is exact.
y x

The solution is  M dx   N  y dy  c

   y cos x  sin y  y dx   0 dy  c

Thus y sin x  x sin y  xy  c, is the required solution.

8.Solve :  2 xy  y  tan y  dx   x 2  x tan 2 y  sec 2 y  dy  0


Let M  2 xy  y  tan y ' and ' N  x 2  x tan 2 y  sec 2 y
M N
  2 x  1  sec 2 y ' and '  2 x  tan 2 y
y x
M
But  2 x  1  1  tan 2 y   2 x  tan 2 y
y

M N
Since  , the equation is exact.
y x

The solution is  Mdx   N  y dy  c

ie.  2 xy  y  tan y dx   sec 2 ydy  c  x 2 y  xy  xtany  tan y  c,


Is the required solution.

Department of Mathematics Page 139


Calculus and Linear Algebra 18MAT11

9.Solve : ye xy dx   xe xy  2 y  dy  0
 Let M  ye xy , N  xe xy  2 y
M N
 ye xy x  e xy ;  xe xy y  e xy
y x
M N
Since 
y x

The equation is exact.

Solution is given by

 Mdx   N  y dy  c
e xy
ie.,  ye dx   2 ydy  c ie., y
xy
 y2  c
y
Thus e xy  y 2  c, Is the required solution.

  
10.Solve y 2 e xy  4 x3 dx  2 xye xy  3 y 2 dy  0
2 2


Let M  y 2e xy  4 x3
2

and N  2 xye xy  3 y 2
2

M N
  y 2 .e xy .2 xy  e xy .2 y,  2 xye xy . y 2  e xy .2 y
2 2 2 2

y x

M
  2 xy 3e xy  2 ye xy
2 2
N
 2 xy 3e xy  2 ye xy
2 2
y And
x

M N
Since  , the given equation is exact.
y x

The solution is  Mdx   N  y dy  c

 
ie.,  y 2e xy  4 x3 dx   3 y 2 dy  c
2

2
e xy
ie., y 2 . 2
 x4  y3  c
y
Thus e xy  x 4  y 3  c Is the required solution
2

Department of Mathematics Page 140


Calculus and Linear Algebra 18MAT11

.Exercise

1. Solve

2. Solve

3. Solve

4. Solve

5. Solve

6. Solve

7. Solve

Department of Mathematics Page 141


Calculus and Linear Algebra 18MAT11

III. Bernoulli’s Differential Equation


We first briefly discuss linear differential equation which is an essential prerequisite for the
study of Bernoulli‟s differential equation.

A differential equation is said to be linear if the dependent variable and its derivative occurs in
the first degree only and they are not multiplied together.

Standard form of a linear equation and its solution

A differential equation of the form

dy
 Py  Q . . . . .(1)
dx

Where P and Q are functions of x only is called a linear equation „y‟ and we shall solve the
same.

Multiplying (1) by e
Pdx
we have,

d   Pdx 
e  P ye   Qe   Pdx
Pdx dy Pdx Pdx
or e y  Qe
dx dx  

Integrating w .r. t. ‟x‟ on both sides we have,

e y   Qe 
Pdx Pdx
dx  c

Thus ye    Qe 
Pdx Pdx
dx  c

Is the solution of linear equation (1).

dx
An equation of the form;  Px  Q Where P and Q are functions of „y‟only is called a linear
dy
equation
in x.
The solution can simply be written by interchanging the role of x and

y in the solution obtained already for the linear equation in y.

 xe    Qe 
Pdy Pdy
dy  c

This is the solution for the linear equation in x.

Department of Mathematics Page 142


Calculus and Linear Algebra 18MAT11

Note:

1. When the DE is in the linear form, we assume the solution in the appropriate form and
present the
solution of the given equation.
The terms e e ' and ' e
Pdx Pdy
2. are called Integrating Factors.

Worked examples

dy
 y cot x  cos x
11. Solve: dx

dy
 y cot x  cos x
dx

dy
Is of the form  Py  Q, where P  cot x and Q  cos x
dx
 IF  e   e  e    sin x
Pdx cot xdx log sin x

The solution is y (IF)=  Q (IF) dx+ C

ie., y sin x   cos x sin x dx  c


sin 2 x
ie., y sin x   dx  c
2
 cos 2 x
 y sin x   c, Is the required solution.
4

.We now proceed to discuss Bernoulli‟s differential equation, whose solution is obtained by
reducing it to the linear form of DE.

dy
The DE of the form:  Py  Qy ", where P and Q are functions of x is called as Bernoulli‟s
dx
differential equation in y.

We first divide the equation throughout by y” to obtain

1 dy
n
 Py1 n  Q
y dx

Department of Mathematics Page 143


Calculus and Linear Algebra 18MAT11

put y1 n  t
dy dt
 1  n  y  n 
dx dx

1 dy 1 dt
Or 
y dx 1  n  dx
n

1 dt
Hence (1) becomes,
1  n  dx
1 dt
Hence (1) becomes,  Pt  Q
1  n  dx
dt
Or  1  n  P.t  1  n  Q, Which is a linear equation in‟t‟.
dx

dx
Similarly  Px  Qx n , where P and Q are function of y is called Bernoulli‟s equation in x.
dy
We first divide by x n and later put x1n  t

Differentiate w.r.t. y to obtain a linear equation in t=t(y).

WORKED PROBLEMS-Set 7

dy y
12. Solve:   y2 x
dx x

This is Bernoulli‟s equation. Dividing the given equation by y 2 we have

1 dy 1 1 1 dy dt
2
  x Put  i  
y dx yx y y 2 dx dx

dt t dt t
Hence (1) becomes   x or   x
dx x dx x

dt 1
This equation is a linear equation of the form  Pt  Q Where P 
dx x

Department of Mathematics Page 144


Calculus and Linear Algebra 18MAT11

and Q   x
1
 IF  e   e  x  c  log x 
Pdx  dx 1
x

Thesolution is t (IF)= Q(IF) dx+C

1 1
ie., t.    x. dx  c
x x

1
Thus   x  c, is the required solution.
xy

dy 1  1  3 y3
13. Solve:  1   y  0
dx 2  x  x

Dividing the given equation by y 3 we have,

1 dy 1  1  1 3
 1    . . . . .(1)
y 3 dx 2  x  y 2 x

1 2 dy dt 1 dy 1 dt
Put 2
 t 3  or 3 
y y dx dx y dx 2 dx

1 dt t  1  3
Hence 1 becomes  1   
2 dx 2  x  x
dt  1  6
 1   t 
dx  x  x

dt 1 6
This equation is of he form  Pt  Q W where P  1  And Q=
dx x x

 1
 1 dx
 IF  e  e  x   e x .x
Pdx

Department of Mathematics Page 145


Calculus and Linear Algebra 18MAT11

Thesolution is t (IF)=  Q (IF) dx+ C


6
ie., t.e x x   e x xdx  c
x

xe x
Thus  6e x  c , is the required solution.
y2

dy sin x cos 2 x
14. Solve:  y tan x 
dx y2

Multiplying the given equation by y 2 we have

dy
y2  y 3 tan x  sin x cos 2 x . . . .(1)
dx

dy dt dy 1 dt
Put y 3  t  3 y 2  or y 2 
dx dx dx 3 dx

1 dt
Hence (1) becomes  t tan x  sin x cos 2 x
3 dx

dt
Or  3tan x.t  3sin x cos 2 x
dx

This equation is of the form

dt
 Pt  Q, Where P=-3tanx and Q  3sin x cos 2 x
dx

 IF  e   e
 '3tan xdx
 e3logsec x    sec x   cos3 x
Pdx 3

Thesolution is y(IF)=  Q(IF) dx+C

Ie., t cos3 x   3sin x cos2 x.cos3 xdx  c

Ie., t cos3 x  3 sin x cos5 xdx  c

put cos x  u  p sin xdx  du

t cos3 x  3 u 3du  c

Department of Mathematics Page 146


Calculus and Linear Algebra 18MAT11

u6
 t cos3 x   c
2

 cos 6 x
 y cos x    c is the required solution.
3
Thus
2

dr
15. Solve: r sin   cos   r2
d

dr
We have cos   r sin   r 2
d

cos  dr 1
Dividing by r 2 we get,  sin   1 -------------- (1)
r 2 d r

1
Put  y and differentiate w.r.t „  ‟
r

1 dr dy
  And hence (1) becomes
r 2 d d

dy dy
 cos   y sin   1 or   tan   y  sec 
d d

dy
This is of the form,  Py  Q where
d

P  P    tan  and Q  Q    Sec

Here I .F  e  e
Pd tan  d
 elogsec   sec

Solution is given by r ( I .F )   Q( I .F ) d c

 r sec   sec2 d  c

 r sec  tan   c

Thus r sec  tan   c , is the required solution.

Department of Mathematics Page 147


Calculus and Linear Algebra 18MAT11

Exercise

1. Solve

2. Solve

3. Solve

4. Solve

5. Solve

Applications of Differential Equations

Orthogonal Trajectories

Basically we know that, two curves intersect each other orthogonally if the tangents at the point
of intersection are at right angles. Further we know from differential calculus that, for a
dy
Cartesian curve y  f  x  , m  represents the slope of the tangent.
dx

dy
In order to show that two curves intersect orthogonally we simply obtain for the two curves
dx
say m1 , m2 and establish m1m2  1 being the condition for two lines to be perpendicular. In
fact the Orthogonality of two polar curves also has been discussed in Module-1

With the knowledge of different equations, given a family of curves it is possible to determine
another family of curves which intersects each member of the given family orthogonally and
we discuss this concept in detail.

Orthogonal trajectories of Cartesian and polar family of curves

An equation of the form f  x, y, c   0 where c is a fixed constant represents a curve.

For example x 2  y 2  4 is a circle, y 2  4 x is a parabola etc.

On the other hand if c is an arbitrary constant (c is a parameter) the equation f  x, y   c


represents a one parameter family of curves. For each value of c we get different curves of the
same family.

Department of Mathematics Page 148


Calculus and Linear Algebra 18MAT11

For example x 2  y 2  r 2 (r is arbitrary) represents a family of concentric circles.

Definition: If two families of curves are such that every member of one family intersects every
member of the other family at right angles then they are said to be orthogonal trajectories of
each other.

Method of finding the orthogonal trajectories

Case (i) Cartesian family f  x, y, c   0

We differentiate w.r.t x and eliminate the parameter c. The equation so obtained is called as the
differential equation of the given family.

dy
We know that if tan  is the slope of a given line then the slope of the line perpendicular
dx
1 dy
to it is  . Accordingly in the differential equation of the given family we shall
tan dx
dy dx
replace by  to arrive at a new differential equation. Solving this new differential
dx dy
equation we get the orthogonal trajectories of the given family of curves.

Self-orthogonal family: If the differential equation of the given family remains unaltered after
dy dx
replacing by  then the given family of curves is said to be self-orthogonal.
dx dy

Case-(ii): Polar family f  r, , c   0

d
We know that tan   r for a polar curve where  is the angle between the radius vector
dr
and the tangent. 2  1  900 is the condition for two polar curves to be orthogonal.

2  900  1  tan 2  tan  900  1 

Department of Mathematics Page 149


Calculus and Linear Algebra 18MAT11

1
I.e. tan 2   cot 1 or tan 2 
tan 1

d d
But tan 1  r for the given curve and tan 2  r for the orthogonal curve at the same
dr dr
point.

d 1
r for the curve to be replaced by
dr d
r
dr

d dr
 r 2 to be replaced by or vice-versa.
dr d

In other words, we have to differentiate f  r, , c   0 w.r.t  and eliminate c to obtain the D.E
dr d
of the given family. We have to replace by  r 2 to obtain the new DE and solve the
d dr
same to obtain the required orthogonal trajectories.

Worked Problems

16. Find the orthogonal trajectories of the family of parabolas y 2  4ax

y2
Consider  4a ----------------- (1)
x
(if the parameter is on one side of the equation exclusively, then the same gets eliminated once
we differentiate)
Now differentiating (1) w.r.t x we have
dy
x.2 y  y 2 .1
dx dy
2
 0 Or 2 xy  y 2  0
x dx

dy
I.e. 2 x  y  0, is the DE of the given family.
dx

dy dx  dx 
Replacing by  we have, 2 x     y  0 or 2 xdx  ydy  0
dx dy  dy 

Department of Mathematics Page 150


Calculus and Linear Algebra 18MAT11

y2
  2xdx   ydy  c ie. x 2   c or 2x2  y 2  2c  k  say 
2

Thus 2 x 2  y 2  k , is the required orthogonal trajectory.

17. Find the orthogonal trajectories of the family of asteroids

x2 3  y 2 3  a2 3 Consider x 2 3  y 2 3  a 2 3

Differentiating w.r.t.x, we have

2 1 3 2 1 3 dy
.x  . y 0
3 3 dx

dy
Ie., x 1 3  y 1/3  0 , is the D.E of the given family.
dx

dy dx
Replacing by  we have,
dx dy

 dx 
x 1 3  y 1 3     0 i.e. x 1 3 dy  y 1 3 dx
 dy 

I.e. y1 3dy  x1 3dx by separating the variables.

  y1 3dy   x1 3dx  c

y4 3 x4 3 4c
I.e.   c or x 4 3  y 4 3    k  say 
 4 3  4 3 3

Thus x 4 3  y 4 3  k is the required orthogonal trajectory.

Department of Mathematics Page 151


Calculus and Linear Algebra 18MAT11

18. Find the orthogonal trajectories of the family y 2  cx3

y2
We have 3  c
x

Differentiating w.r.t x we have,

dy
x3 .2 y  y 2 .3x 2
dx dy
6
 0 Or 2 x3 y  3x 2 y 2
x dx

dy dy dx
I.e. 2 x  3 y Replacing by  we have,
dx dx dy

 dx 
2x     3 y
 dy 

I.e. 2 xdx  3 ydy  0

  2 xdx   3 ydy  c

3y2
I.e. x 2   c or 2x2  3 y2  2c  k  say 
2

Thus 2 x 2  3 y 2  k , is the required orthogonal trajectory.

19. Show that the family of curves x 3  3 xy 2  c1 and y 3  3 x 2 y  c2 are orthogonal trajectories
of each other.

Let us consider x 3  3 xy 2  c1 and differentiate w.r.t x.

 dy 
 3 x 2  3  x.2 y  y 2   0
 dx 

Department of Mathematics Page 152


Calculus and Linear Algebra 18MAT11

dy
I.e. x 2  y 2  2 xy , is the D.E of the given family.
dx

dy dx  dx 
Replacing by  we have, x 2  y 2  2 xy   
dx dy  dy 

Or 2 xydx   x2  y 2  dy  0

(This is a homogenous equation. But it is also exact)

M N
Let M  2 xy and N  x 2  y 2   2 x and  2 x.
y x

Hence the equation is exact.

The solution is given by  Mdx   N  y dy  c

y3
Ie.,  2xydx    y dy  c  x y 
2 2
 c or 3x 2 y  y 3  3c
3

 y3  3x2 y  c2  say  Is the required O.T where C2  3c

Thus x 3  3 xy 2  c1 and y 3  3 x 2 y  c2 are orthogonal trajectories of each other.

20. Show that the orthogonal trajectories of a family of circles passing through the origin
having their centers on the x-axis is a family of circles passing through the origin having their
centre‟s on the y-axis.

We shall draw a circle of the given family so as to write its equation.


.
Centre=(a,0) and radius is „a‟ since the circle passes through the origin.

 Equation of the given family of circles is

Department of Mathematics Page 153


Calculus and Linear Algebra 18MAT11

 x  a    y  0  a2
2 2

ie., x 2  2ax  y 2  0            (1)

Differentiating w. r. t x we have,

dy
2 x  2aa  2 yy1  0 Where y1 
dx

 a  x  yy1 And substituting this value of „a‟ in (1) we get ,

x2  2  x  yy1  x  y 2  0

I.e. y 2  x 2  2 xyy1 is the D.E. of the given family.

dy dx  dx 
Now let us replace y1  by   y 2  x2  2 xy   
dx dy  dy 

dx x 2  y 2
Or  which is a homogeneous equation.
dy 2 xy

2 
dx dv dv v 2  1
Put x  vy  v y Hence v  y  y
dy dy dy 2vy 2

I.e. y
dv v 2  1
 v or y
dv

1  v 2

dy 2v dy 2v

2vdv dy
  by separating the variables.
1 v 2
y

 c i.e., log 1  v 2   log y  c


2v dy
 dv  
1 v 2
y

Department of Mathematics Page 154


Calculus and Linear Algebra 18MAT11

x  x2 
I.e., log 1  v 2  y   log k  say  where v   1  2  y  k
y  y 

Or x 2  y 2  ky i.e. x 2  y 2  ky  0 and k=2b for convenience.

 x2  y 2  2by  0 or x2   y  b   b2  0
2

I.e. x2   y  b   b2 This is the equation of the family of circles passing through the origin
2

having their centre on the y-axis.

Thus we have proved the required result.

21. Find the orthogonal trajectories of the family of curves

x2 y2
  1, Where  is the parameter
a 2 b2  

x2 y2
We have 2  1 ------------ (1)
a b2  

Differentiating w .r .t „x‟ we have,

2 x 2 yy1 dy
 2  0, Where y1 
a 2
b  dx

x  yy
I.e.  2 1 ------------(2)
a 2
b 

x2  y2 x2  a2  y2
Also from (1) 2  1  2 0r  2 ------------(3)
a b  a2 b 

Department of Mathematics Page 155


Calculus and Linear Algebra 18MAT11

Now, dividing (2) by (3) we get,

x yy x y
 21 or 2  1
x a
2 2
y x a 2
y

dy dx
Now let us replace y1  by 
dx dy

x 1  dx 
   
x a
2 2
y  dy 

Or y1 dy  
x 2

dx
 a2 
x
by separating the variables .
dx
  ydy    xdx  a 2   c
x

y 2  x2
   a 2 log x  c
2 2

Thus x 2  y 2  2a 2 log x  b  0 where b=2c, is the required orthogonal trajectory.

22. Find the orthogonal trajectories of the family of coaxial circles x 2  y 2  2 x  c  0, 


being the parameter.

OR

Show that the orthogonal trajectories of a family of coaxial circles

x 2  y 2  2 x  c  0,  Being the parameter is also a system of coaxial circles.

Consider x 2  y 2  2 x  c  0 . . . (1)

Differentiating w .r .t „x‟ we have,

    x  yy1  and substituting this value in (1) we get,

x 2
 y 2   2  x  y y1  x  c  0

Department of Mathematics Page 156


Calculus and Linear Algebra 18MAT11

Ie., y 2  x 2  2 xyy1  c  0 or 2xyy1  x 2  y 2  c

x2  y2  c
 y1 
2 xy

dy dx
Now replacing y1  .by.  we have
dx dy

dx x 2  y 2  c dx
 or 
x

y2  c  
dy 2 xy dy 2 y 2 xy

dx x 2  c
 2x    y   ……(2)
dy y  y

dx dt
Now put x 2  t 2 x 
dy dy

dt t  c
Hence (2) becomes    y  
dy y  y

This is a linear equation of the form

dt 1  c
 Pt  Q, where P  and Q    y  
dy y  y

1
 y dy
Hence IF  e
Pdy 1
e  e log y 
y

The solution is t ( IF )   Q( IF ) dy  c1

t  c1
Ie.,     y   dy  c1
y  y y

t 1
ie.,    dy  c  2 dy  c1
y y

x2 c
ie.,   y   c1 or x 2   y 2  c  c1 y
y y

 x 2  y 2  c1 y  c  0 ; Let c1  2  and c  c2

Department of Mathematics Page 157


Calculus and Linear Algebra 18MAT11

Thus x 2  y 2  2  y  c2  0 (μ is the parameter) is the required orthogonal trajectory which is


also a coaxial system of circles.

23. Show that the family of parabolas y2  4a  x  xa  is self orthogonal.

Consider y 2  4a  x  a  . . .(1)

Differentiating w .r .t „x‟ we have

dy yy1 dy
2y  4a a  where y1 
dx 2 dx

Substituting this value of „a‟ in (1) we have,

 yy 
y 2  2 yy1  x  1  or y  2 xy1  yy12
 2 

Thus we have, y  2 xy1  yy12 …(2)

This is the D.E. of the given family.

1
Now replacing y1 by , (2) becomes
y1

2
 1   1  2 x y
y  2 x    y   or y  2
 y1   y1  y1 y1

 yy12  2 xy1  y ------------(3)

The DE. Of the orthogonal family which is same as (2) being the DE. Of the given family.

Thus the family of parabolas y 2  4a  x  a  is self-orthogonal.

24. Find the orthogonal trajectories of the family r  a 1  sin  

Wehave r  a 1  sin  

 log r  log a  log 1  sin  

Department of Mathematics Page 158


Calculus and Linear Algebra 18MAT11

Differentiating w .r .t „θ‟ we have,

1 dr cos 
 0
r d 1  sin 

dr d
Replacing by  r 2 we get,
d dr

1  2 d  cos 
 r 
r dr  1  sin 

d cos  1  sin  dr
 r  or d   by separating the variables.
dr 1  sin  cos  r

dr 1  sin 
Hence  r

cos 
d  c

 log r   sec d   tan  d  c

 log r  log sec  tan    log sec   c

 log  r  sec   tan   sec    log b  say 


 1 sin   1
 r   b
 cos  cos   cos 
r 1  sin   r 1  sin  
ie.,  b or b
cos 2
1  sin 2  
Thus r  b 1  sin   is the required orthogonal trajectory.

25. Find the orthogonal trajectories of the family 2a r  1  cos 

We have 2a r  1  cos 
 log 2a  log r  log 1  cos  
Differentiating w .r .t „θ’
1 dr sin 
 
r d 1  cos 

dr d
Replacing by  r 2 and simplifying RHS we have
d dr

Department of Mathematics Page 159


Calculus and Linear Algebra 18MAT11

1  2 d  2sin  2  cos  2 
 r 
r  dr  2sin 2  2 

d dr
r  cot  2  or tan  2  d 
dr r

dr
  tan  2 d  c
r 
log sec  2 
 log r  c
1 2 
 log  r sec2  2    log b  say 

Or r sec2  2  b or r cos2  2  b

This is the required equation of the orthogonal trajectory which can also be put in the form:

1
r. 1  cos   b or r 1  cos    2b
2

Thus 2b r  1  cos  is the required orthogonal trajectory.

26. Find the orthogonal trajectories of the family

r n cos n  a n
We have r n cos n  a n
 n log r  log  cos n   n log a

Differentiating w .r .t „θ‟ we have,

n dr   sin n  1 dr
 0   tan n
r d  cos n  r d

dr d
Replacing by  r 2 we have,
d dr

1  2 d  d
 r   tan n or r  tan n
r dr  dr

d dr
  By separating the variables.
tan n r

Department of Mathematics Page 160


Calculus and Linear Algebra 18MAT11

dr

r 
 cot n d  c

1
Ie. log r  log  sin n   c or n log r  log  sin n   nc
n

Ie. log  r n sin n   log b  say 

Thus r n sin n  b, is the required orthogonal trajectory.

27. Find the orthogonal trajectories of the family of curves

r n  a n cos n
r n  a n cos n
 n log r  n log a  log  cos n 

Differentiating w .r .t „θ‟ we have,

n dr n sin n 1 dr
 or   tan n
r d cos n r d

dr d
Replacing by  r 2 we get
d dr

1  2 d  d
 r    tan n or r  tan n
r dr  dr

d dr dr
      cot n d  c
tan n r r

1
I.e, log r  log  sin n   c or n log r  log  sin n   nc
n

 rn 
  log k  say   r  k sin n
n
I.e, log 
 sin n 

Thus r n  k sin n , is the required orthogonal trajectory.

28. Show that the orthogonal trajectories of the family of Cardioids

r  a cos2  2 is another family of cardioids r  b sin 2  2 .

We have r  a cos2  2

Department of Mathematics Page 161


Calculus and Linear Algebra 18MAT11

 log r  log a  2logcos  2

Differentiating w .r .t „θ‟ we have,

1 dr 1 1
 0  2. .  sin  2  .
r d cos  2  2

1 dr
   tan  2 
r d

dr d
Replacing by  r 2 we have,
d dr

1  2 d 
 r    tan  2 
r dr 
d
ie.,  r   tan  2 
dr

dr
Or cot  2  d  by separating the variables
r

dr
  cot  2 d  c
r 

 log r  2logsin  2  c

 r 
 log  2   log b  say 
 sin  2  

Thus r  b sin 2  2 is the required orthogonal trajectory.

29. Find the orthogonal trajectories of the family of curves , r  4a sec tan  .

We have r  4a sec  tan 

 log r  log  4a   log sec   log  tan  

Differentiating w .r .t „θ‟ we have,

1 dr sec  tan  sec 2 


 0 
r d sec  tan 

Department of Mathematics Page 162


Calculus and Linear Algebra 18MAT11

1 dr tan 2   sec 2 
 
r d tan 

dr d
Replacing by  r 2 we have,
d dr

1  2 d  tan 2   sec 2 
 r 
r dr  tan 

tan  dr
Or d  by separating the variables
tan   sec 
2 2
r

dr tan 
  d  c
r tan   sec2 
2

sin  cos 
 log r   d  c
sin  cos 2   1 cos 2 
2

 log r  log 1  sin 2    c


1
2

 log r 1  sin 2    log b  say   r 1  sin 2   b


 

Thus r 2 1  sin 2    b2 is the required orthogonal trajectory.

30. Using the concept of orthogonal trajectories show that the family of curves
r  a sin   cos  and r  b sin   cos  intersect each other orthogonally.

We shall show that the orthogonal trajectories of the first family

r  a sin   cos  is the other family r  b  sin   cos  .

Consider r  a  sin   cos  


 log r  log a  log  sin   cos  

Differentiating w .r .t „θ‟ we have,

1  2 d  cos   sin  d cos   sin 


 r  or r 
r dr  cos   sin  dr cos   sin 

Department of Mathematics Page 163


Calculus and Linear Algebra 18MAT11

cos   sin  dr
 d  By separating the variables.
cos   sin  r

dr cos   sin 
  d  c
r cos   sin 

 log r  log  cos  sin    c

 r 
 log   log k  say 
 cos   sin  
 r  k  cos   sin   or r  k  sin   cos  

Thus by denoting k  b, wehave r  b sin   cos  as required.

31. Show that the orthogonal trajectories of the family r  1  cos  a is the family
r 1  cos   b, where a, b are constants.

Consider r 1  cos    a
 log r  log 1  cos    log a

1 dr sin 
Differentiating w .r .t „θ‟ we have,  0
r d 1  cos 

dr d
Replacing by  r 2 and simplifying the second term we have,
d dr

1  2 d  2sin  2  cos  2 
 r 
r dr  2sin 2  2 

d dr
r  cot  2  Or tan  2  d  by separating the variables.
dr r

dr
  tan  2 d  c
r 
ie., log r  2 log sec  2   c

Department of Mathematics Page 164


Calculus and Linear Algebra 18MAT11

 r 
 log  2   log k  say 
 sec  2  
1
 r cos 2  2   k or r. 1  cos    k
2

 r 1  cos   2k and let 2k  b

Thus r 1  cos   b, is the required orthogonal trajectory.

32. Test for self Orthogonality r n  a sin n

Consider r  a sin n
n

 n log r  log a  log  sin n 

Differentiating w .r .t „θ‟ we have,

n dr n cos n 1 dr
 0 or  cot n
r d sin n r d

dr d
Replacing by  r 2 we have
d dr
1  2 d  d
 r   cot n or r  cot n
r dr  dr

dr
 tan n d  by separating the variables.
r

dr

r 
 tan n d  c

log  sec n 
ie., log r   c or n log r  log  sec n   nc
n

Ie., log  r n sec n   log b  say   r n sec n  b

 r n  b cos n is the required orthogonal trajectory and we conclude that the given family is
not self-orthogonal.

Department of Mathematics Page 165


Calculus and Linear Algebra 18MAT11

Exercise

Find the orthogonal trajectories of the following

1.
2.

3. Show that the family of ellipse is self-orthogonal


4. Show that the family of curves is self-orthogonal.
5.
6.
7.
8.
9.
10.

Newton’s Law of Cooling

The law states that, “the change of temperature of a body is proportional to the difference
between the temperature of a body and that of the surrounding medium”.

Let t10 c be the initial temperature of the body and t 20 C be the constant temperature of the
medium. Further let T 0C be the temperature of the body at any time t.

Then by Newton‟s law of cooling,

dT dT
 T  t2  Or  k T  t2 
dt dt

Where k is the constant of proportionality and the negative sign indicates the cooling of the
body with the increase of the time.

Since t10 C is the initial temperature of the body, mathematically it is equivalent to the condition
that T  t1 when t=0 or T(0)= t1.

We shall solve the first order DE.

Department of Mathematics Page 166


Calculus and Linear Algebra 18MAT11

dT
  k T  t2  with the condition T  0   t1
dt
dT

T  T2 
  kdt  

 log T  t2   kt   or T  t2   ekt 


c  e  cons tan t.
I.e., T  t2  ce  kt where

Applying the initial condition, T  t1 when t=0 we have,

t1  t2  ce0 or c  t1  t2

T  t2  t1  t2  ekt

Or T  t2   t1  t2  e kt

This is the expression for the temperature function.

Worked Problems

33. A body in air at 250 C cools from 1000 C to 750 C IN 1 MINUTE. Find the temperature of
the body at the end of 3 minutes.

According to the Newton‟s Law of cooling the expression for the temperature function at
any time is

T  t2  t1  t2  ekt

t1  100, t2  25 and T  75 when t  1.


We have by data,
T  25  75e kt

t1  100, t2  25 and T  75 when t  1.


We have by data,
T  25  75e  kt

By applying the initial condition, we have

50 2 3
75  25  75e k or e k   or ek   1.5
75 3 2

ek  1.5  k  loge 1.5  0.4055

Department of Mathematics Page 167


Calculus and Linear Algebra 18MAT11

We have to find T when t=3

Using the value of k we have T  25  75e0.4055t


 T t 3  25  75e1.2165  47.22

Thus the temperature of the body at the end of 3 minutes is 47.220 C

34. If the temperature of the air is 30o C and a metal ball cools from 100o C to 70o C in 15
minutes, find

How long will it take for the metal ball to reach a temperature of 40o C.

According to the Newton‟s law of cooling, the expression for the temperature function at
any time t is

T  t2  t1  t2  ekt

We have data, t 1 =100, t 2 =30 and T=70 when t=15

 T=30 +70 e  kt

By applying the initial condition we have,

40 4 7
70=30+70e 15k =  or e 15k   1.75
70 7 4

1
 1.75  15k  log 1.75 Or k  log e 1.75  0.0373
15 k
e 15

Hence we have T  30  70 e 0.0373t

We have to find‟t‟ when T=40.

1
 40=30+70e 0.0373 t
or e 0373 t 
7

Equivalently, e 0.0373 t  7  0.0373 t = log e 7

log e 7
 t=  52.17  52.2
0.0373

Thus we conclude that it will take 52. 2 minutes for the metal ball to reach a temperature of
40oC.

Department of Mathematics Page 168


Calculus and Linear Algebra 18MAT11

35. A bottle of mineral water at a room temperature of 72o F is kept in a refrigerator where the
temperature is

44o F. After half an hour, water cooled to 61o F

(i) What is the temperature of the mineral water in another half an hour ?

(ii) How long will it take to cool to 50oF?

According to the Newton‟s law of cooling, the expression for the temperature function at
any time t is

T  t 2 t1  t2  ekt

We have by data t 1  72, t2  44 and T= 61 when t=30 mins.

T  44  28e kt

By applying the initial condition we have,

30k 30 k 17 28
61=44+28e or e  or e 30 k 
28 17

28
  30k  log e  28 /17   k  0.0166
30 k
e 17

We have to find T when t=30+30=60 (mins)

 T(t 60)  44  28e


0.016660
 54.3

Thus the temperature of the mineral water after another half an hour ( 1hr=60 mts) is 54.3oF

(i) We have to find t when T=50


(ii) The expression becomes
3
50  44  28 e 0.0166t  e 0.0166t 
14
 t  92.8

Thus we conclude that it will take 93 minutes (about one and half hours) for the cooling of the
mineral water to

Department of Mathematics Page 169


Calculus and Linear Algebra 18MAT11

Exercise

1. A body at temperature is placed in a room of constant temperature at time


At the end of 5 minutes the body has cooled to a temperature .
a. Find the temperature of the body at the end of 10 minutes.
b. When will be the temperature of the body be

2. Water at temperature cools in 10minutes to in a room temperature of


a. Find the temperature of the water after 20 minutes.
b. When will be the temperature of the water be 4 .

3. Water at temperature takes in 5 minutes to warm up to in a room temperature


of . Find the temperature of the water after 20 minutes

Differential equations of first order and higher degree.

dy
If y=f(x), we use the notation  p throughout this unit.
dx

A differential equation of first order and n th degree is the form

A 0 p n  A1 p n 1  A2 p n  2    An  o ------------(1)

Where A0, A2, ... An are functions of x and y. This being a DE of first order, the associated
general solution will contain only one arbitrary constant. We proceed to discuss equation
solvable for p or y or x, wherein the problem is reduced to that of solving one or more
differential equations of first order and first degree. We finally discuss the solution of
Clairaut‟s equation.

Equation solvable for p

Supposing that the LHS of (1) is expressed as a product of n linear factors, then the equivalent
form of (1) is

 p  f1  x, y   p  f 2  x, y    p  f n  s, y   0 ----------(2)

  p  f1  x, y   0,  p  f 2  x. y   0,...  p  f n  x, y   0

All these are differential equations of first order and first degree. They can be solved by the
known methods. If F 1 x, y, c   0, F2  x, y, c   0,...Fn  x, y, c   0 respectively represents the

Department of Mathematics Page 170


Calculus and Linear Algebra 18MAT11

solution of these equations then the general solution is given by the product of all these
solutions.

Note: We need to present the general solution with the same arbitrary constant in each factor.

WORKED PROBLEMS
2
 dy   dy 
36. Solve:    7    12  0
 dx   dx 

The given equation is p 2  7 p  12  0


ie.,  p  3 p  4   0 or p  3, 4

dy
We have  3  y  3x  c or y  3 x  c  0
dx
dy
Also,  4  y  4x  c or y  4 x  c  0
dx

Thus the general solution is given by,  y  3x  c  y  4x  c   0

2
 dy  dy
37. Solve y     x  y   x  0
 dx  dx

The given equation is

yp 2   x  y  p  x  0

 x  y   x  y  4 xy
2

p
2y

p
 y  x   x  y
2y

yxx y yxx y
p or p
2y 2y

 p 1 or p  x y

dy
We have, 1 y  x  c or  y  x  c  0
dx

Department of Mathematics Page 171


Calculus and Linear Algebra 18MAT11

dy  x
Also,  or ydy  xdx  0   ydy   xdx  k
dx y

y 2 x2

2

2
k or y 2  x 2  2k or x 2
 y2  c  0

Thus the general solution is given by  y  x  c   x2  y 2  c   0

2
 dy 
38. Solve: xy     x 2  y 2   xy  0
dy
 dx  dx

The given equation is

xy p 2   x 2  y 2  p  xy  0

x  y2   x  y 2   4x2 y 2
2 2 2

p
2 xy

p
x 2
 y 2    x2  y 2 
2 xy

x2  y 2  x2  y 2 x2  y 2  x2  y 2
 p or p
2 xy 2 xy

x y
 p or p
y x

39. Solve: p 2  2 py cot x  y 2

p 2  2 py cot x  y 2  0 , by data.

2 y cot x  4 y 2 cot 2 x  4 y 2
p
2
2 y cot x  2 y cos ecx
p  y   cot x  cos ecx 
2

 p  y   cot x  cos ecx  or p  y   cot x  cos ecx 

Department of Mathematics Page 172


Calculus and Linear Algebra 18MAT11

dy
we have  y   cot x  cos ecx 
dx
dy
or   cos ecx  cot x  dx
y

dy
   cos ecx  cot x dx  k
y 

 log y  log tan  x 2  logsin x  k

 c tan  x 2  
Or log y  log   where log c  k
 sin x 

c tan  x 2  c tan  x 2  c c
y   
sin x 2sin  x 2  cos  x 2  2cos x 2 1  cos x
2

 y 1  cos x   c

dy
Also  y   cot x  cos ecx    y  cot x  cos ecx 
dx

dy
Or   cot x  cos ecx  dx
y

dy
   cot x  cos ecx dx  k
y 
ie., log y  log sin x  log tan  x 2   k

Or log  y sin xtan  x 2   log c where log c  k

sin  x 2 
 y.2sin  x 2  cos  x 2  c
cox  x 2 

 y.2sin2  x 2  c or y 1  cos x  c 

Thus the general solution is  y 1  cos x   c y 1  cos x   c  0

dy dx x y
40. Solve;   
dx dy y x

Department of Mathematics Page 173


Calculus and Linear Algebra 18MAT11

dy 1 dx
With the notation p  , we have 
dx p dy

The given equation is ,

1 x y p2 1 x y
p    or  
p y x p y x

x y
 p2  p    1  0
y x

2
 x y  x y
 y  x   y  x  4
   
p
2

 x y  x y
 y  x  y  x 
y
p    ; p  x or
2 y x

We have,

dy x
 or xdx  ydy  0   xdx   ydy  k
dx y

x2 y 2
  k or x 2  y 2  c  0, where c  2k
2 2

dy  y dy dx dy dx
Also,  or    k
dx x y x y x

log y  log x  k or log  xy   log c  xy  c  0, where log c  k

Thus the general solution is  x2  y 2  c   xy  c   0

Exercise

Department of Mathematics Page 174


Calculus and Linear Algebra 18MAT11

Equations solvable for P

(1)

(2)
(3)
(4)
(5)
(6)

Clairaut’s equation
The equation of the form
y  px  f  p  ------------------------(1)
.is known as Clairaut‟s equation.
This being in the form y  F  x, p  , that is solvable for y, we differentiate (1) w.r.t „ x‟

dy dp dp
 p  x  f ( p )
dx dx dx
dp
Or 0   x  f   p  
dx 

dp
Or 0   x  f   p  
dx 

dp
This implies that  0 and hence p  c
dx

Using, p=c in (1) we obtain the general solution of Clairaut‟s equation in the form

y  cx  f  c  --------------------------(2)

NOTE:

1. The general solution of Clairaut‟s equation is obtained by simply replacing „p‟ by „c‟.

2. The solution can be written instantly when the equation is in Clairaut‟s form or when the
equation is

Put into the Clairaut‟s form.

3. We can also obtain the singular solution.

Department of Mathematics Page 175


Calculus and Linear Algebra 18MAT11

Worked Problems

a
41. Solve; y  px 
p

The given equation is Clairaut‟s equation of the form y  px  f  p  whose general


solution is

y  cx  f  c 

a
Thus the general solution is y  cx 
c

Note: Singular solution

Differentiating partially w.r.t c we have,

a a
0 x 2
 x or c 2   c  a x
c x

y  cx   a / c  becomes,
Hence
y  a x.x  a x a or y  2 ax

Thus y 2  4ax is the singular solution.

42. Obtain the general and singular solution of the equation p  log  px  y 

p  log  px  y  , by data

Or e p  px  y or y  px  e p

Since y  px  e p is in the form of Clairaut‟s equation y  px  f  p  , the general solution is

y  cx  f  c  .

Thus the general solution is y  cx  ec

Now differentiating partially w.r.t c we have,

0  x  ec or ec  x  c  log x

Department of Mathematics Page 176


Calculus and Linear Algebra 18MAT11

Thus the singular solution is y  x log x  x

51. Solve  p  1  y  px   1
2

1 1
We have, y  px  or y  px 
 p  1  p  1
2 2

This equation being in the form of Clairaut‟s equation y  px  f  p  the general solution is

y  cx  f  c 

1
Thus the general solution is y  cx 
 c  1
2

43. Show that the equation xp 2  px  1  y  0 is Clairaut‟s equation. Hence obtain the
general and

singular solution.

xp 2  px  py  1  y  0, by data

,  xp2  px  1  y  p  1

xp  p  1  1
Ie., y 
p 1

1
Or y  px  ------------------------(1)
p 1

Is in the Clairaut‟s form y  px  f  p  whose general solution is y  cx  f  c 

1
Thus the general solution is y  cx 
c 1

Now differentiating partially w.r.t c we have,

1 1 1 1
0 x  c  1    c  1  or c  1
2
or
 c  1
2
x x x

Hence the general solution becomes

Department of Mathematics Page 177


Calculus and Linear Algebra 18MAT11

 1 
y   1 x  x
 x 

y x  x  x or x y  2 x

Thus the singular solution is  x  y   4 x


2

Remark: We can also obtain solution by the method: Solvable for y.

44. Modify the following equation into Clairaut‟s form. Hence obtain the associated general
and singular solutions.

xp 2  py  kp  a  0
xp 2  py  kp  a  0, by data

,  xp 2  kp  a  py

p  xp  k   a a
y or y  xp  k 
p p

 a
 y  px   k   ----------------------------(1)
 p

Here (1) is in the Clairaut‟s form y  px  f  p  whose general solution is

y  cx  f  c 

 a
Thus the general solution is y  cx   k  
 c

Now differentiating partially w.r.t „c‟ we have,

a a a
0 x or  x or c 2   c  a x
c2 c 2
x

Hence the general solution becomes,

y  a x x  k  a x a or y  k  2 ax

Department of Mathematics Page 178


Calculus and Linear Algebra 18MAT11

Thus the singular solution is  y  k   4ax


2

Remark: We can also obtain the solution in the method: Solvable for y.

45. Obtain the general solution and the singular solution of the following equation as Clairaut‟s
equation: xp3  yp 2  1  0

xp3  yp 2  1  0 by data

xp 3  1
yp 2  xp 3  1 or y
Ie., p2

1
Ie., y  px  is in the Clairaut‟s form y  px  f  p  whose general solution is
p2

y  cx  f  c 

1
Thus the general solution is y  cx 
c2

Now, differentiating partially w.r.t, c we have,


13
2 2 2
0  x  3 or c3   c   
c x x

Hence the general solution becomes,


13 23
2  x x2 3
y    x    or y  21 3 x 2 3  2 3
 x 2 2
ie., 2 y  2 x  x
23 23 23
or 2 y  3x 2 3
23

 2 y    3x 
3 23 3
23
or 4 y 3  27 x 2
Equivalently,

Thus the singular solution is 4 y 3  27 x 2

46. Solve the equation  px  y  py  x   2 p by reducing into Clairaut‟s form, taking the
substitutions X  x 2 , Y  y 2

Department of Mathematics Page 179


Calculus and Linear Algebra 18MAT11

dX
X  x2   2x
dx
dY
Y  y2   2y
dy

dy dy dY dX dY
p  and let P 
Now, dx dY dX dx dx

1 x
, p  .P.2 x or p  P
2y y

x
, p  P
Y

Consider  px  y  py  x   2 p
 X  X  X
ie.,  P X  Y  P Y  X2 P
 Y  Y  Y

 PX  Y  X
 P  1 X 2 P
OR Y Y

2P
,   PX  Y  P  1  2 P Or PX  Y 
P 1

2P
 Y  PX  is in the Clairaut‟s form and hence the associated general solution is
P 1

2c 2c
Y  cX  or y 2  cx 2 
c 1 c 1

2c
Thus the required general solution of the given equation is y 2  cx 2 
c 1

47. Find the general and singular solution of the equation x2  y  px   p2 y by reducing into
Clairaut‟s form using the substitution X  x 2 , Y  y 2

This problem is similar to Problem, Proceeding on the same lines we have

Department of Mathematics Page 180


Calculus and Linear Algebra 18MAT11

X dY
p P, where P 
Y dX The given equation

x 2  y  px   p 2 y becomes,
 X  X
X  Y  P X   p 2 Y
 Y  Y

X
Y  PX   XP 2
Or Y  PX  p 2
Y Y

 Y  PX  p 2 is in the Clairaut‟s form.

The associated general solution is Y  cX  c2

Thus the required general solution of the given equation is y 2  cx 2  c 2

Now differentiating w.r.t c partially we have,

0  x 2  2c or c   x2 2

Hence the general solution becomes,

 x4 x4
y2   or 4 y 2   x4
2 4

Thus the required singular solution is x 4  4 y 2  0

58. Solve p  cos y cos px  sin y sin px

p  cos y cos px  sin y sin px, by data.

 p  cos  y  px  or cos1 p  y  px

 y  px  cos 1 p is in the Clairaut‟s form y  px  f  p  whose general solution is

y  cx  f  c 

Thus the required general solution is y  cx  cos 1 c

Department of Mathematics Page 181


Calculus and Linear Algebra 18MAT11

Exercise

(1) Obtain the general and singular solution

(2) Obtain the general and singular solution


(3) Obtain the general
(4) Obtain the general and singular solution
(5) Obtain the general and singular solution
(6) Obtain the general and singular solution
(7) Solve the equation by reducing into clairaut‟s form
taking the substitutions
(8) Solve the equation by reducing into Clairaut‟s form, taking
the substitutions

(9) Solve e4 x (p 1)  e2 y p 2  0 by using the substitution X  e 2 x , Y  e 2 y

Department of Mathematics Page 182


Calculus and Linear Algebra 18MAT11

MODULE-5

LINEAR ALGEBRA

I. Introduction

II. Rank of a matrix by Echelon form.

III. Solution of system of linear equations-Consistency.

IV. Gauss-Elimination method.

V. Gauss-Jordan Method.

VI. Gauss-Seidel method.

VII. The eigen values and eigen vectors.

VIII. Rayleigh power method.

IX. Diagonalization of matrices.

Department of Mathematics Page 183


Calculus and Linear Algebra 18MAT11

I. Introduction

Linear algebra is central to almost all areas of mathematics. For instance, linear algebra is
fundamental in modern presentations of geometry, including for defining basic objects such as
lines, planes and rotations. Also, functional analysis may be basically viewed as the application
of linear algebra to spaces of functions. Linear algebra is also used in most sciences and
engineering areas, because it allows modeling many natural phenomena, and efficiently
computing with such models. For nonlinear systems, which cannot be modeled with linear
algebra, linear algebra is often used as a first-order approximation.

History

The study of linear algebra first emerged from the introduction of determinants, for solving
systems of linear equations. Determinants were considered by Leibniz in 1693, and
subsequently, in 1750, Gabriel Cramer used them for giving explicit solutions of linear
systems, now called Cramer's Rule. Later, Gauss further developed the theory of solving linear
systems by using Gaussian elimination, which was initially listed as advancement in geodesy.

The study of matrix algebra first emerged in England in the mid-1800s. In 1844 Hermann
Grassmann published his "Theory of Extension" which included foundational new topics of
what is today called linear algebra. In 1848, James Joseph Sylvester introduced the term matrix,
which is Latin for "womb". While studying compositions of linear transformations, Arthur
Cayley was led to define matrix multiplication and inverses. Crucially, Cayley used a single
letter to denote a matrix, thus treating a matrix as an aggregate object. He also realized the
connection between matrices and determinants, and wrote "There would be many things to say
about this theory of matrices which should; it seems to me, precede the theory of determinants".

Linear algebra took its modern form in the first half of the twentieth century, when many ideas
and methods of previous centuries were generalized as abstract algebra. The use of matrices in
quantum mechanics, special relativity, and statistics helped spread the subject of linear algebra
beyond pure mathematics. The development of computers led to increased research in efficient
algorithms for Gaussian elimination and matrix decompositions, and linear algebra became an
essential tool for modeling and simulations

Department of Mathematics Page 184


Calculus and Linear Algebra 18MAT11

II. Rank of a matrix by Echelon form.

First, using elementary row operations, the matrix A is reduced to echelon form. The number

of non-zero rows in the echelon form is equal to rank A . It denoted by   A .

1 2 4 3 
2 4 6 8 
1. Determine the rank of the matrix by reducing into echelon form A =  
 4 8 12 16 
 
1 2 3 4 
Solution:
1 2 4 3 
2 4 6 8 
Let A =  
 4 8 12 16 
 
1 2 3 4 
Operating R2  R2  2R1, R3  R3  4R1, R4  R4  R1 
1 2 4 3 
0 0  2 2 
 
0 0  4 4 
 
0 0  1 1 
 1 1 
Operating R2  2 R2 , R3  4 R3 
 1 2 4 3
0 0  1 1
 
0 0  1 1
 
0 0  1 1
Operating R3  R3  R2 , R4  R4  R2 
1 2 4 3
0 0  1 1 

0 0 0 0
 
0 0 0 0

   A  2

Department of Mathematics Page 185


Calculus and Linear Algebra 18MAT11

2. Determine the rank of the matrix by reducing into echelon form

Solution:
2 3  1 1
1 1  2 4 
Let A = 
3 1 3 2 
 
6 3 0 7 
Operating R1  R2 
1 1  2 4 
 2 3  1 1 
 
3 1 3 2 
 
6 3 0 7 
Operating R2  R2  2R1, R3  R3  3R1, R4  R4  6R1 
1 1  2 4 
0 5 3 7 

0 4 9 10 
 
0 9 12 17 
Operating R3  5R3  4R2 , R4  5R4  9R2 
1 1  2 4 
0 5 3 7 

0 0 33 22 
 
0 0 33 22 
 1 1 
Operating R3  11 R3 , R4  11 R4 
1 1  2 4 
0 5 3 7 

0 0 1 1
 
0 0 1 1
Operating R4  R4  R3 
1 1  2 4 
0 5 3 7 

0 0 1 1
 
0 0 0 0

   A  3

Department of Mathematics Page 186


Calculus and Linear Algebra 18MAT11

0 1  3 1
1 0 1 1 
3. Determine the rank of the matrix by reducing into echelon form A = 
3 1 0 2
 
1 1 2 0 
Solution:
0 1  3 1
1 0 1 1 
Let A = 
3 1 0 2
 
1 1 2 0 
Operating R1  R2 
1 0 1 1 
 0 1  3 1 
 
3 1 0 2
 
1 1  2 0 

Operating R3  R3  3R1, R4  R4  R1 
1 0 1 1 
0 1  3 1
 
0 1  3 1
 
0 1  3 1

Operating R3  R3  R2 , R4  R4  R2 
1 0 1 1
0 1  3 1

0 0 0 0
 
0 0 0 0

   A  2

Department of Mathematics Page 187


Calculus and Linear Algebra 18MAT11

 0 2 2 3 
2 5  4 6 
4. Determine the rank of the matrix by reducing into echelon form A  
 1 3 2 2 
 
2 4 1 6 
Solution:

 0 2 2 3 
2 5  4 6 
Let A 
 1 3 2 2 
 
2 4 1 6 
Operating R1  R2 
 2 5 4 6 
0 2  2 3 

 1 3 2 2 
 
2 4 1 6 
Operating R3  2R3  R1, R4  R4  R1 
2 5  4 6
0 2  2 3 

 0 1 0 2
 
0  1 3 0
Operating R3  2R3  R2 , R4  2R4  R2 
2 5  4 6
0 2  2 3 

0 0  2 7
 
0 0 4 3
Operating R4  R4  2R3 
2 5  4 6 
0 2  2 3 
 
0 0  2 7 
 
0 0 0 17 
   A  4

Department of Mathematics Page 188


Calculus and Linear Algebra 18MAT11

91 92 93 94 95 
92 93 94 95 96 

5. Determine the rank of the matrix by reducing into echelon form A  93 94 95 96 97 
 
94 95 96 97 98 
95 96 97 98 99 
Solution:
91 92 93 94 95 
92 93 94 95 96 
 
Let A  93 94 95 96 97 
 
94 95 96 97 98 
95 96 97 98 99 
Operating R2  R2  R1, R3  R3  R1, R4  R4  R1, R4  R5  R1 
91 92 93 94 95
1 1 1 1 1 

 2 2 2 3 2
 
3 3 3 3 3
 4 4 4 4 4 
 1 1 1 
Operating R3  2 R3 , R4  3 R3 , R4  4 R4 
91 92 93 94 95
1 1 1 1 1 
 
1 1 1 1 1 
 
1 1 1 1 1 
 1 1 1 1 1 
Operating R1  R2 
1 1 1 1 
1
91 92 93 95
94

1 1 1 1 
1
 
1 1 1 1
1 
 1 1 1 1 
1
Operating R2  R2  91R1, R3  R3  R1, R4  R4  R1, R4  R5  R1 
1 1 1 1 1
0 1 2 3 4 

0 0 0 0 0
 
0 0 0 0 0
 0 0 0 0 0 

Department of Mathematics Page 189


Calculus and Linear Algebra 18MAT11

   A  2
4 0 2 1
2 1 3 4 
6. Determine the rank of the matrix by reducing into echelon form A  
2 3 4 7
 
2 3 1 4
Solution:
4 0 2 1 
2 1 3 4 
Let A   
2 3 4 7 
 
2 3 1 4 
Operating R1  R2 
2 1 3 4
4 0 2 1 

2 3 4 7
 
2 3 1 4
Operating R2  R2  2R1, R3  R3  R1, R4  R4  R1 
2 1 3 4 
0 2  4 7 

0 2 1 3
 
0 2 2 0 
Operating R3  R3  R2 , R4  R4  R2 
2 1 3 4 
0 2  4 7 

0 0  3 4 
 
0 0  6 7 
Operating R4  R4  2R3 
2 1 3 4 
 0 2  4 7 
 
0 0  3 4 
 
0 0 0 1
   A  4

Department of Mathematics Page 190


Calculus and Linear Algebra 18MAT11

Exercise

Find the rank of the following matrices by reducing into echelon form.

 2 1 1  1 1 1 6
1 3 2   3 1 1   1 2 0 1  1 1
 2 1 4    3 4 1 2  2 5 
   4 1 2    3 1 1 8
1 11 14     2 3 2 5   
 1 1 1  2 2 3 7

1 1 2  2 3 5 4 
1 2 3  0 2 3 4 
   
0 1 1  4 8 13 12 

III. Solution of system of linear equations – consistency.

A non- homogeneous system of m equations in n unknowns represented by the equation


AX  B is consistent if and only if the coefficient matrix A and the augmented
matrix [ A : B] have the same rank.

1. T
est the following system for consistency and solve

x  2 y  3z  1
2 x  3 y  8z  2
x yz 3

Solution: The coefficient matrix A and the augmented matrix [ A : B] is given by

1 2 3 1 2 3 : 1
A  2 3 8 [ A : B]  2 3 8 : 2
   

1 1 1
 
1 1 1 : 3

Use the row operations R2  R2  2 R1 , R3  R3  R1

Department of Mathematics Page 191


Calculus and Linear Algebra 18MAT11

1 2 3 : 1 
[ A : B] 0 1 2 : 0 
 

0 1 2 : 2 

R3  R3  R2
1 2 : 1 
3
0 1 2 : 0 
 

0 0 4 : 2 

Rank of A= Rank of [ A : B]
There the given system of equations are consistent and has unique solution.
The resultant system of equations is given by
x  2 y  3z  1
 y  2z  0
4 z  2

By using back substitution, we obtain


1
z
2
 1 
y  2z  2    1
 2 
3 9
x  1  2 y  3z  1  2  
2 2
9 1
The solution is given by x  , y  1, z 
2 2
2. T
est the following system for consistency and solve

x yz 4
2x  y  z  1
x  y  2z  2

Solution: The coefficient matrix A and the augmented matrix [ A : B] is given by

1 1 1 1 1 1 4
:
A  2 1 1 [ A : B]  2 1 1 : 1
   

1 1 2 
1 1 2 : 2

Department of Mathematics Page 192


Calculus and Linear Algebra 18MAT11

Use the row operations R2  R2  2 R1 , R3  R3  R1

1 1 1 4  :
[ A : B] 0 1 3 : 7 
 

0 2 1 : 2 
R3  R3  2 R2
1 1 14  :
0 1 3 : 7 

 0 0 7 : 12 
Rank of A= Rank of [ A : B]
There the given system of equations are consistent and has unique solution.
The resultant system of equations is given by
x yz  4
 y  3 z  7
7 z  12

By using back substitution, we obtain


1 2
12 y 7 z  3  7
z 7 7
7
13 12 3
x  4  
7 7 7
3 13 12
The solution is given by x  , y  , z 
7 7 7

3. Test the following system for consistency and solve

3x  y  2 z  3
2 x  3 y  z  3
x  2y  z  4

Solution: The coefficient matrix A and the augmented matrix [ A : B] is given by

Department of Mathematics Page 193


Calculus and Linear Algebra 18MAT11

3 1 2 3 1 2 : 3 
A  2 3 1 [ A : B]   2 3 1 : 3
   

1 2 1 
1 2 1 : 4 

Use the row operations R1  R3


1 2 1 : 4 
[ A : B]   2 3 1 : 3
 

3 1 2 : 3 

Use the row operations R2  R2  2 R1 , R3  R3  3R1
1 2 1  : 4
[ A : B] 0 7 3 : 11
 

0 5 1 : 9 

Use the row operations R3  7 R3  5 R2
1 2 1 : 4
0 7 3 : 11
 

0 0 8 : 8 

Rank of A= Rank of [ A : B]
There the given system of equations are consistent and has unique solution.
The resultant system of equations is given by
x  2y  z  4
7 y  3 z  11
8 z  8

By using back substitution, we obtain

8 z  8 7 y  z  3 1 1
8 7 y    1z 1 3
z  1
8 7 y    1 1 3 (
7 y   1 4
y2
x  4  2y  z
x  4  2(2)  ( 1)  1
The solution is given by x  1, y  2, z  1

Department of Mathematics Page 194


Calculus and Linear Algebra 18MAT11

4. Test the following system for consistency and solve

x yz 2
3x  y  2 z  6
3x  y  z  18

Solution: The coefficient matrix A and the augmented matrix [ A : B] is given by

1 1 1 1 1 2 
1 :
A  3 1 2 [ A : B ]  3 1 2 : 6 
   

3 1 1
 
3 1 1 : 18

Use the row operations R2  R2  3R1 , R3  R3  3R1


1 1 1  : 4
[ A : B] 0 2 1 : 12 
 

0 4 2 : 24  
R3  R3  2 R2
1 1 1 :  2
0 2 1 : 12 
 

0 0 0 : 0  
Rank of A= Rank of [ A : B] is less than number unknowns
There the given system of equations are consistent and has infinitely many solutions
The resultant system of equations is given by
x yz 2
2 y  z  12

By using back substitution, we obtain


zk
x yz 2
2 y  z  12
x  yz2
2 y  z  12
2 y  k  12 x
 k  12   k  2
2
y
 k  12  k
2 x  4 
2

Department of Mathematics Page 195


Calculus and Linear Algebra 18MAT11

x  4 
k
, y 
 k  12  , z  k
The solution is given by
2 2

Exercise

Test the following for consistency and solve

1. x  y  2 z  4, 2 x  y  3z  9,3x  y  z  2

2. x  4 y  z  5, x  y  6 z  17,3x  y  z  4.

3. 5x  3 y  7 z  5,3x  10 y  2 z  9,7 x  2 y  10 z  5

4. 2 x  y  z  10,3x  2 y  3z  18, x  4 y  9 z  16

5. 4 x  2 y  6 z  8, x  y  3z  1,15x  3 y  9 z  21

6. x  3 y  8z  10  0,3x  y  4 z  0, 2 x  5 y  6 z  13

IV. Gauss Elimination Method


a1 x  b1 y  c1 z  d1
Consider a system of linear equations a2 x  b2 y  c2 z  d 2
a3 x  b3 y  c3 z  d3

The augmented matrix of the given system of equations is given by


 a1 b1 c1 : d1 
[ A : B]  
 a2 b2 c2 : d2 


 a3 b3 c3 : d3 

Department of Mathematics Page 196


Calculus and Linear Algebra 18MAT11

Reduce the augmented matrix into upper diagonal matrix by using elementary row operations
 a '1 b '1 c '1 : d '1 
 
[ A : B]   0 b'2 c '2 : d '2 
 0 0 c '3 : d '3 

By using resultant system of equations obtain the solution

1. Solve by Gauss elimination method


x  2y  z  3
3x  y  2 z  1
2 x  2 y  3z  2

Solution:

1 2 1 3 :
 2 : 1
Consider the augmented matrix [ A : B ]   3 1 

2 2 3 : 2

Use the row operations R2  R2  3R1 , R3  R3  2 R1


1 2 1 : 3 
[ A : B] 0 7 5 : 8 
 

 0  6 5 :  4 

R3  7 R3  6 R2
1 2 1 : 3
0 7 5 : 8
 

0 0 5 : 20 

The resultant system of equations is given by

Department of Mathematics Page 197


Calculus and Linear Algebra 18MAT11

7 y  5 z   8
  7 y  8  5 z
5 z  20 x  2y  z  3
 7 y  8  5(4)
20  x  3 2y  z
z 4  7 y  28
5 28  x  3  2(4)  4  1
y 4
7

x  1, y  4, z  4.

2. Solve by Gauss elimination method


x  2y  z  3
3x  3 y  2 z  5
3x  5 y  5 z  2

Solution:
1 2 1 : 3
 2 : 5
Consider the augmented matrix [ A : B ]   2 3 

 3 5 5 : 2 
Use the row operations R2  R2  2 R1 , R3  R3  3R1
1 2 1 : 3 
[ A : B] 0 1 0 : 1
 

0 11 2 : 7 

R3  R3  11R2
1 2 1 3 
:
0 1 0 : 1
 

0 0 2 : 4

The resultant system of equations is given by

x  2y  z  3
 y  1
2z  4

Department of Mathematics Page 198


Calculus and Linear Algebra 18MAT11

2z  4
 y  1
4
z 2 y 1
2
x  3  2(1)  2  1

x  1, y  1, z  2
3. Apply Gauss elimination method to solve the equations x  y  z  9;
2 x  y  z  0; 2 x  5 y  7 z  52.

Solution:
1 1 1 : 9 
Let  A : B    2 1  1 : 0 
 
 2 5 7 : 52 
Operating R2  R2  2R1
R3  R3  2R1
1 1 1 : 9 
 0  1  3 :  18 
0 3 5 : 34 
Operating R2   R2
1 1 1 : 9 
 0 1 3 : 18 
0 3 5 : 34 
Operating R3  R3  3R2
1 1 1 : 9 
 0 1 3 : 18 
0 0  4 :  20 
1
Operating R3  R3
4
1 1 1 : 9
  0 1 3 : 18 
 0 0 1 : 5 

y  3z  18 x yz 9
 z 5 y  18  3(5) x  935
y 3 x 1

Department of Mathematics Page 199


Calculus and Linear Algebra 18MAT11

4. Apply Gauss elimination method to solve the equations


2 x  y  z  10; 3x  2 y  3z  18; x  4 y  9 z  16.
Solution:
 2 1 1 : 10 
Let  A : B   3 2 3 : 18

1 4 9 : 16 
Operating R1  R3
1 4 9 : 16 

 3 2 3 : 18
 2 1 1 : 10 
Operating R2  R2  3R1
R3  R3  2R1
1 4 9 : 16 
 0  10  24 :  30 

0  7  17 :  22 
1
Operating R2  R2
2
R3   R3
1 4 9 : 16 
 0 5 12 : 15 
0 7 17 : 22 
Operating R3  5R3  7 R2
1 4 9 : 16 
 0 5 12 : 15 
0 0 1 : 5 

5 y  12 z  15 x  4 y  9 z  16
 z 5 5 y  15  12(5) x  16  36  45
y  9 x7

5. Apply Gauss elimination method to solve the equations


2 x  y  4 z  9; 4 x  11y  z  33; 8x  3 y  2 z  20.

Solution:
2 1 4 : 12 
Let  A : B   4 11  1 : 33 

 
8  3 2 : 20 

Department of Mathematics Page 200


Calculus and Linear Algebra 18MAT11

Operating R2  R2  2R1
R3  R3  4R1
2 1 4 : 12 
 0 9  9 : 9 

0  7  14 :  28
1 1
Operating R2  R2 , R3  R3
9 7
2 1 4 : 12 

 0 1  1 : 1 
0 1 2 : 4 
Operating R3  R3  R2
2 1 4 : 12 
 0 1  1 : 1 

0 0 3 : 3 

y  z 1 2 x  y  4 z  12
3z  3
 y  11 2 x  12  2  4
z 1
y2 x3

6. Apply Gauss elimination method to solve the equations


2 x  y  z  3; x  y  z  9; x  2 y  3z  8.

Solution:
2 1  1 : 3 
Let  A : B   1 1 1 : 9 
 
1  2 3 : 8 
Operating R1  R2
1 1 1 : 9 
  2 1  1 : 3
1  2 3 : 8 
Operating R2  R2  2R1
R3  R3  R1
1 1 1 : 9 
 0 3  2 : 1 
0 5  7 : 13
Operating R3  3R3  5R2

Department of Mathematics Page 201


Calculus and Linear Algebra 18MAT11

1 1 1 : 9 
  0 3  2 : 1 
 0 0  11 : 44 

3y  2z  1 x  2 y  3z  8
11z  44
 3 y  1  2(4) x  8  2(3)  3(4)
z4
y 3 x2

7. Apply Gauss elimination method to solve the equations


x  2 y  3z  2; 3x  y  4 z  4; 2 x  y  2 z  5.
Solution:
 2 1 1 : 10 
Let  A : B   3 2 3 : 18 

1 4 9 : 16 
Operating R1  R3
1 4 9 : 16 
 3 2 3 : 18 
 2 1 1 : 10 
Operating R2  R2  3R1
R3  R3  2R1
1 4 9 : 16 
 0  10  24 :  30 

0  7  17 :  22 
1
Operating R2  R2
2
R3   R3
1 4 9 : 16 
 0 5 12 : 15 
0 7 17 : 22 
Operating R3  5R3  7 R2
1 4 9 : 16 
 0 5 12 : 15 
0 0 1 : 5 

Department of Mathematics Page 202


Calculus and Linear Algebra 18MAT11

5 y  12 z  15 x  4 y  9 z  16
 z 5 5 y  15  12(5) x  16  4( 9)  9(5)
y  9 x7

8. Apply Gauss elimination method to solve the equations


5 x1  x2  x3  x4  4; x1  7 x2  x3  x4  2; x1  x2  6 x3  x4  5; x1  x2  x3  4 x4  6.

Solution:
5 1 1 1: 4 
1 7 1 1 : 12 
Let  A : B   
1 1 1 1 : 5
 
1 1 1 4 :  6
Operating R1  R4
1 1 4 :  6
1
17 1 1 : 12 

11 1 1 : 5
 
51 1 1 : 4
Operating R2  R2  R1 , R3  R3  R1 , R4  R4  5R1
1 1 4 : 6 
1
0 6 0  3 : 18 

0 0 5 3 : 1 
 
0  4  4  19 : 34 
1
Operating R2  R2
3
1 1 1 4 :  6
0 2 0  1 : 6 
 
0 0 5  3 : 1 
 
0  4  4  19 : 34 
Operating R4  R4  2 R2
1 1 1 4 :  6 
0 2 0  1 : 6 
 
0 0 5  3 : 1 
 
0 0  4  21 : 46 
Operating R4  5R4  4 R3

Department of Mathematics Page 203


Calculus and Linear Algebra 18MAT11

1 1 1 4 : 6 
0 2 0  1 : 6 

0 0 5 3 : 1 
 
0 0 0  117 : 234 

5 x3  3x4  1 2 x2  x4  6 x1  x2  x3  4 x4  6
117 x4  234
 5 x3  1  3(2) 2 x2  6  2 x1  6  1  2  8
x4  2
x3  1 x2  2 x1  1

9. Apply Gauss elimination method to solve the equations


2 x  y  3z  1; 3x  4 y  5z  0; x  3 y  6 z  0.

Solution:
2  1 3 : 1
Let  A : B   3 4  5 : 0 

 
1 3  6 : 0 
Operating R1  R3
1 3  6 :0 
  3 4  5 : 0 
 2  1 3 : 1 
Operating R2  R2  3R1 , R3  R3  2 R1
1 3  6 :0 
 0 13  23 : 0 
0  7 15 : 1 
Operating R3  13R3  7 R1
1 3  6 :0 
 0 13  23 : 0 
0 0 34 : 13

34 z  13 13 y  23 z  0 x  3y  6z  0
 13 13 y  23 z x  6z  3y
z
34 23 x 1
y
34

Department of Mathematics Page 204


Calculus and Linear Algebra 18MAT11

Exercise

Apply Gauss elimination method to solve the equations.

1. x  2 y  2 z  5;2 x  y  3z  6;3x  y  2 z  4

2. x  2 y  z  3;3x  y  2 z  1;2 x  2 y  3z  2.

3. x  y  2 z  4;2 x  y  3z  9;3x  y  z  2.

4. x  4 y  z  5; x  y  6 z  17;3x  y  z  4.

5. 5x  3 y  7 z  5;3x  10 y  2 z  9;7 x  2 y  10 z  5

6. 2 x  y  z  10;3x  2 y  3z  18; x  4 y  9 z  16.

V. Gauss-Jordan Method

Consider a system of linear equations


a1 x  b1 y  c1 z  d1
a2 x  b2 y  c2 z  d 2
a3 x  b3 y  c3 z  d3

The augmented matrix of the given system of equations is given by


 a1 b1 c1 : d1 
[ A : B]  
 a2 b2 c2 : d2 


 a3 b3 c3 : d3 

Reduce the augmented matrix into diagonall matrix by using elimentary row operations
 a '1 b '1 c '1 : d '1 
 
[ A : B]   0 b'2 c '2 : d '2 
 0 0 c '3 : d '3 

By using resultant system of equations obtain the solution

Department of Mathematics Page 205


Calculus and Linear Algebra 18MAT11

1. Solve the following system of equations by Gauss-Jordan method

x  y  z  9, 2 x  3 y  4 z  13, 3x  4 y  5 z  40

Solution:
Consider augmented matrix of the given system of equations
1 1 19  :
[ A : B]   2 3 4 : 13 
 
3
 4 5 : 40 

Operating R2  R2  2 R1 , R3  R3  3R1
1 1 1 : 9 
0 5 2 : 5
 

0 1 2 : 13 

Operating R2  R3
1 1 1 : 9 
0 1 2 : 13 
 

0 5 2 : 5

Operating R1  R1  R2 , R3  R3  5R2
1 0 1 4 :
0 1 2 : 13 
 

0 0 12 : 60 
R3
Operating R3 
12
1 0 1 4 
:
0 1 2 : 13 
 

0 0 1 : 5  
Operating R1  R1  R3 , R2  R2  2 R3

1 0 0 : 1
0 1 0 : 3
 

0 0 1 : 5
x  1, y  3, z  5

Department of Mathematics Page 206


Calculus and Linear Algebra 18MAT11

2. Solve the following system of equations by Gauss-Jordan method


x  y  z  9, 2 x  y  z  0, 2 x  5 y  7 z  52
Solution:
Consider augmented matrix of the given system of equations
1 1 1 9 
:
[ A : B]   2 1 1 : 0 
 

2 5 7 : 52 

Operating R2  R2  2 R1 , R3  R3  3R1
1 1 1 : 9 
0 1 3 : 18
 

0 3 5 : 34 

Operating R2  (1) R2
1 1 1 :9 
0 1 3 : 18 
 

0 3 5 : 34 

Operating R1  R1  R2 , R3  R3  3R2
1 0 2 : 9 
0 1 3 : 18 
 

0 0 4 : 20 

R3
Operating R3 
4
1 0 2 9 :
0 1 3 : 18 
 

0 0 1 : 5  
Operating R1  R1  2 R3 , R2  R2  3R3

1 0 0 : 1
0 1 0 : 3
 

0 0 1 : 5
x  1, y  3, z  5

Department of Mathematics Page 207


Calculus and Linear Algebra 18MAT11

Exercise

Solve the following system of equation Gauss-Jordan method

1. 2 x  y  3z  9, x  y  z  6, x  y  z  2.

2. 2 x  y  z  10,3x  2 y  3z  18, x  4 y  9 z  16.

3. 20 x  y  2 z  17,3x  20 y  z  18, 2 x  3 y  20 z  25.

4. x  3 y  3z  16, x  4 y  3z  18, x  3 y  4 z  19.

5. 3x  y  z  3, 2 x  8 y  z  5, x  2 y  9 z  8.

VI. Gauss-Seidel method

1. Apply Gauss-Seidel iterative method to solve the equations 20 x  y  2 z  17;


3x  20 y  z  18; 2 x  3 y  20 z  25.

Solution:

Given system is diagonally dominant

Therefore given equations may be rewritten as

1
x k 1  17  y k  2 z k  (1)
20
1
y k 1   18  3x k 1  z k  (2)
20
1
z k 1   25  2 x k 1  3 y k 1  (3)
20 
 
With initial values x0 , y 0 , z 0   0,0,0 

Stage1; k  0 equation (1)-(3) yield

1  1
x1  17  y  0  2 z  0   17  0  0  0.85
20 20
1  1
y 1   18  3x1  z  0    18  0  2.55  1.0275
20 20
1  1
z 1   20  25  1.7  3.0805  1.0109
1 1 
25  2 x  3 y 
20 

 
 first approximation is x1 , y 1 , z 1   0.85, 1.0275,1.0109 

Department of Mathematics Page 208


Calculus and Linear Algebra 18MAT11

Stage2; k  1 equation (1)-(3) yield

1  1
x 2   17  y 1  2 z 1   17  1.0275  2.0219  1.0025
20 20
1  1
y  2   20  18  1.0109  3.0075  0.9998
 2 1 
18  3 x  z 
20 
1  1
z  2   25  2 x 2  3 y  2    25  2.005  2.9994  0.9998
20 20
 
 Second approximation is x 2 , y 2 , z  2  1.0025, 0.9998,0.9998

Stage3; k  2 equation (1)-(3) yield

1  1
x  3   20 17  0.9998  1.9996  0.9999
 2  2 
17  y  2 z 
20 
1  1
y  3   18  3x3  z  2    18  0.9998  2.9999  1.0000
20 20
1  1
z  3   25  2 x3  3 y 3    25  1.9999  3.0000  1.0000
20 20

Hence the solutions is x  1, y  1, z  1.

2. Apply Gauss-Seidel iterative method to solve the equations x  y  54 z  110;


27 x  6 y  z  85; 6 x  15 y  2 z  72.
1
x k 1  85  6 y k  z k  (1)
20
1
y k 1  72  6 x k 1  2 z k  (2)
15
1
z k 1  110  x k 1  y k 1  (3)
54 

 
With initial values x0 , y 0 , z 0   0,0,0 

Stage1; k  0 equation (1)-(3) yield

1 
x1   85  6 y  0  z  0   2.5556
27
1
y    72  6 x1  2 z  0   3.5111
1

15
1 
z 1  110  x1  y 1   1.9247
54

Department of Mathematics Page 209


Calculus and Linear Algebra 18MAT11

 
 First approximation is x1 , y 1 , z 1   2.5556,3.5111,1.9247 

Stage2; k  1 equation (1)-(3) yield

1 
x 2   85  6 y 1  z 1   2.5133
27
1
y  2  72  6 x 2  2 z 1   3.5381
15
1 
z  
2
110  x 2  y  2   1.9250
54
 
 Second approximation is x 2 , y  2 , z  2   2.5133,3.5381,1.9250 

Stage3; k  2 equation (1)-(3) yield

1 
x  3   85  6 y  2  z  2   2.4332
27
1
y    72  6 x3  2 z  2   3.5701
3

15
1 
z  3  110  x3  y 3   1.9259
54
 
 Third approximation is x3 , y3 , z 3   2.4332,3.5701,1.9259 

Stage4; k  3 equation (1)-(3) yield

1 
x 4   85  6 y 3  z 3   2.4261
27
1
y  4  72  6 x3  2 z  2   3.5728
15
1 
z  4  110  x 4  y  4   1.9259
54

Hence the solutions is x  2.4261, y  3.5728, z  1.9259.

3. Apply Gauss-Seidel iterative method to solve the equations 10 x  y  z  12;


2 x  10 y  z  13; 2 x  2 y  10 z  14.

1
x k 1  12  y k  z k  (1)
10 

Department of Mathematics Page 210


Calculus and Linear Algebra 18MAT11

1
y k 1  13  2 x k 1  z k  (2)
10
1
z k 1  14  2 x k 1  2 y k 1  (3)
10
 
With initial values x0 , y 0 , z 0   0,0,0 

Stage1; k  0 equation (1)-(3) yield

1 
x1  12  y  0  z  0   1.2
10
1
y 1  13  2 x1  z  0   1.06
10
1
z 1  14  2 x1  2 y 1   0.948
10
 
 First approximation is x1 , y 1 , z 1  1.2,1.06, 0.948

Stage2; k  1 equation (1)-(3) yield

1 
x 2  12  y 1  z 1   0.9992
10
1
y  2  13  2 x 2  z 1   1.0054
10
1
z    14  2 x 2  2 y  2   0.9991
2

10
 
 Second approximation is x 2 , y  2 , z  2   0.9992,1.0054, 0.9991

Stage3; k  2 equation (1)-(3) yield

1 
x  3  12  y  2  z  2   0.9996
10
1
y    13  2 x3  z  2   1.0002
3

10
1
z 3  14  2 x3  2 y 3   1.0000
10

Hence the solutions is x  1, y  1, z  1.

Department of Mathematics Page 211


Calculus and Linear Algebra 18MAT11

4. Apply Gauss-Seidel iterative method to solve: 5x  2 y  z  12;


x  4 y  2 z  15; x  2 y  5z  20 carryout 4 iteration, taking initial approximation to the
solution as 1, 0,3

Solution:

1
x k 1  12  2 y k  z k  (1)
5
1
y k 1  15  x k 1  2 z k  (2)
4
1
z k 1   20  x k 1  2 y k 1  (3)
5
 
With initial values x0 , y 0 , z 0  1,0,3
Stage1; k  0 equation (1)-(3) yield

1
x1  12  2 y  0  z  0   1.8
5
1
y  15  x1  2 z  0   1.8
1
4
1
z 1   20  x1  2 y 1   2.92
5
 
 First approximation is x1 , y 1 , z 1  1.8,1.8, 2.92 

Stage2; k  1 equation (1)-(3) yield

1
x 2  12  2 y 1  z 1   1.096
5
1
y  2  15  x 2  2 z 1   2.016
4
1
z  2   20  x 2  2 y  2   2.9744
5
 
 Second approximation is x 2 , y  2 , z  2  1.096, 2.016, 2.9744 

Stage3; k  2 equation (1)-(3) yield

1
x3  12  2 y  2  z  2   0.9987
5
1
y 3  15  x3  2 z  2   2.0131
4
1
z 3   20  x 3  2 y 3   2.9950
5

Department of Mathematics Page 212


Calculus and Linear Algebra 18MAT11

 
 Third approximation is x3 , y 3 , z 3   0.9958, 2.0036, 2.9994 
Stage4; k  3 equation (1)-(3) yield

1
x 4  12  2 y 3  z 3   0.9958
5
1
y  4  15  x 4  2 z 3   2.0036
4
1
z  4   20  x 4  2 y  4   2.9994
5

Hence the solutions is x  0.9958 1, y  2.0036 2, z  2.9994 3.

Exercise

Apply Gauss-Seidel iterative method to solve the following equations.

1. 10 x  y  z  12; x  10 y  z  12; x  y  10z  12.

2. 5x  2 y  z  12; x  4 y  2 z  15; x  2 y  5z  0.

3. 28x  4 y  z  32; 2 x  77 y  4 z  35; x  3 y  10 z  24

4. 5x  2 y  z  4; x  6 y  2 z  1;3x  y  5z  13.

5. 9 x  2 y  4 z  20; x  10 y  4 z  6; 2 x  4 y  10 z  15

6. 8x  y  z  8; 2 x  4 y  z  4; x  3 y  5z  5

VII. Eigen values and eigenvectors

The roots of the characteristic equation A   I  0 , are called the eigenvalues of the matrix A .

These eigenvalues may be real or complex, and not necessarily distinct. Corresponding to each

eigen value  i , there is a non-zero solution x  xi of A   I x  0 ; xi is called the

eigenvector of A corresponding to the eigen value  i .

Department of Mathematics Page 213


Calculus and Linear Algebra 18MAT11

1 4
1. Find eigen values and eigenvectors of matrix A   
2 3
Solution:

The character equation of A is A   I  0

1  4
0
2 3
3    3   2  8  0
 2  4  5  0
  1   5  0
  1,   5 are the eigen values

Case1.   1

1   4   x  0 
 2 
 3     y  0 
 2 4  x  0 
 2 4  y   0 
    

2x  4 y  0
x  2 y
x y

2 1
 x  2 
 y    1 
   
2 
Thus X1    is the eigen vector corresponding to   1
 1 
Case2.   5

1   4   x  0 
 2 
 3     y  0 
4 4  x  0 
 2 2  y   0 
    

4 x  4 y  0
x y
x y

1 1

Department of Mathematics Page 214


Calculus and Linear Algebra 18MAT11

 x  1 
 y   1 
   
1 
Thus X 2    is the eigen vector corresponding to   5.
1 

 1 3
2. Find eigen values and eigenvectors of matrix A   
 2 4

Solution:
The character equation of A is A   I  0

1   3
0
2 4
 1   4     8  0
 2  3  2  0
  1   2  0
  1,   2 are the eigen values

Case1.   1

 1   3   x  0 
 2     y   0 
 4    
 2 3    0 
x
2 3  y   0 
    

2 x  3 y  0
2x  3 y
x y

3 2
3 
Thus X1    is the eigen vector corresponding to   1
2 

Case2.   2

 1   3   x  0 
 2 
 4     y  0 

Department of Mathematics Page 215


Calculus and Linear Algebra 18MAT11

 3 3  x  0 
2 2  y   0 
    

3x  3 y  0
x y
x y

1 1
1 
Thus X 2    is the eigen vector corresponding to   2
1 

Important points to be remember

If the characteristic polynomial of an 3 × 3 matrix A is written as

 3  p1  2  p2   p3  0
then the coefficients p1 , p2 , p3 can be computed using
p1    D   Diogonal sum
p2    Dm   Minor of the Diogonal
p3  A  Determinant

1 1 3
3. Find eigen values and eigenvectors of matrix A  1 5 1
3 1 1
Solution:

The character equation of A is A   I  0


1  1 3
1 5 1 0 (1)
3 1 1 

The characteristic polynomial of an 3 × 3 matrix A is given by

 3  p1  2  p2   p3  0 (2)
Where
p1    D   1  5  1  7

Department of Mathematics Page 216


Calculus and Linear Algebra 18MAT11

p2    Dm  
5 1 1 3 1 1
  = 5 1  1  9  5 1  0
1 1 3 1 1 5

1 1 3
p3  A  1 5 1  1 5  1  11  3  3 1  15   36
3 1 1

Then eq. (2) becomes

 3  7 2  0   36  0
To solve this cubic equation we shall find root by inspection By putting

  0  0  0  36  36  0
  1  1  7  36  28  0
  1  1  7  36  30  0
  2  8  28  36  16  0
  2  8  28  36  0

Thus   2 is a root. The other two roots can found by synthetic division as follow

-2 1 -7 0 36

0 -2 18 -36

1 -9 18 0

 2  9   18  0
  3   6  0
  2, 3, 6 are the eigen values

Case1.   2

1   1 3   x  0 
 1 5 1   y   0 

 3 1 1     z  0 

3 1 3  x   0 
1 7 1  y   0 
    
3 1 3  z  0 

Department of Mathematics Page 217


Calculus and Linear Algebra 18MAT11

3x  y  3z  0 (i)
x  7y  z  0 (ii)
3x  y  3z  0 (iii)
Applying the rule of cross multiplication for (i) and (ii)
x y z
 
1 3 3 3 3 1
7 1 1 1 1 7
x y z
 
1  21 3  3 21  1
x y z
 
20 0 20

 x   20   1
y   0    0
     
 z   20   1 
 1
Thus X 1   0  is the eigen vector corresponding to   2
 1 

Case2.   3

1   1 3   x  0 
 1 5 1   y   0 

 3 1 1     z  0 

 2 1 3   x  0 
 1 2 1   y   0 
    
 3 1 2   z  0 

2 x  y  3z  0 (i)
x  2y  z  0 (ii)
3x  y  2 z  0 (iii)
Applying the rule of cross multiplication for (i) and (ii)
x y z
 
1 3 3 2 2 1
2 1 1 1 1 2

Department of Mathematics Page 218


Calculus and Linear Algebra 18MAT11

x y z
 
1  6 3  2 4  1
x y z
 
5 5 5

 x   1 
y   1 
   
 z   1 
 1 
Thus X 2   0  is the eigen vector corresponding to   3
  1 

Case3.   6

1   1 3   x  0 
 1 5 1   y   0 

 3 1 1     z  0 

 5 1 3   x  0 
 1 1 1   y   0 
    
 3 1 5  z  0 
5x  y  3z  0 (i)
x y z 0 (ii)
3x  y  z  0 (iii)
Applying the rule of cross multiplication for (i) and (ii)
x y z
 
1 3 3 5 5 1
1 1 1 1 1 1
x y z
 
1  3 3  5 5 1
x y z
 
4 8 4

 x  1  1 
y   2  Thus X 3   2  is the eigen vector corresponding to   6
   
 z   1   1 

Department of Mathematics Page 219


Calculus and Linear Algebra 18MAT11

 8 6 2 
4. Find eigen values and eigenvectors of matrix A   6 7 4 
 2 4 3 
Solution:

The character equation of A is A   I  0


8 6 2
6 7 4  0 (1)
2 4 3

The characteristic polynomial of an 3 × 3 matrix A is given by

 3  p1  2  p2   p3  0 (2)
Where
 
p1   D  8  7  3  18
4 6
p2    Dm  
7 8 2 8
  = 45
4 3 2 3 6 7

8 6 2
p3  A  6 7 4  8  5   6  10   2 10   0
2 4 3

Then eqs(1) becomes

 3  18 2  45   0  0
 3  18 2  45   0
    3   15  0

  0, 3, 15 are the eigen values

Case1.   0

8   6 2   x  0 
 6 7   4   y   0 
    
 2 4 3     z  0 

Department of Mathematics Page 220


Calculus and Linear Algebra 18MAT11

8x  6 y  2 z  0 
 x y z
6 x  7 y  4 z  0    
24  14 12  32 56  36
4 x  4 y  3z  0  

x y z
 
10 20 20

 x  1 
 y   2 
   
 z   2 
1 
Thus X 1   2  is the eigen vector corresponding to   0
 2 

Case2.   3

8   6 2   x  0 
 6 7   4   y   0 
    
 2 4 3     z  0 

5x  6 y  2 z  0 
 x y z
6 x  4 y  4 z  0    
24  8 12  20 20  36
4 x  4 y  0 z  0 

x y z
 
16 8 16

 x  2 
 y   1 
   
 z   2 
2 
Thus X 2   1  is the eigen vector corresponding to   3
  2 
Case3.   15

 7 6 2   x  0 
 6 8 4   y   0 
    
 2 4 12   z  0 

Department of Mathematics Page 221


Calculus and Linear Algebra 18MAT11

7 x  6 y  2 z  0 
 x y z
6 x  8 y  4 z  0    
24  16 12  28 56  36
4 x  4 y  12 z  0 

x y z
 
40 40 20

x   2 
 y    2
   
 z   1 
2 
Thus X 3   2  is the eigen vector corresponding to   15
 1 

2 0 1 
5. Find eigen values and eigenvectors of matrix A   0 2 0 
 1 0 2 
Solution:

The character equation of A is A   I  0


2 0 1
0 2 0 0 (1)
1 0 2

The characteristic polynomial of an 3 × 3 matrix A is given by

 3  p1  2  p2   p3  0 (2)
Where
p1    D   2  2  2  6

p2    Dm  
2 0 2 1 2 0
  = 11
0 2 1 2 0 2

2 0 1
p3  A  0 2 0  2  4   1 2   6
1 0 2

Department of Mathematics Page 222


Calculus and Linear Algebra 18MAT11

then equation (1) becomes

 3  6 2  11  6  0
 1  2  3  0
  1, 2, 3 are the eigen values

Case1.   1

2   0 1   x  0 
 0 2 0   y   0 

 1 0 2     z  0 

x  0y  z  0 
 x y z
0 x  y  0 z  0   
0 1 0  0 1  0
x  0 y  z  0 

x y z
 
1 0 1

 x   1
 y   0 
   
 z   1 
  1
Thus X 1   0  is the eigen vector corresponding to   1
 1 

Case2.   2

2   0 1   x  0 
 0 2 0   y   0 

 1 0 2     z  0 

0x  0 y  z  0
0x  0 y  0z  0
x  0 y  0z  0
From above equations, we get
x  0, y  k , z  0

Department of Mathematics Page 223


Calculus and Linear Algebra 18MAT11

 x  0 
 y   k 
   
 z   0 
0
Thus X 2   1  is the eigen vector corresponding to   2
 0 
Case3.   3

2   0 1   x  0 
 0 2 0   y   0 

 1 0 2     z  0 

 x  0 y  z  0
 x y z
0x  y  0z  0    
0 1 0  0 1 0
x  0 y  z  0 

x y z
 
1 0 1

 x  1 
 y   0 
   
 z   1 
1
Thus X 3  0  is the eigen vector corresponding to   3
 1 
 7 2 0 
6. Find eigen values and eigenvectors of matrix A   2 6 2 
 0 2 5 
Solution:

The character equation of A is A   I  0


7 2 0
2 6 2  0 (1)
0 2 5

The characteristic polynomial of an 3 × 3 matrix A is given by

Department of Mathematics Page 224


Calculus and Linear Algebra 18MAT11

 3  p1  2  p2   p3  0 (2)
Where
p1    D   7  6  5  18
2 2
p2    Dm  
6 7 0 7
  = 99
2 5 0 5 2 6

7 2 0
p3  A  2 6 2  7  30  4   2  10  0   162
0 2 5

Then eqs(1) becomes

 3  18 2  99   162  0
   3  6  9  0
  3, 6,9. are the eigen values

Case1.   3

7   2 0   x  0 
 2 6   2   y   0 
    
 0 2 5     z  0 

4x  2 y  0z  0 
 x y z
2 x  3 y  2 z  0    
4  0 8  0 12  4
0x  2 y  2z  0  

x y z
 
4 8 8

 x  1 
 y   2 
   
 z   2 
1 
Thus X 1   2  is the eigen vector corresponding to   3
 2 

Department of Mathematics Page 225


Calculus and Linear Algebra 18MAT11

Case2.   6

7   2 0   x  0 
 2 6   2   y   0 
    
 0 2 5     z  0 

x  2 y  0z  0 
 x y z
2 x  0 y  2 z  0    
4 2 4
0 x  2 y  z  0 

x y z
 
4 2 4

 x  2 
 y   1 
   
 z   2 
2 
Thus X 2   1  is the eigen vector corresponding to   3
 2 
Case3.   9

7   2 0   x  0 
 2 6   2   y   0 
    
 0 2 5     z  0 

2 x  2 y  0 z  0 
 x y z x y z
2 x  3 y  2 z  0      
4 4 2 4 4 2
0x  2 y  z  0  

 x  2 
 y    2
   
 z   1 
2 
Thus X 3   2  is the eigen vector corresponding to   9
 1 

Department of Mathematics Page 226


Calculus and Linear Algebra 18MAT11

Exercise

Find eigen values and eigenvectors of the following matrices.


 4 3
1.  
2 9

1 4 
2. 3 2 
 

 1 2 
3.  5 4 
 

 2 1 1 
 1 1 2 
4.  
 1 2 1 

 6 2 2 
 2 3 1
5.  
 2 1 3 

1 1 3 
1 5 1 
6.  
3 1 1

 5 2 0 
 
7.  2 6 2 
 0 2 7 

Department of Mathematics Page 227


Calculus and Linear Algebra 18MAT11

VIII. The Rayleigh power method

1. Find the largest eigen values and the corresponding eigenvector of the matrix
2 0 1 
A   0 2 0 
 1 0 2 
Solution:

X 0  1, 0, 0 as initial approximation. Then we find


T
We take

 2 0 1  1   2 1 
X (1)
 AX (0)
  0 2 0  0   0   2 0    (1) X (1)
     
1 0 2  0 0   0.5

 2 0 1  1   2.5 1 
X (2)
 AX (1)
  0 2 0  0   0   2.5 0    (2) X (2)
     
1 0 2  0.5  2  0.8

 2 0 1  1   2.8 1 
X (3)
 AX (2)
  0 2 0  0   0   2.8 0    (3) X (3)
     
1 0 2  0.8  2.6  0.93

 2 0 1  1   2.93  1 
X (4)
 AX (3)
  0 2 0  0   0   2.93 0    (4) X (4)
     
1 0 2  0.93  2.86  0.98

 2 0 1  1   2.98 1 
X (5)
 AX (4)
  0 2 0  0   0   2.98 0 
     
1 0 2  0.98  2.96  0.99

1 
Thus approximation largest eigen values   2.98 and  0  is the corresponding
 0.99 
eigenvector.

Department of Mathematics Page 228


Calculus and Linear Algebra 18MAT11

2. Find the largest eigen values and the corresponding eigenvector of the matrix
 2 1 0 
A   1 2 1
 0 1 2 
Solution:

X 0  1,1,1 as initial approximation. Then we find


T
We take

 2 1 0  1 1  1 
X (1)
 AX (0)
  1 2 1 1  0   1 0    (1) X (1)
     
 0 1 2  1 1  1 

 2 1 0  1   2   1
X (2)  AX (1)   1 2 1 0    2   2  1   (2) X (2)
 0 1 2  1   2   1 

 2 1 0   1   3   0.75
X (3)
 AX (2)
  1 2 1  1   4   4  1    (3) X (3)
     

 0 1 2   1   3   0.75 

 2 1 0   0.75  2.5  0.71


X (4)
 AX (3)
  1 2 1  1    3.5  3.5  1    (4) X (4)
     
 0 1 2   0.75   2.5  0.71

 2 1 0  0.71  2.42   0.708


X (5)  AX (4)   1 2 1  1    3.42  3.42  1    (5) X (5)
     
 0 1 2  0.71  2.42   0.708

 2 1 0  0.708  2.416  0.707 


X (6)
 AX (5)
  1 2 1  1    3.416   3.416  1    (6) X (6)
     
 0 1 2  0.708  2.416  0.707 

 2 1 0  0.707   2.4146  0.707 


X (7)
 AX (6)
  1 2 1  1    3.4146   3.4146  1 
     
 0 1 2  0.707   2.4146  0.707 

Department of Mathematics Page 229


Calculus and Linear Algebra 18MAT11

0.707 
Thus approximation largest eigen values   3.3146 and  1  is the corresponding
0.707 
eigenvector.

3. Use the power method to find the largest eigen values and the corresponding eigenvector of
 25 1 2 
the matrix A   1 3 0 
 2 0 4 
Solution:

X  0  1, 0, 0 as initial approximation. Then we find


T
We take

 25 1 2  1   25 1 
X (1)
 AX (0)
  1 3 0  0   1   25 0.04    (1) X (1)
     

 2 0 4  0   2  0.08

 25 1 2  1   25.2  1 
X (2)
 AX (1)
  1 3 0  0.04   1.12   25.2 0.04    (2) X (2)
     
 2 0 4  0.08  1.68  0.07 

 25 1 2  1   25.2  1 
X (2)  AX (1)   1 3 0  0.04   1.12   25.2 0.04 
     
 2 0 4  0.07  1.72  0.07 

1 
Thus approximation largest eigen values   25.2 and  0.04  is the corresponding
 0.07 
eigenvector.

4. Find the largest eigen values and the corresponding eigenvector of the matrix
 6 2 2 
A   2 3 1
 2 1 3 
Solution:
X  0  1,1,1 as initial approximation. Then we find
T
We take

Department of Mathematics Page 230


Calculus and Linear Algebra 18MAT11

 6 2 2  1 6  1 
X (1)  AX (0)   2 3 1 1  0   6 0    (1) X (1)
     
 2 1 3  1  4  0.67 

 6 2 2  1  7.34  1 
X (2)
 AX (1)
  2 3 1 0    2.67   7.34  0.36    (2) X (2)
      
 2 1 3  0.67   4.01  0.55 

6 2 2  1  7.82  1 
X (3)
 AX (2)
  2 3 1  0.36    3.63  7.82  0.46    (3) X (3)
     
 2 1 3  0.55   4.01  0.51 
6 2 2  1  7.94  1 
X (4)  AX (3)   2 3 1  0.46    3.89   7.94  0.49    (4) X (4)
     
 2 1 3  0.51  3.99  0.5 

 6 2 2  1  7.98  1 
X (5)
 AX (4)
  2 3 1  0.49    3.97   7.98  0.5   (5) X (5)
     
 2 1 3  0.5  3.99  0.5 

 6 2 2  1  8  1 
X (6)
 AX (6)
  2 3 1  0.5   4  8  0.5
     
 2 1 3  0.5   4  0.5 

1 
Thus approximation largest eigen values   8 and  0.5  is the corresponding eigenvector.
 0.5 

Exercise
Find the largest eigen values and the corresponding eigenvector of the matrix by Rayleigh
Power method.
 4 5
1. 
1 2 
 6 2 2 
2.  2 3 1
 
 2 1 3 

Department of Mathematics Page 231


Calculus and Linear Algebra 18MAT11

2 0 1
3. 0 2 0 

 1 0 2 
1 6 1
4. 1 2 0 

 0 0 2 
5 0 1
5. 0 2 0 

1 0 5 
10 2 1
6.  2 10 1 

 2 1 10 

IX. Diagonalization of a square matrix

Working procedure for diagonalization of a square matrix A of order 2  2 (Spectral matrix)

Step1. Find eigen values 1 , 2 . of a square matrix A

Step2. Find eigenvectors X 1 , X 2 . corresponding to the eigen values 1 , 2 .


x x2 
Step3.Write the modal matrix P   X 1 X 2    1
 y1 y2 

Step4. Find inverse matrix P 1 


 Adj P  .
P
 0 
Step5.Finally obtain diagonal matrix D  P 1 AP   1 
 0 2 

Computation of power of a square matrix A

Diagonalization of square matrix A also helps us to find the powers of


A: A 2 , A 3 , A 4 ......etc
In general A n  P D n P 1

Department of Mathematics Page 232


Calculus and Linear Algebra 18MAT11

 4 1
1. Diagonalise the matrix A   
 2 3
Solution:
The character equation of A is A   I  0
4 1
0
2 3
(4   )(3   )  2  0
 2  7  10  0
  2  5  0
  2,   5 are the eigen values
4   1   x  0 
 2 
 3     y  0 
(4   ) x  y  0 
        (1)
2 x  (3   ) y  0 
Case1. Put   2
We get
2x  y  0
x  2 y
x y

2 1
 x  2 
 y    1 
   
2 
Thus X1    is the eigen vector corresponding to   1
 1 
Case2.   5
x  y  0
2x  2 y  0
x y
x y

1 1
 x  1 
 y   1 
   
1 
Thus X 2    is the eigen vector corresponding to   5.
1 
 1 1
Thus the model matrix is given by P  [ X 1 , X 2 ]   2 1
 

Department of Mathematics Page 233


Calculus and Linear Algebra 18MAT11

1 1
 
1 AdjP  2 1 
P  
P 3
1 1 1 1 4 1  1 1  2 0
The diagonal matrix is D  P AP   
3 2 1  2 3  2 1 0 5

 19 7 
2. Diagonalise the matrix A   
 42 16
Solution:

The character equation of A is A   I  0

19   7
0
42 16  
(19   )(16   )  294  0

 2  3  10  0
   2   5  0
  2,   5 are the eigen values

 A   I  X   0
19   7   x  0 
 42 
 16     y  0 

(19   ) x  7 y  0 
        (1)
42 x  (16   ) y  0 
Case1. Put   2
We get
21x  7 y  0
42 x  14 y  0
3x  y  0
x y

1 3

Department of Mathematics Page 234


Calculus and Linear Algebra 18MAT11

 x  1 
 y   3 
   
1 
Thus X1    is the eigen vector corresponding to   2
3 
Case2.   5
14 x  7 y  0
42 x  21 y  0
2x  y
x y

1 2
 x  1 
 y   2 
   
1 
Thus X 2    is the eigen vector corresponding to   5.
2 
1 1 
Thus the model matrix is given by P  [ X 1 , X 2 ]  3 2
 
 2 1
 
1 AdjP  3 1   2 1 
P    
P 1  3 1
1 2 1  19 7  1 1  2 0 
The diagonal matrix is D  P AP   3 1 42 16 3 2  0 5
     

1 4
3. Obtain a modal matrix and the corresponding spectral matrix for the matrix A   
2 3
Solution:
The character equation of A is A   I  0

1  4
0
2 3

3    3   2  8  0
 2  4  5  0

Department of Mathematics Page 235


Calculus and Linear Algebra 18MAT11

  1   5  0
  1,   5 are the eigen values

Case1.   1

1   4   x  0 
 2 
 3     y  0 

 2 4  x  0 
 2 4  y   0 
    
2x  4 y  0
x  2 y
x y

2 1
 x  2 
 y    1 
   
2 
Thus X1    is the eigen vector corresponding to   1
 1 
Case2.   5
1   4   x  0 
 2 
 3     y  0 

4 4  x  0 
 2 2  y   0 
    
4 x  4 y  0
x y
x y

1 1
 x  1 
 y   1 
   
1 
Thus X 2    is the eigen vector corresponding to   5.
1 
 2 1
Thus the model matrix is given by P  [ X 1 , X 2 ]  1 1
 
Department of Mathematics Page 236
Calculus and Linear Algebra 18MAT11

1 1
 
1 AdjP 1 2  1 1 1
P    
P 3 3 1 2 
1 1 1 1 1 4 2 1  1 0
The diagonal matrix is D  P AP   
3 1 2  2 3 1 1  0 5

 1 3
4. Diagonalise the matrix A   
 2 4
Solution:

The character equation of A is A   I  0

1   3
0
2 4
 1   4     8  0
 2  3  2  0
  1   2  0
  1,   2 are the eigen values
Case1.   1
 1   3   x  0 
 2 
 4     y  0 
2 3  x  0 
2 3  y   0 
    

2 x  3 y  0
2x  3 y
x y

3 2
3 
Thus X1    is the eigen vector corresponding to   1
2 

Case2.   2

 1   3   x  0 
 2 
 4     y  0 

Department of Mathematics Page 237


Calculus and Linear Algebra 18MAT11

 3 3  x  0 
2 2  y   0 
    

3x  3 y  0
x y
x y

1 1
1 
Thus X 2    is the eigen vector corresponding to   2
1 
 3 1
Thus the model matrix is given by P  [ X1 , X 2 ]   
 2 1
 1 1
 
AdjP  2 3   1 1
P 1    
P 1  2 3 
1  1 1  1 3 3 1 1 0
The diagonal matrix is D  P AP  2 3  2 4 2 1  0 2
     

Exercise

Diagonalise the following matrices

4 1
1. 3
 2 
5 3
2. 3
 5
5 4
3. 1
 2 
1 2
4. 2
 1 

Department of Mathematics Page 238

You might also like